Download as docx, pdf, or txt
Download as docx, pdf, or txt
You are on page 1of 174

TEST 3

LISTENING
PART 1:
Complete the notes below. Write NO MORE THAN THREE WORDS AND/OR A NUMBER for each
answer.
Customer’s name: 1. ______________________
Customer number: 45993
Product: 2. ______________________
Cost of the product: 3. ______________________
Language(s) in manual: 4. ______________________
Order number: 5. ______________________
Customer’s phone number: 348-28841
PART 2: Circle the letter before the best answers according to the recording
1. What is one of Shawn's concerns about the dog?
A. It can be somewhat aggressive.
B. It eats too much food at one time.
C. The dog might mess on his carpet.
2. What is Shawn supposed to do between 3:00-4:00 p.m. for the dog?
A. Take the dog for some exercise with a Frisbee.
B. Feed him an afternoon doggie treat for a snack.
C. Let the dog watch a program on television.
3. Which point is NOT true about the cat?
A. The cat becomes a little moody at times.
B. The cat enjoys listening to rock music.
C. The cat will run away if it gets outside the house.
4. What can we infer from the conversation on the snake's reaction to Shawn?
A. The snake appears to warm up to him.
B. The snake doesn't care for Shawn at all.
C. The snake is extremely shy of Shawn.
5. What is Shawn's final response to Norman?
A. Norman should seek someone who is well-trained with animals.
B. Norman should take his animals to the zoo for special care.
C. Norman agrees to watch them for a lot of extra money.
PART 3: Decide whether the following statements are true (T) or false (F) or not given (NG)
1. Jackie was surprised that David had problems placing his order.
2. David needs to order the software for his office.
3. Jackie gives him the 25% discount even though he's not ordering online.
4. Jackie tells David that the free microphones are usually not very good quality.
5. David buys both versions of the software.
PART 4:
1. Both Hilary and Mike comment that not everybody ___________ what SETI is.
2. Hilary says that SETI researchers all believe in ___________ planets.
3. She uses the snail example to illustrate __________ involved.
4. NASA sent messages on spacecraft during the ____________.
5. Current SETI techniques involve _____________ rather than sending signals.
6. In an ideal world, NASA would like to find ___________ and not just simple plant life.
7. It's possible to get depressed by the continued ___________ but Hilary remains optimistic.
8. One reason an advanced form of life cannot be detected is that they may have ___________
themselves.
9. Hilary mentions nuclear proliferation, overpopulation and ___________ as examples of how
our civilization puts itself in danger.
10. If Hilary met any aliens, she would ask them how it was possible for them ____________.
LEXICO-GRAMMAR
PART 1: Choose the best options
1. Mr and Mrs Jones are such permissive parents that their son can __________ no matter what he
insists.
A. jump on the bandwagon B. go against the grain
C. make their blood boil D. wind them around his little finger
2. You are bound to find information on the stock market crash of 1987 in the newspaper________.
A. files B. archives C. records D. collections
3. According to psychiatrists, many violent criminals harbour feelings of ________and insecurity.
A shortage B insufficiency C scarcity D inadequacy
4. If the market does not improve, these businesses may not be able to ________ their debts, and we
may see more bankruptcies.
A. disgorge B. milk C. indemnify D. service
5. The mini dress was _________, but now it is making a comeback.
A. a fad once thought to be finished B. once thought a fad to be finishing
C. thought a fad to be finished once D. once thought to be a finishing fad
6. Lying can be used to ________ the character of a brother or to flatter a friend.
A. backbite B. impugn C. spurn D. abnegate
7. It is ________that the Minister of Justice should be accused of corruption
A sarcastic B sardonic C ironic D cynical
8. You can’t believe a word that woman says –she is a ________liar.
A. dedicated B. committed C. compulsive D. devoted
9. He was so lonely he _________ the sound of a human voice.
A. angled for B. ached for C. egged on D. fussed over
10. I think that this painting can be ________in a number of different but equally valid ways.
A dissected B interpreted C translated D rendered
11. The photo would have been wonderful had it not been________focus.
A beyond B far from C out of D without
11. The photo would have been wonderful had it not been________focus.
A beyond B far from C out of D without
12. Tina would rather you _______ your son as she is too tired after the three-day conference.
A. had picked up B. did pick up C. would pick up D. picked up
13. Despite his reputation as a tough guy, he admits that he was scared ________ when he first
arrived in New York.
A. bananas B. dozy C. witless D. half dead
14. The protest began nearly two weeks ago and was initially sparked by plans to ________a park to
build a shopping center.
A. desiccate B. unmake C. extirpate D. bulldoze
15. The man in the market was selling leather coats very cheaply: they were such bargains
that were soon __________.
A. snapped up B. cleared off C. done for D. sold up
16. If you ask an artist how she draws so well, she is _________ to say, “Well, I just look at
something and draw what I see”.
A. prone B. predisposed C. loathe D. liable
17. The police arranged to __________ with their informant at a disused warehouse.
A. decouple B. rendezvous C. reconnoitre D. corroborate
18. I’ve just heard that argument before and quite frankly it just doesn’t ______!
A. face the music B. hit the nail on the head
C. carry weight D. hold water
19. Unfortunately, ________ stamping out the drugs trade, these programs are simply forcing drug
growers to move into more inaccessible regions.
A. besides B. rather than C. as for D. in converse to
20. I don’t like to make friends with the person who always ____ people _____ behind their
backs.
A. slag/off B. slap/ around C. strike/out D. hate/on
PART 2: Find and correct ten mistakes/ errors in the following passage
Having a roof over your head is a basic human need, but there are 1.2 billion people in the world with
adequate housing. This may change thanks for a revolutionary, low-cost use of 3D printers to construct
houses. With 3D printing, materials are joined together or reinforcing by using a computer-controlled
device to create a three-dimensional object. Two companies have joined forces to try and ease
homeless around the world by building affording homes using 3D printing. Tech company ICON has
developed a method for printing a one-floor, 60-square-meter house out of cement in a day for just
$10,000. This is a fraction of both the time and cost needing to build a similar construction using
conventional methods.
ICON has teamed up with the non-profit, internationally housing organization New Story. Together,
they will start building homes in developing countries. Their joint venture will see 100 new homes
constructing in El Salvador next year. New Story's co-founder Alexandria Lafci acknowledged that the
100 homes were just a drop in the ocean. She said: "There are over 100 million people living in slum
conditions, in what we call survive mode." She also saw possibilities for 3D-printed houses to become
regular in richer countries in years to come. However, she said that for the moment: "The tech is ready
now to print very high-quality, safe homes in the places we're building."
PART 3: Fill in each gap with the corect form of the word given
Over half a century ago, scientists found that they could record the electrical
signals of the brain at work. What at first appeared a random hotchpotch of
activity became a pattern of elegant waves (1. RHYTHM) determined. Ever
since, scientists have wondered whether the secrets of our thoughts,
(2. PERCEIVE) and even (3. CONSCIOUS) itself might be hidden in the patterns of
our brain waves. The question of why we have brain waves is, (4. ARGUE), as
hotly debated today as it was when the patterns were discovered. But the
meaning, and even the existence, of fast rhythms in the alert brain is highly
(5. CONTROVERSY).
What is problematic is that you cannot perceive these rhythms directly, they are
so well hidden in the noise created by other brain activity, but many
(6. SEARCH) now hold the (7. CONVINCE) that the significance of these brain
waves should not be (8. ESTIMATE).
The latest suggestion is that the rhythms could be (9. DECIDE) in detecting
progresses going on in different regions of the brain. Some believe that these
rhythms might even interact, and in doing so help the brain to package
information into (10. COHERE) thoughts. How we bring together these related
signals of the brain is a puzzle as yet unsolved.

READING
PART 1: Choose the best options to complete the passage
FRIDAY THE THIRTEENTH
Police are hunting for a hit-and-run driver who knocked a teenage cyclist off her bike in East
Street. Sarah Tucker, 17, had a lucky escape on Friday, 13th May, when she was sent reeling
by a black Volvo on her way home from work.
She bruised her thigh and shoulder and her bicycle was 1_______. The driver stopped for a
moment but then drove off without 2________ a name or address and before Sarah could
get his number. “I tried to get out of his way, but I couldn’t” she said. “Everyone at work
kept going on about it being Friday 13th. I’m not a bit 3________ and wouldn’t change any
of my plans just because Friday 13th is supposed to be unlucky, I don’t usually take any
4_______ of that sort of thing but I will now. I think I’ll stay in bed.”
The accident 5________ at the junction with Westwood Road at about 6.30pm as Sarah
was making her 6________ home to the Harley Estate.
The Volvo pulled out of Westwood onto Henley Road in front of the teenager’s bicycle. “He
could at 7_________ have helped her up. I don’t see why he should get away with it,” said
her father, Derek, “Sarah was lucky. I don’t know why the driver didn’t see her. He can’t
have been 8_______ attention. It is unfortunate that nobody took down the number.”
Though still too 9_______ to ride a bike, Sarah was able to go back to 10_________ in
Marlow on Monday.
1. A. damaged B. harmed C. devastated D. crashed
2. A. noting B. presenting C. leaving D. suggesting
3. A. irrational B. superstitious C. unreasonable D. prejudiced
4. A. notice B. consider C. note D. care
5. A. came across B. turned up C. finished up D. took place
6. A. route B. way C. course D. path
7. A. once B. most C. least D. best
8. A. giving B. paying C. attracting D. providing
9. A. discouraged B. confused C. overcome D. shaken
10. A. work B. job C. post D. employment
PART 2: Fill in each gap with ONE suitable word
Perhaps the greatest value of biodiversity is yet unknown. Scientists have discovered and named only
1.75 million species — less than 20 percent of those (1)________ to exist. And of those identified,
only a (2) ________ has been examined for potential medicinal, agricultural or (3) ________ value.
Much of Earth’s great biodiversity is (4) ________ disappearing, even before we know what is missing.
Most biologists agree that life on Earth now is (5) ________ with the most severe extinction episode
since the event that drove the dinosaurs to (6) _______ 65 million years ago. Species of plants,
animals, fungi and microscopic organisms such as (7) ________ are being lost at alarming rates – so
many, in fact, that biologists estimate that three (8) ________ go extinct every hour. Scientists around
the (9) _________ are cataloging and studying global biodiversity in hopes that they might (10)
_______ understand it, or at least slow the rate of loss.
PART 3: Read the passage and choose the best options to answer the questions
Vincent Van Gogh was born in Groot Zundert, in The Netherlands on March 30th 1853, to parents
Theodorus Van Gogh, a preacher, and Ana Cornelia Carbentus. In 1869 at the age of 16, Van Gogh
began a career, not as a painter, but as an art dealer with the firm Goupil & Cie. He spent 7 years at
Goupil & Cie where daily contacts with works of art kindled his appreciation of paintings and
drawings. Gradually Vincent lost interest in his work and decided to try his hand teaching at a Catholic
School for boys. His growing interest religion and his desire to help the poor eventually drove him to
become a clergyman. In 1878, he became a lay preacher in one of the most impoverished regions in
Western Europe: the coal-mining district of the Borinage in Belgium. Vincent sympathized with the
poverty-stricken miners and gave away most of his food and clothing to ease their burdened lives. His
extreme commitment to the miners drew disfavor from the church, which dismissed him of his post.
Vincent, however, decided to remain with the miners and began to paint them and their
families, chronicling their harsh conditions.
Soon after, thanks to his brother’s financial help, Vincent decided to go to Brussels in 1880 to begin
studies in art. During the next 10 years, Vincent painted around 872 painting. In 1882, Vincent began
living with Clasina Maria Hoornik, also known as Sien, and her children, in the Hague. Their volatile
personalities and the strain of living in complete poverty created stormy relationship. Vincent was
devoted to Sien and her children, but art always came first. As his drawing and painting skills
advanced, his relationship with Sien deteriorated and they parted ways in September 1883.
In 1886, Vincent moved in with his brother-Theo in Paris where he met Paul Gauguin and various
other artists, who had a tremendous impact on his ongoing evolution as an artist. Never truly happy in
large cities, Vincent decided to move to Aries Province in the south of France, where he rented a
studio and invited Paul Gauguin to live with him. In December 1888, Vincent experienced a
psychotic episode in which he cut off a piece of his left ear. After his episode, he was in and out of
asylums for the next year. It was thought that Van Gogh was actually epileptic and that is why people
thought he had fits of insanity throughout his life. He painted one of his best-known painting, Starry
Night, during one of his stays in the asylum. In mid-1890, Vincent left the asylum and spent the last
few months of his life in Auvers, France. On July 27th 1890, Vincent Van Gogh shot himself in the
chest. Two days later he died with his younger brother-Theo by his side. He left behind a wonderful
array of paintings that make him one of the most influential painters of our time.
1: The word "chronicling" in paragraph 1 is closest meaning to "_________".
A. recording B. classifying C. suffering D. colouring
2: Van Gogh decided to become a clergyman due to ________.
A. his love of art B. his teachings at the Catholic School for boys
C. the Goupil & Cie art dealer firm D. his developing enthusiasm in Christianity
3: Vincent's extreme commitment to the miners resulted in ________.
A. his painting the miners and their families
B. his sympathizing with the miners
C. the church discharging him of his duty
D. the church giving food and clothing to the miners
4: The word "deteriorated" in paragraph 2 is closest meaning to "_________".
A. detested B. became worse and worse
C. developed D. turned down
5: Vincent and Clasina's relationship was stormy because of ________.
A. his devotion to art B. the stress of living in poor conditions
C. her children D. their tame personalities
6: Vincent first went into an asylum because ________.
A. he painted Starry Night B. he was epileptic
C. he cut off part of his ear D. he was insane throughout his life
7: Vincent went to Aries because ________.
A. he did not get along with his brother, Theo
B. he wanted to live in Gauguin's house in Aries
C. he wanted to live in a bigger city
D. he disliked big cities
8: Vincent moved to Paris ________.
A. in order to evolve as an artist B. to live with his brother
C. to meet other artists D. to live with Paul Gauguin
9: Van Gogh was believed to be ________.
A. insane B. a loner C. epileptic D. an inadequate painter
10: The word "episode" in paragraph 3 refers to "_________".
A. important event in his life B. unbelievable fact
C. unfortunate time D. one of several parts of a story on television
PART 4: Read the passage and do the tasks follow
Thomas Harriot
The Discovery of Refraction
A When light travels from one medium to another, it generally bends, or refracts. The law of refraction
gives us a way of predicting the amount of bending. Refraction has many applications in optics and
technology. A lens uses refraction to form an image of an object for many different purposes, such as
magnification. A prism uses refraction to form a spectrum of colors from an incident beam of light.
Refraction also plays an important role in the formation of a mirage and other optical illusions. The
law of refraction is also known as Snell’s Law, named after Willobrord, Snell, who discovered the law in
1621. Although Snell’s sine law of refraction is now taught routinely in undergraduate courses, the
quest for it spanned many centuries and involved many celebrated scientists. Perhaps the most
interesting thing is that the first discovery of the sine law, made by the sixteenth-century English
scientist Thomas Harriot (1560-1621), has been almost completely overlooked by physicists, despite
much published material describing his contribution.
B A contemporary of Shakespeare, Elizabeth I, Johannes Kepler and Galilei Galileo, Thomas Harriot
(1560-1621) was an English scientist and mathematician. His principal biographer, J. W. Shirley, was
quoted saying that in his time he was “England’s most profound mathematician, most imaginative and
methodical experimental scientist” . As a mathematician, he contributed to the development of
algebra, and introduced the symbols of ”>” , and ”<” for ”more than” and ”less than.” He also studied
navigation and astronomy. On September 17, 1607, Harriot observed a comet, later Identified as
Hailey-s. With his painstaking observations, later workers were able to compute the comet’s orbit.
Harriot was also the first to use a telescope to observe the heavens in England. He made sketches of
the moon in 1609, and then developed lenses of increasing magnification. By April 1611, he had
developed a lens with a magnification of 32. Between October 17, 1610 and February 26, 1612,he
observed the moons of Jupiter, which had already discovered by Galileo. While observing Jupiter, s
moons,he made a discovery of his own: sunspots, which he viewed 199 times between December 8,
1610 and January 18, 1613. These observations allowed him to figure out the sun’s period of rotation.
C He was also an early English explorer of North America. He was a friend of the English courtier and
explorer Sir Walter Raleigh, and travelled to Virginia as a scientific observer on a colonising expedition
in 1585. On June 30, 1585, his ship anchored at Roanoke Island ,off Virginia. On shore,Harriot
observed the topography, flora and fauna, made many drawings and maps, and met the native people
who spoke a language the English called Algonquian. Harriot worked out a phonetic transcription of
the native people’s speech sounds and began to learn the language, which enabled him to converse to
some extent with other natives the English encountered. Harriot wrote his report for Raleigh and
published it as A Briefe and True Report of the New Found Land of Virginia in 1588. Raleigh gave
Harriot his own estate in Ireland, and Harriot began a survey of Raleigh’s Irish holdings. He also
undertook a study of ballistics and ship design for Raleigh in advance of the Spanish Armada’s arrival.
D Harriot kept regular correspondence with other scientists and mathematicians, especially in England
but also in mainland Europe, notably with Johannes Kepler. About twenty years before Snell’s
discovery, Johannes Kepler (1571-1630) had also looked for the law of refraction, but used the early
data of Ptolemy. Unfortunately, Ptolemy’s data was in error, so Kepler could obtain only an
approximation which he published in 1604. Kepler later tried to obtain additional experimental results
on refraction, and corresponded with Thomas Harriot from 1606 to 1609 since Kepler had heard
Harriot had carried out some detailed experiments. In 1606, Harriot sent Kepler some tables of
refraction data for different materials at a constant incident angle, but didn’t provide enough detail for
the data to be very useful. Kepler requested further information, but Harriot was not forthcoming,
and it appears that Kepler eventually gave up the correspondence, frustrated with Harriot’s
reluctance.
E Apart from the correspondence with Kepler, there is no evidence that Harriot ever published his
detailed results on refraction. His personal notes, however, reveal extensive studies significantly
predating those of Kepler, Snell and Descartes. Harriot carried out many experiments on refraction in
the 1590s, and from his notes it is clear that he had discovered the sine law at least as early as 1602.
Around 1606, he had studied dispersion in prisms (predating Newton by around 60 years), measured
the refractive indices of different liquids placed in a hollow glass prism, studied refraction in crystal
spheres, and correctly understood refraction in the rainbow before Descartes.
F As his studies of refraction, Harriot’ s discoveries in other fields were largely unpublished during his
lifetime, and until this century, Harriot was known only for an account of his travels in Virginia
published in 1588, ,and for a treatise on algebra published posthumously in 1631. The reason why
Harriot kept his results unpublished is unclear. Harriot wrote to Kepler that poor health prevented him
from providing more information, but it is also possible that he was afraid of the seventeenth
century’s English religious establishment which was suspicious of the work carried out by
mathematicians and scientists.
G After the discovery of sunspots, Harriot’ s scientific work dwindled. The cause of his diminished
productivity might have been a cancer discovered on his nose. Harriot died on July 2, 1621, in London,
but his story did not end with his death. Recent research has revealed his wide range of interests and
his genuinely original discoveries. What some writers describe as his “thousands upon thousands of
sheets of mathematics and of scientific observations” appeared to be lost until 1784, when they were
found in Henry Percy’s country estate by one of Percy’s descendants. She gave them to Franz Xaver
Zach,her husband’s son’s tutor. Zach eventually put some of the papers in the hands of the Oxford
University Press, but much work was required to prepare them for publication, and it has never been
done. Scholars have begun to study them,, and an appreciation of Harriot’s contribution started to
grow in the second half of the twentieth century. Harriot’s study of refraction is but one example
where his work overlapped with independent studies carried out by others in Europe, but in any
historical treatment of optics his contribution rightfully deserves to be acknowledged.
QUESTION 1-5
Choose the correct heading for paragraphs B-E and G from the list of headings below. Write the
correct number, i-x, next to the paragraphs.
List of Headings
i A misunderstanding in the history of science
ii Thomas Harriot’s biography
iii Unknown reasons for his unpublished works
iv Harriot’s 1588 publication on North America studies
v Expedition to the New World
vi Reluctant cooperation with Kepler
vii Belated appreciation of Harriot’s contribution
viii Religious pressures keeping him from publishing
ix Correspondence with Kepler
x Interests and researches into multiple fields of study
Example Answer
Para A i
1. Paragraph B 2. Paragraph C 3. Paragraph D
4. Paragraph E 5. Paragraph G
QUESTION 6-10
Answer the questions below using NO MORE THAN THREE WORDS from the passage for each answer.
Various modem applications base on an image produced by lens uses refraction, such as
6_____________. And a spectrum of colors from a beam of light can be produced with
7____________. Harriot travelled to Virginia and mainly did research which focused on two subjects
of American 8_____________. After, he also enter upon a study of flight dynamics and
9_______________ for one of his friends much ahead of major European competitor. He undertook
extensive other studies which were only noted down personally yet predated than many other great
scientists. One result, for example, corrected the misconception about the idea of 10____________.
WRITING
PART 1: Rewriting
1. It was the goalkeeper that saved the match for us.
→Had
2. Tim insisted on being told the complete story.
→Nothing
3. Jane’s husband will be returning from South America quite soon.
→It won’t .
4. The permit expires at the end of this month.
→The permit is not
5. I don’t really like her, even though I admire her achievements.
→Much as
6. As an antidote to their disappointment, he bought them ice-cream. (OFFSET)

7. If interest rates are cut, the economic situation may improve. (REDUCTION)
→ .
8. I don’t personally care if they come or not. (MATTER)

9. Local residents said they were against the new traffic scheme. (DISAPPROVAL)

10. Products which seem to lack credibility are not popular. (CALL)
TEST 4
PART I. LISTENING (32 points)
PART I. LISTENING (40 points)
I. You will listen to part of an interview with the press officer of the National Fitness Association, in
which he gives advice on how to join the right gym. For questions 1 - 7, choose the answer (A, B or
C) which fits best according to what you hear. You will hear the recording twice. Write your answer
in the box provided. (14points)
1. Why is it advisable to research gyms in your area before you join?
A. Some gyms have too much of a social scene.
B. Different gyms suit people with different needs.
C. It's impossible to lose weight in the wrong gym.
2. Mark warns that joining a gym which is unsuitable for you ____________
A. will make you want to give up.
B. might prove embarrassing.
C. could end up being expensive.
3. According to Mark, when should you visit a gym for the first time?
A. When it's at its busiest
B. On any weekday
C. At a relatively quiet time
4. Apart from the standard monthly cost of being a member, you should find out ________
A. whether personal trainers are compulsory.
B. whether there are any hidden costs.
C. whether the changing rooms are expensive.
5. If the gym asks you to sign a contract, you should ______________
A. be suspicious of that gym.
B. study it closely on your own.
C. see what happens if you leave.
6. Why does Mark say that life memberships are not good?
A. You might decide to live elsewhere.
B. The gym might move far away from you.
C. You might change your mind about exercise.
7. Mark says that doing research before joining is worth it because ______________
A. it can make a difference to the overall cost.
B. it will encourage you to start exercising.
C. the right gym can improve your quality of life.
Answers:

1. 2. 3. 4. 5. 6. 7.

II. You will hear part of a radio talk about how to choose houseplants. For questions 8-1,
complete the sentences that summarize what the speaker says with NO MORE THAN THREE
words. You will hear the recording twice. Write your answer in the box provided. (16 points)
Before you get the plant, choose the 8____________________.
You must make sure there is enough 9____________________.
Check that plants have been well looked after at the garden centre.
Plants in poor condition mean the garden centre might not be 10____________________.
Choose plants with healthy green 11____________________.
It could take months or years for a plant to reach the size you want.
It may be better to buy a large plant even if it is 12____________________.
Do not buy plants that have just been put 13____________________.
It is not a good idea to buy plants in 14_____________________.
You should look under the leaves for unwanted 15_____________________.
Answers:

8. 9. 10. 11.

12. 13. 14. 15.

III. Listen to the conversation and decide if each statement is True (T) or False (F). (10 points)
A. Marilena talks about her sister in England and her nephews and nieces.
1. _____ One of Marilena’s sisters lives with her.
2. _____ The two sisters get on really well.
3. _____ Marilena’s sister came to England because Marilena asked her to.
4. _____ Marilena’s sister trained as an electrical engineer.
5. _____ All Marilena’s grandparents are dead.
B. Marilena talks about her sister’s life in Romania.
6. _____ Marilena’s sister only has 100 of her own money to live on for a month after
she’s paid her mortgage.
7. _____ Marilena says food is much less expensive in Romania than in London.
8. _____ Marilena’s sister and brother-in-law’s joint income per month is 400.
9. _____ When Marilena goes back to Romania on holiday she gives her sister 1,000.
10. _____ Marilena is planning to return to Romania once she has saved enough money.
Answers:

1. 2. 3. 4. 5.

6. 7. 8. 9. 10.

PART II: GRAMMAR AND VOCABULARY (60 points)


I. Select the best option for each sentence. Write your answers in the box provided (20 pts)
1. Lack of sleep over the last few months is finally ________ Jane.
A. catching up with B. getting on with C. coming over D. putting on
2. Sally has an ________ command of the Chinese language.
A. extreme B. outstanding C. utter D. intensive
3. I’ve had this car for 12 years, but now I’m having more and more problems with it. Clearly it’s
________.
A. on its hind legs B. got its back up C. got its heart set D. on its last leg
4. All things ________, she is the best student to represent our school.
A. considered B. involved C. taken D. dealt with
5. To succeed in this job, you have to be utterly ________.
A. hot-blooded B. single-minded C. kind-hearted D. near-sighted
6. The Prime Minister gave a press conference to deny the charges ________ at him.
A. leveled B. accused C. targeted D. blamed
7. When the morning came, the scene of where the bomb had fallen was one of ________
devastation.
A. great B. utter C. entire D. extreme
8. There is still a ________ of hope that the rescuers will find survivors.
A. spray B. ray C. light D. spot
9. ________, modelling is actually hard work.
A. Even it may seem glamorous B. Yet it may seem glamorous
C. However glamorous it may seem D. Glamorous as though it is
10. The city zoo is building a new section to ________ their larger mammals.
A. house B. store C. shade D. capture
11. After so many years, it is great to see him ________ his ambitions.
A. get B. realise C. possess D. deserve
12. The review committee ________ three practicing lawyers and a retired businessman.
A. consists B. comprises C. is made up D. encloses
13. Don’t worry: this is nothing that ________ you.
A. matters B. entails C. concerns D. complicates
14. As always, I am ________ with everything you say.
A. agree B. agreeing C. agreeable D. in agreement
15. It may be raining, but I’m ________ enjoying myself.
A. thoroughly B. highly C. extremely D. desperately
16. In the event, we found your advice absolutely ________.
A. unworthy B. valuable C. invaluable D. impecunious
17. In the end, I just lost my ________ and started gabbling incoherently.
A. head B. mind C. brain D. intelligence
18. Despite all the interruptions, he ________ with his work.
A. stuck at B. held on C. hung out D. pressed on
19. When the funds finally ________, they had to abandon the scheme.
A. faded away B. clamped down C. petered out D. fobbed off
20. The team won the championship four years ________.
A. running B. passing C. following D. rotating

Answers:

1. 2. 3. 4. 5.

6. 7. 8. 9. 10.

11. 12. 13. 14. 15.

16. 17. 18. 19. 20.

II. There are TEN mistakes in the passage below. Read the passage carefully, underline the mistakes
and write your corrections on the corresponding lines in the box for Answers. (20 pts)
Answers
Each week Hilary Mullock, also known as Doctor Doppit, visiting London’s ____________
General Hospital. Carrying balloons and magic tricks better than a
____________
stethoscope, she administers her own special kind of medicine. Employed
by the Theodora Children’s Trust, Hilary brings fun and laughter for the ____________
patients in the children’s wards, making a hospital staying a less difficult
____________
experience for these young patients.
____________
Having studied drama at university, Hilary later became interested in
children’s theatre. Seeing an advert with a clown doctor, she knew she had ____________
founded the ideal job. Before taking up the position, Hilary had to complete
four weeks of training, being instructed in balloon modeling and magic ____________
tricks.
____________
According to a spokesperson for the Theodora Children’s Trust,
____________
hospitalized children, having been excluding from their normal day-to-day
routine and the family environment, are likely to be frightening and ____________
homesick. Clown doctors like Hilary has a valuable part to play in helping
____________
them forget their problems for a while. Humour, it has been seen, has a
positively impact on health. In fact, certain chemicals produced in the body ____________
by laughter have even been shown to act as natural painkillers. Laughter ____________
really is the best medicine, it seems.
____________
____________
____________
____________
____________
____________

Answers:

1. 2. 3. 4. 5.

6. 7. 8. 9. 10.

III. Use the word given in capitals at the end of some of the lines to form a word that fits in the
space in the same line. Write your answers in the box provided (10 pts)
Until comparatively recent times science and technology performed RELATE
different and separate functions, the progress of one so often completely
HISTORY
(1) ________ to the progress of the other.
PHILOSOPHY
(2) ________ have established that, since the earliest times, the
improvements in our way of life have resulted from an empirical TAKE
approach, that is a process of trial and error, by which equipment and
REVOLUTION
tools are made to satisfy important needs. It is to this approach that we
owe the evolution of technology. Our modern concept of science, both PLACE
(3) ________ and pragmatic in approach, stems from the seventeenth INNOVATE
century, when extensive investigations into the natural laws governing
the behavior of matter were (4) ________. It was this (5) ________ style FRUIT
of thought which led to a science-based technology. Scientific knowledge
was not in itself seen as a (6) ________ for the earlier system of trial and STRONG
error, but it did help the technical (7) ________ to see which path of
RELY
experimentation might be more (8) ________. With the industrialization
of the nineteenth century, the bond between science and technology (9)
________. In our own time, the mutual (10) ________ of one discipline
upon the other has increased still further.
Answers:

1. 2. 3. 4. 5.

6. 7. 8. 9. 10.

PART III: READING (42 points)


I. Choose the words that best complete the sentences in the text. Write your answers in the box
provided (15 pts)
Despite the continued 1.________ of those early town perks, it wasn't until the Depression
that modern Hershey started to take shape. Perhaps the only town in the country actually
to 2_________ during the 1930s, it thrived because Hershey vowed his Utopia would
never see a breadline. lnstead he 3_________ a massive building boom that gave rise to
the most visited buildings in today's Hershey and delivered wages to more than 600
workers. He admitted that his 4____________ were partly selfish: "lf I don't provide work
for them, I'll have to feed them. And since building materials are now at their lowest cost
levels, I'm going to build and give them jobs."
He seems to have 5__________ no expense; most of the new buildings were strikingly
6________. The first to be finished was the three-million-dollar limestone Community
Center, home to the 1,904-seat Venetian-style Hershey Community Theater, which has
played 7______ since 1933 to touring Broadway shows and to music, dance, and opera
performances. lt offers just as much to look at when the lights are on and the curtains
closed. The floors in the 8_________ named Grand Lobby are polished ltalian lava rock,
surrounded by marble walls and capped with a bas-relief ceiling showing sheaves of wheat,
beehives, swans, and scenes from Roman mythology. With the 9________ inner foyer,
Hershey thumbed his nose even harder at the ravages of the Depression: The arched ceiling
is tiled in gold, the fire curtain bears a painting of Venice, and the ceiling is 10_______with
88 tiny lightbulbs to re-create a star-lit night.
1. A. flexibility B. rigidity C. elasticity D. resilience
2. A. prosper B. decline C. get on D. flower
3. A. trusted B. funded C. accounted D. stocked
4. A. pretensions B. objections C. preoccupation D. intentions
5. A. spared B. spent C. allowed D. justified
6. A. impoverished B. unattractive C. poor D. opulent
7. A. hosting B. housing C. host D. homogeneously
8. A. aptly B. inappropriately C. seemingly D. frightfully
9. A. dizzying B. gaudy C. dazzling D. bland
10. A. holed B. studded C. supported D. magnified
II. Read the text below and think of the word which best fits each space. Use only ONE WORD in
each space. Write your answers in the box provided (10 pts)
Throughout our lives, right from the moment when (1) ________ infants we cry to express hunger, we
are engaging in social interaction of one form or another. Each and (2) ________ time we encounter
fellow human beings, some kind of social interaction will take place, (3) ________ it’s getting on a bus
and paying the fare for the journey, or socializing with friends. It goes without (4) ________,
therefore, that we need the ability to communicate. Without some method of transmitting intentions,
we would be at a complete loss when it comes to interacting socially.
Communication involves the exchange of information, which can be (5) ________ from a gesture to a
friend signaling boredom to the presentation of a university thesis which may (6) ________ ever be
read by a handful of others, or it could be something in (7) ________ the two.
Our highly developed languages set us apart from animals. But for these languages, we could not
communicate sophisticated or abstract ideas. (8) ________ could we talk or write about people or
objects (9) ________ immediately present. (10) ________ we restricted to discussing objects already
present, we would be unable to make abstract generalisations about the world.
Answers:

1. 2. 3. 4. 5.

6. 7. 8. 9. 10.
III. Read the following passage and choose the correct answer to each of the following questions.
Write your answers in the box provided (10 pts)
Exquisite patterns and surface ornamentation were an integral part of the aesthetics of the late
Victorian era. In America, these developments were incorporated into the themes of national
expositions and artistic movements, as cottage industries grew and productivity in the decorative arts
flourished. The last three decades of the 19th century saw a change in sensibility that resulted in new
stylistic approaches in American decorative arts, a departure from the previous era of Rococo and
Renaissance Revival excess. Shapes became more angular, smoother and less flamboyant. The popular
carvings and deep modeling of earlier years disappeared as ornamentation became more linear and
lighter in appearance. Decoration focused on the surface with rich and elegant patterns adorning
furniture, objects of every sort, and architectural and interior decorations. This artistic reawakening
was prompted by the effects of the Industrial Revolution on contemporary design.
This new attitude, with its focus on ornament and decorative, was later referred to as the Aesthetic
Movement, but it also encompassed the early Arts and Crafts Movement as well. The purpose was to
bring a refined sensibility and components of “good taste” to the domestic interior. Art and good taste
not only denoted good character, but also could be used to induce proper moral conduct and actions,
thereby contributing to the betterment of society. This placed a heavy burden on
designers/decorators as well as on women as keepers of the home. Americans drew inspiration from
the writing and work of English artists. This was a period of great eclecticism. Tastes ranged from the
Modern Gothic through the Persian, Greek and Islamic, to the Japanese, and with more than a nod to
Mother Nature. Yet, regardless of the influence, surface pattern reigned supreme. English reformers
dictated that ornament should be derived from nature, and pattern should be flat and stylized. Forms
were accentuated by colored outlines, or often with touches of gold. The emphasis was on art and on
development of a refined sensibility. It was all a matter of taste.
1. What is the main topic of the passage?
A. Defining the “Aesthetic Movement”
B. Decorative arts in late 19th century America
C. English influences on American decorative arts in the late 19th century
D.The change in tastes from “Rocco and Renaissance Revival” to the “Aesthetic Movement” in the late
19th century America
2. The word “integral” in line 1 is closest in meaning to
A. essential B. additional C. important D. beautifying
3. According to the passage, during the Aesthetic Movement popular carvings and deep modeling of
earlier years ________.
A. were popular B. again became popular
C. disappeared D. defined good taste
4. The word “elegant” is closes in meaning to ________.
A. beautiful B. ornamental C. colorful D. refined
5. According to the passage, the purpose of the Aesthetic Movement was to ________.
A. induce proper moral conducts and actions
B. define what was meant by good taste in the domestic interior
C. encompass Arts and Crafts as well as ornament and decoration
D. define good character and contribute to the betterment of society
6. The phrase “new attitude” refers to ________.
A. including the early Arts and Craft Movement as well
B. artistic reawakening
C. the Industrial Revolution
D. Rococo and Renaissance Revival
7. The word “denoted” is closest in meaning to ________.
A. promoted B. facilitated C. developed D. signified
8. Which of the following can be inferred from the passage?
A. designers and decorators were mainly responsible for starting the new attitude
B. the movement led to a higher standard of morality in late 19th century America
C. the Americans considered the English to be the arbiters of good taste
D. women, as keepers of the home, faced a heavy burden
9. According to the passage, which of the following remained most important, regardless of influences
from other countries?
A. surface pattern B. English opinions
C. good taste D. Proper moral conduct and actions
10. Which of the following is NOT mentioned as feature of the Aesthetic Movement?
A. shapes became less flamboyant B. ornamentation became lighter in appearance
C. forms were accentuated by colored lines D. decorations focused mainly on furniture
Answers:

1. 2. 3. 4. 5.

6. 7. 8. 9. 10.
IV. There are seven paragraphs A – G. Choose the correct heading for each paragraph from the
list of headings below. Write your answers in the box provided. (7 pts)

List of Headings 1. Paragraph A


I. Optimistic beliefs held by the writers of childrens’s literature 2. Paragraph B
II. The attitude of certain adults towards children’s literature 3. Paragraph C
III. The attraction of children’s literature 4. Paragraph D
IV. A contrast that categorises books as children’s literature 5. Paragraph E
V. A false assumption made about children’s literature 6. Paragraph F
VI. The conventional view of children’s literature 7. Paragraph G
VII. Some good and bad features of children’s literature
VIII. Classifying a book as children’s literature
IX. The treatment of various themes in children’s literature
X. Another way of looking at children’s literature

A I am sometimes asked why anyone who is not a teacher or a librarian or the parent of little kids
should concern herself with children’s books and folklore. I know the standard answers: that many
famous writers have written for children, and that the great children’s books are also great literature;
that these books and tales are an important source of archetype and symbol, and that they can help
us to understand the structure and functions of the novel.
B All this is true. But I think we should also take children’s literature seriously because it is sometimes
subversive: because its values are not always those of the conventional adult world. Of course, in a
sense much great literature is subversive, since its very existence implies that what matters is art,
imagination and truth. In what we call the real world, what usually counts is money, power and public
success.
C The great subversive works of children’s literature suggest that there are other views of human life
besides those of the shopping mall and the corporation. They mock current assumptions and express
the imaginative, unconventional, noncommercial view of the world in its simplest and purest form.
They appeal to the imaginative, questioning, rebellious child within all of us, renew our instinctive
energy, and act as a force for change. That is why such literature is worthy of our attention and will
endure long after more conventional tales have been forgotten.
D An interesting question is what – besides intention – makes a particular story a ‘children’s book’?
With the exception of picture books for toddlers, these works are not necessarily shorter or simpler
than so-called adult fiction, and they are surely not less well written. The heroes and heroines of these
tales, it is true, are often children: but then so are the protagonists of Henry Jame’s What Maisie Knew
and Toni Morrison’s The Bluest Eye. Yet the barrier between children’s books and adult fiction
remains; editors, critics and readers seem to have little trouble in assigning a given work to one
category or the other.
E In classic children’s fiction a pastoral convention is maintained. It is assumed that the world of
childhood is simpler and more natural than that of adults, and that children, though they may have
faults, are essentially good or at least capable of becoming so. The transformation of selfish, whiny,
disagreeable Mary and hysterical, demanding Colin in Frances Hodgson Burnett’s The Secret Garden is
a paradigm. Of course, there are often unpleasant minor juvenile characters who give the protagonist
a lot of trouble and are defeated or evaded rather than reeducated. But on occasion even the angry
bully and the lying sneak can be reformed and forgiven. Richard Hughes’s A High Wind in Jamaica,
though most of its characters are children, never appears on lists of recommended juvenile fiction; not
so much because of the elaborations of its diction (which is no more complex than that of, say,
Treasure Island), but because in it children are irretrievably damaged and corrupted.
F Adults in most children’s books, on the other hand, are usually stuck with their characters and
incapable of alteration or growth. If they are really unpleasant, the only thing that can rescue them is
the natural goodness of a child. Here again, Mrs. Burnett provides the classic example, in Little Lord
Fauntleroy. (Scrooge’s somewhat similar change of heart in Dicken’s A Christmas Carol, however, is
due mainly to regret for his past and terror of the future. This is one of the things that makes the book
a family rather than a juvenile romance; another is the helpless passivity of the principal child
character, Tiny Tim.).
G Of the three principal preoccupations of adult fiction – sex, money and death – the first is absent
from classic children’s literature and the other two either absent or much muted. Money is a motive in
children’s literature, in the sense that many stories deal with a search for treasure of some sort. These
quests, unlike real-life ones, are almost always successful, though occasionally what is found in the
end is some form of family happiness, which is declared by the author and the characters to be a ‘real
treasure’. Simple economic survival, however, is almost never the problem; what is sought, rather, is a
magical (sometimes literally magical) surplus of wealth. Death, which was a common theme in
nineteenth-century fiction for children, was almost banished during the first half of the twentieth
century. Since then it has begun to reappear; the breakthrough book was E. B. White’s Charlotte’s
Web. Today not only animals but people die, notably in the sort of books that get awards and are
recommended by librarians and psychologists for children who have lost a relative. But even today the
characters who die tend to be of another generation; the protagonist and his or her friends survive.
Though there are some interesting exceptions, even the most subversive of contemporary children’s
books usually follow these conventions. They portray an ideal world of perfectible beings, free of the
necessity for survival.
Answers:
Paragraph 1 2 3 4 5 6 7

Heading

PART IV: WRITING (50 points)


I. Finish each of the following sentences in such a way that it means the same as the sentence
printed before it. (20 points)
1. That week, the train was late everyday except for Friday. RUN
Only once ___________________________________________________________.
2. I really think you should be more assertive about your rights. UP
It’s about time ________________________________________________________.
3. Graham took back his words on noticing there were fresh strawberries on the menu. TUNE
Graham sang _________________________________________________________.
4. Paul couldn’t stop thinking about the argument he had had with his brother. WENT
Paul ________________________________________________________________.
5. My grandmother would always make us eat everything on our plates when we visited her.
WASTE
Nothing we __________________________________________________________.
6. Alternative medicine is a complete mystery to some people.
Some people are ______________________________________________________.
7. You may be disqualified if you don’t obey the regulations. Failure
______________________________________________________________.
8. He tried hard so that he could win the first prize.
Having ______________________________________________________________.
9. I only recognized him when he came into the light.
It was not until _______________________________________________________.
10. Mass tourism has been one of the causes of the environmental problems.
Mass tourism is _______________________________________________________.

TEST 5
LISTENING (50 POINTS)
Task 1: Listen to his talk again and choose the best answer A, B or C for each question below.
1. What word describes the man's feelings at the beginning of the conversation about traveling
overseas?
A. relaxed B. apprehensive C. anxious
2. What one thing surprised him in the marketplace?
A. The quality of merchandise seemed to vary widely from place to place.
B. Shop owners would raise the price of goods if they saw you were a foreigner.
C. You could bargain over the price with shopkeepers to get the best deal.
3. What did the man NOT mention about the way people looked at him during his visit?
A. People stared at him out of a distrust of foreigners.
B. People were just inquisitive at his presence there.
C. Such situations gave him opportunities to interact with others.
4. What does the man say about the traffic conditions?
A. The people seem to pass through traffic as if unaffected by everything around them.
B. Buses and taxis are the main modes of transportation in many parts of the country.
C. The flow of traffic, at times, remains constant due to an intricate road system.
5. What is one conclusion the man states at the end of the travel log?
A. Discovery of different cultures through direct experience is key to cultural adaptation.
B. Miscommunication is a natural event in the process of bridging cultural boundaries.
C. All countries share some similar characteristics that bind them together.

Task 2: Listen to a piece of news and decide which of the statements are T(rue), which ones are
F(alse)
1. The group of refugees travel from Ayvalik to Lesbos by bus.
2. 4 people died in a capsize near Kalymos.
3. A tip-off showed Turkish border guards a convoy of Syrian refugees.
4. Two of the three drivers were arrested.
5. Turkish gendarme find another 92 refugees in another accident.
Task 3: Listen to a piece of news about handicrafts in Bukhara city and fill in the blank WITH NO
MORE THAN TWO WORDS AND/OR A NUMBER.
Old city of Bukhara, Uzbekistan – city of handicrafts:
1. Handmade carpets:
- made by women.
- placed (1) _______________ or (2) _______________ of every Uzbek house.
- can be easily recognized by their (3) ______________ and (5) ______________
- mostly patterned with (5) _______________, (6) ________________ and flowers.
2. Pottery:
- produced with old techniques and (7)_______________.
- highly popular.
- use such colors as (8) ____________, ___________ and _____________
- follow at least 24 steps to create a complete product.
3. Gold embroidery:
- in the past: only for emirs and (9)_____________
- nowadays: (10) ____________ is widely used for many kinds of clothes.
LEXICAL – GRAMMAR (50 POINTS)
Part I. Choose the correct answer for each gap in the following sentences (20 points).
1. In the ________ of security, personnel must wear their identity badges at all time.
A. requirement B. interests C. demands D. assistance
2. The strike was________ owing to a last minute agreement with the management.
A. called off B. broken up C. set back D. put down
3. Lindsay’s excuses for being late are beginning to________ rather thin.
A. get B. turn C. wear D. go
4. ________, the people who come to this club are in their twenties and thirties.
A. By and large B. Altogether C. To a degree D. Virtually
5. My cousin was nervous about being interviewed on television but she rose to
the________ wonderfully.
A. event B. performance C. incident D. occasion
6. The train service has been a ________ since they introduced the new schedules.
A. shambles B. rumpus C. chaos D. fracas
7. Is an inexperienced civil servant ________ to the task of running the company?
A. capable B. skilled C. eligible D. suited
8. John’s got very ________ feelings about taking on more responsibility at the moment.
A. puzzled B. jumbled C. mixed D. muddled
9. You’ve lived in the city for most of your life, so ________ you’re used to the noise.
A. apparently B. presumably C. allegedly D. predictably
10. The storm ripped our tent to________ last night.
A. slices B. shreds C. strips D. specks
11. He cannot ________ ignorance as his excuse: he should have known what was happening in his
own department.
A. insist B. plead C. refer D. defend
12. If you don’t pay your bill, the Electricity Board will________ your power supply.
A. dismantle B. sever C. disconnect D. uncouple
13. There is a lot of friendly ________ between the supports of the two teams.
A. contest B. rivalry C. contention D. defiance
14. Despite the high divorce rate, the ________ of marriage remains popular.
A. practice B. habit C. institution D. state
15. The zoo attendant opened the cage and tried to________ the animal back in.
A. coax B. induce C. seduce D. convince
16. I lost too much money betting at the races last time, so you won’t ________ me to go again.
A. convince B. impress C. persuade D. urge
17.He has been ______________for gross misconduct.
A. impounded B. impeached C. impelled D. impaled
18.Not getting the promotion felt like a real kick in the ____________as I’d put in so much hard work
for the company.
A. head B. teeth C. back D. leg
19.China, which has been producing cars joint ventures with foreign partners, is now launching
its own brand ____________the global market.
A. through/on B. for/over C. of/throughout D.alongside/ in
20.Everyone in the company detests getting close to him because he’s always trying to
___________the boss.
A.suck up to B. cry out for C. pin down to D. fall over to
Your answers:
1....................2....................... 3..................... 4............... 5................
6....................7....................... 8..................... 9............... 10...............
11............. 12................... 13..................... 14............... 15................
16.............. 17.................... 18..................... 19............... 20...............

Part II. Complete the sentences with the correct form of words (10 points).
Mobile phones emit (1) (MICRO) ________ radio emissions. Researchers are questioning whether (2)
(EXPOSE) ________ to these radio waves might lead to brain cancer.
So far, the data are (3) (CONCLUDE) ________ The scientific evidence does not enable us to say
with (4) (CERTAIN) ________ that mobile phones are (5) (CATEGORY) ________ safe. On the other
hand, current research has not yet demonstrated clear adverse effects (6) (ASSOCIATE) ________with
the (7) (LONG) ________ use of mobile phones.
Numerous studies are now going on in (8) (VARY) ________ countries. Some of the results are (9)
(CONTRADICT) ________ but others have shown an association between mobile phone use and
cancer. However, these studies are preliminary and the issue needs further, (10)
(TERM) ________ investigation.
Until the scientific data are more definite, it is prudent for people to try not to use mobile phones
for long periods of time. Don’t think that hands-free phones are any safer either. At the moment,
research is in fact showing the opposite and they may just be dangerous.
Your answers:
1....................2....................... 3..................... 4............... 5................
6....................7....................... 8..................... 9............... 10...............
Part III. There are ten errors in the following passage. Underline them in the text and correct them
in the numbered box (10 points)
Why do some people back on when faced with a threat, while others stand up to it? When
giving a difficult task, why do some people see it through, white others give up? It all comes
down to personalities. But where does that come from? Some scientists believe that most of traits
are inherited. The others take the opposite view: personality, they say, is formed by our
environment and parents do not pass it on in their children. The truth is probably anywhere in
between. Some traits are clearly determined by your environment: However your genetic
background, if you grow up in Sweden, you'll probably speak Swedish. On the other hand, when
it comes to traits like the colour of your eyes or your blood type, it is clear that genetics alone
accounts for them. There are also traits which are partly inherited but partly shaped by
environment: your weight, and even your height and skin colour, are examples. Of course,
genetics and the environment together are not the total picture. Your free will - your ability to take
decisions - is also a factor in shaping your identity, but how big a factor?
You'll have to make your own mind up for that!
Your answers:
1............................ 2................................ 3……......................
4............................ 5................................ 6……......................
7............................ 8................................ 9…………...............
10..........................
READING (50 POINTS)
Part I. Choose the best answer for each gap in the following passage (10 points).
In the early morning of 23 January, 2009, the most powerful storm for a decade hit western France.
With wind speeds in (1) ________ of 120 miles per hour, it flattened forests, (2) ________ down
power lines and caused massive destruction to buildings and roads. But it also left behind an
extraordinary creation. Seven miles out to sea at the (3) ________ where the Atlantic Ocean meets
the estuary of the River Gironde, a small island had (4) _________ out of water. Locals soon gave it the
name The Mysterious Isle. What was so remarkable, (5) ________ its sudden apparition, was the fact
that the island (6) _________ intact in what is often quite a hostile sea environment. It could well
become a permanent feature.
Scientists (7) ________ realised that the island's appearance (8) ________ a unique opportunity to
study the creation and development of a new ecosystem. Within months, it had been colonised by
seabirds, insects and vegetation. Unfortunately, however, they were not alone in (9) ________ the
island attractive. It became increasingly difficult to (10) ________ the site from human visitors. In its
first year, day trippers came in powered dinghies, a parachute club used it as a landing strip, a rave
party was even held there one night.
1. A. surplus B. advance C. excess D. put
2. A. fetched B. brought C. carried D. sent
3. A. scene B. mark C. stage D. point
4. A. risen B. growth C. lifted D. surfaced
5. A. in spite of B. instead of C. apart from D. on account of
6. A. prolonged B. remained C. resided D. preserved
7 A. quickly B. briskly C. hastily D. speedily
8. A. delivered B. awarded C. proposed D. offered
9. A. regarding B. finding C. seeking D. deciding
10. A. prevent B. preserve C. protect D. prohibit

Your answers:
1....................2....................... 3..................... 4............... 5................
6....................7....................... 8..................... 9............... 10...............
Part II. Read the following passage and fill in each gap with ONE suitable word ( 15 points).
One of the most dangerous drugs for pregnant women to consume is alcohol. Because alcohol is
delivered quickly into the blood and passes quickly into the tissues and membranes, the human fetus
is particularly vulnerable to (1) ________ effects. The reality is that the (2) ________ effects on a fetus
are so pronounced that babies born after exposure to alcohol are said to be (3) ________ from fetal
alcohol syndrome. As a pregnant woman drinks alcohol, the alcohol is passed into her bloodstream
almost simultaneously. Moreover, because the bloodstream of the fetus is inextricably tied to
(4) ________ of the mother, the alcohol passes directly into the bloodstream of the fetus as well.
And, what is (5) ________, the concentration of alcohol in the fetus is exactly the same as in the
mother.
For the mother, this concentration is not a problem (6) ________ her liver can remove one ounce
of alcohol from her system per hour. However, the fetus's liver is not completely developed
((7) ________ developed it is depends on its stage of development). The rate at which it is able to
eliminate the alcohol from the blood of the fetus is much slower. Eventually, the alcohol will be
returned to the mother's system by passing across the placenta, but this process is slow. By the time
this takes place, major neurological damage may have (8) ________ occurred.
Research has shown that as (9) ________ as one drink of alcohol can produce significant,
irreversible damage to the fetus. Babies born after exposure to alcohol generally exhibit facial
distortion, (10) ________ to concentrate, and difficulty in remembering. Simply speaking, it is
imperative that pregnant women avoid alcohol.
Your answers:
1...................2....................... 3..................... 4............... 5................
6....................7....................... 8..................... 9............... 10...............
Part III. Read the following passage and mark the letter A, B, C, or D on your answer sheet to
indicate the correct answer to each of the following questions (10 points)
Deaf people should enjoy a play as much as the hearing audience. This is the thinking behind the
Americans with Disabilities Act, which requires that theatres become accessible to deaf people.
Theatre producers, on the other hand, are beginning to realise that such a feature makes their
product attractive to a wider audience – with considerable financial rewards.
Interpreted theatre usually takes place at two or three performances in a theatre season. Preference
for tickets is given to the deaf and their guests, but tickets are also sold to hearing people who are
interested in seeing theatre interpreters at work.
It is a colossal project for the interpreters. Ian Cox works for SeeTheatre, a Chicago-based association
of interpreters. He says, ‘for about a month we work with the support of a Sign Language consultant
to translate and rehearse an entire play. The workload is immense. Consider that each actor in the
play has about four weeks to rehearse his or her lines, we have to learn the whole play in as much
time. And, when interpreting musicals, where timing, rhythm and harmonies must be practised, the
workload doubles.’
The most important technical aspect of theatre interpretation for the deaf is the location of the
interpreter. Placement strategies can be categorised into three styles: ‘placed’, where interpreters are
located outside the acting space and do not move; ‘zoned’, where interpreters are within the acting
space, but usually move only during a change of scene or act; and, finally, ‘shadowed’, where
interpreters move freely within the acting space, shadowing the movement of the actors for whom
they are interpreting.
The ‘placed’ style of interpreting in the theatre is by far the most common. The interpreters are side-
by-side and face the audience (although some interpreters interact with each other as appropriate).
The location of the interpreter is generally in one of three places: stage right or stage left, or on the
floor of the house.
The ‘zoned’ style of interpreter placement is a happy medium between the ‘placed’ and ‘shadowed’
styles. Here, interpreters are placed side-by-side within the acting space. Usually, they change position
on stage from scene to scene - or from act to act - in order to be within the same ‘zone’ as the
majority of the action. Zone placement makes it easier for the deaf patron to see the interpreters and
actors at the same time.
The ‘shadowed’ style of interpreting is the most inclusive style of interpreting for the theatre. It
involves placing the interpreters directly within the action - nearly making them ‘sign language actors’.
The interpreters are ‘blocked’ into each scene, and literally shadow the actors. The advantage of this is
clear: the interpreter is in such close proximity to the actor that the deaf patron need not make a
decision about whom to watch - he or she can watch both at the same time. In the best of cases, the
deaf patron mentally blends the interpreter with the actor, and forgets that the actor does not sign.
Theatre interpreters for the deaf are dedicated professionals who take great pride in their work.
‘Theatre,’ says Ian, ‘is the art of communicating beautiful ideas in interesting ways. Interpreted
theatre, therefore, is an art in itself. Thankfully, instead of seeing us as ‘additions’, producers nowadays
think of interpreters for the deaf as a new avenue to creativity.’
Question 1: Theatre interpreters have become commonplace because _______.
A. a new law made their presence compulsory
B. theatres wanted to become more accessible
C. deaf people were avoiding the theatre
D. producers wanted a way to attract audiences
Question 2: Why do hearing people go to see interpreted performances?
A. They give a better understanding of the play.
B. They are even better than standard performances.
C. They offer a chance to admire the work of interpreters.
D. They are the most popular performances of the season.
Question 3: The workload of theatre interpreters is so great because they have to _______.
A. go to many of the rehearsals B. interpret in front of a consultant
C. learn the lines of all the actors D. work in all parts of the production
Question 4: The word ‘patron’ in paragraph 6 is closest in meaning to _______.
A. producer B. sponsor C. playgoer D. playwright
Question 5: In which style(s) of interpreting do the interpreters change their position?
A. the ‘zoned’ and the ‘shadowed’ styles B. the ‘zoned’ style
C. the ‘placed’ and the ’shadowed’ styles D. the ‘shadowed’ style
Question 6: Why does the writer refer to the ‘zoned’ style as 'a happy medium’?
A. It combines elements from the two other styles.
B. It is mostly used in light-hearted plays.
C. It allows the interpreters to mix with the actors.
D. It makes the interpreter's job easier.
Question 7: The word 'this' in paragraph 7 refers to _______.
A. the advantages of the ‘shadowed’ style B. the position of the interpreters
C. the action in a performance D. the placement of the actors
Question 8: What does Ian say about theatre producers in the last paragraph?
A. They don't respect interpreters.
B. They think interpreters are expensive.
C. They see interpreters as a necessary evil.
D. They appreciate the work of interpreters.
Question 9: What is the writer's purpose in the passage?
A. To show the variety in modern theatre. B. To advertise interpreted theatre.
C. To inspire interest in the topic. D. To criticise the attitude of theatre directors.
Question 10: The word ‘shadowing’ in paragraph 4 is closest in meaning to _______.
A. following B. changing C. directing D. controlling
Your answers:
1...................2....................... 3..................... 4............... 5................
6....................7....................... 8..................... 9............... 10...............
Part 3: For questions 1- 13, read the following passage and answer the following questions (1.3
points)
SKYSCRAPER
A. The word skyscraper was originally a nautical term for a tall mast or sail on a sailing ship. Today the
word is used exclusively to refer to a tall habitable building, usually higher than 150 meters (500 feet).
Most skyscrapers serve as office buildings or hotels. The term “high-rise” is also used to describe tall
buildings, but it tends to be applied specifically to residential buildings. Modern building of great
height, constructed on a steel skeleton originated in the United States.
B. Until the 19th century, buildings of over six stories were rare. It was impractical to have people walk
up so many flights of stairs, and water pressure could only provide running water to about 50 feet
(15m). Many mechanical and structural developments in the last quarter of the 19 th century,
contributed to the evolution of building. With the perfection of the high speed elevator after 1887,
skyscrapers were able to attain any desired height. The earliest tall buildings were of solid masonry
construction, with the thick walls of the lower stories usurping a disproportionate amount of floor
space. In order to permit thinner walls through the entire height of the building, architects began to
use cast iron in conjunction with masonry. This was followed by cage construction, in which the iron
frame supported the floors and the masonry walls bore their own weight.
C. The next step was the invention of a system in which the metal framework would support not only
the floors but also the walls. This innovation appeared in the Home Insurance Building in Chicago,
designed in 1883 by William Le Baron Jenney – the first building to employ steel skeleton construction
and embody the general characteristics of a modern skyscraper. The subsequent erection in Chicago
of a number of similar buildings made it the centre of the early skyscraper architecture. In the 1890s
the steel frame was formed into a complete riveted skeleton bearing all the structural loads, with the
exterior or thin curtain walls serving merely as an enclosing screen.
D. Heating and air conditioning played an important role in the structure of skyscrapers. They are the
key units that control the inside atmosphere of all skyscrapers. In the early days, the heat sources in
the building came from fireplaces and stoves. Later on heat in the building is supplied by the hot
water boiler. The boiler heats up the water and sends it out to the radiators through a system of pipes.
This was later replaced by central heating furnace with ventilation ducts that channel heat to various
areas of the building. Air condition originated from refrigeration. The process of refrigeration is to
draw heat away from substances to lower their temperature. Today, the skyscrapers use the central
heating system with ventilation ducts that can be shared with the heating and air conditioning
system. Engineers are working on new ways to make heating and air conditioning more efficient and
environmental friendly.

E. In 1892 the New York Building Law made its first provisions for skeleton constructions. There
followed a period of experimentation to devise efficient floor plans and aesthetically satisfying forms.
In 1916, New York City adopted the Building Zone Resolution, establishing legal control over the height
and plan of buildings and over the factors relating to health, fire hazard, and assurance of adequate
light and air to buildings and streets. Regulations regarding the setting back of exterior walls above a
determined height, largely intended to allow light to reach the streets, gave rise to buildings whose
stepped profiles characterize the American skyscraper of subsequent years.
F. With the complex structural and planning problems solved, architects still seek solutions to the
difficulties of integrating skyscrapers with community requirements of hygiene, transportation and
commercial interest. In New York during the 1950s, public plazas were incorporated into the designs
of the Lever House by Gordon Bunshaft and Seagram Building of Mies van der Robe. These
International style buildings are also examples of the effective use of vast expanses of glass in
skyscrapers. More recently, numerous skyscrapers have been constructed in a number of postmodern
modes.
G. The cost of building a skyscraper is in hundred million dollar in the current market. The skyscraper
is well known for its great height and the social status comes with it and has always associated with
wealth and power. To the general public, big is good; in terms of building, the taller the better. In
general, large company or firm would host its head office in these skyscrapers. For example, Chrysler
had its head office in the Chrysler Building, owned by Chrysler which is one of the big three
automakers. The Sears Tower, the head office for Sears and owned by Sears. It was known for being
the world’s tallest skyscraper in Chicago. Tenants expect clients to know where they are located when
they tell them the Sears Tower as their location. These skyscrapers are owned by the wealthy
individuals whom are in the upper level of financial social status. There is a sense of prestige for
having offices or shops in these skyscrapers.
H. Modern skyscrapers are being redefined by the use of advance technologies. Thanks to the
advancement in technology, skyscrapers are able to reach new height easily. It changes the way
architects design the structure of these buildings. The new function of the skyscraper is to provide
great views, house antennas for communications, telebroadcasting and for entertainment purpose.
The use of computer climate control system made the building a more comfortable environment for
everyone. The great height of the skyscraper like the Sears Tower, it associates itself with the prestige
of being the tallest of all. With the rapid advancement of technology and the influence of fame and
wealth, sky will be the only limit for the next generation of skyscrapers.
From the list of the headings, choose the most suitable headings for paragraphs A – D
Hot water boiler and refrigerator
A period of innovation
Skyscraper and a tall mast
Advanced technology
Words for tall buildings
The wall evolution
Hot and cold
From stone to iron
Paragraph A: ________
Paragraph B: ________
Paragraph C: ________
Paragraph D: ________
Your answers: 1...................2....................... 3..................... 4...............
Match the following innovations with A,B,C and D

Mid-20th century B. late 1880s C. 1890s D. early 1880s

The high-speed elevator: ________


Public square : ________
Steel framework: ________
Riveted skeleton: ________
Your answers: 5................6....................7.......................8.....................
Write TRUE, FALSE, NOT GIVEN
A skyscraper is both a tall sailing mast ad a tall habitable building.
A steel frame is able to support both floors and walls.
Your answers: 9............... 10...............
WRITING (50 POINTS)
Part I: Complete the second sentence so that it has a similar meaning to the first sentence, using the
word given (don’t change the word given) or beginning in such a way that their meanings remain
unchanged (20 POINTS).
1. Vanessa was taken on by a big law firm as soon as she graduated ( LANDED)
On graduation,Vanessa ……………………………………………………………….
2. At the end of the concert, Mariah Carey got everyone in the audience to sing along with her.
( SINGING)
At the end of the concert, Mariah Carey …………………………………………………
3. I will never regret telling the truth. (SPADE)
Not ........................................................................................................................................
4. It was Jane who came up with the idea for the sales promotion. (BRAINS)
Jane ................................................................................................. the sales promotion.
5. Doing regular exercise often helps improve your attitude to life. (DO)
Many.............................................................................................................on life.
6. Students must not enter the exam late, whatever the reason might be. (NO)
…………………………………………………………………………………………...late.
7. In this area, Thailand is much better than all other countries in football. (SHOULDERS)
In this area, Thai land ……………………………………………………..
8. I felt stupid when I realized what we had done. (A)
I felt …………………………………………….we had done.
9. He thinks his friends do not appreciate him. (FOR)
He dislikes ………………………………………………………………….
10. She had four daughters. The youngest of them was her favourite. (EYE)
She had four daughters ………………………………………………………..
TEST 6
A. LISTENING (50 pts):
HƯỚNG DẪN PHẦN THI NGHE HIỂU
 Bài nghe gồm 4 phần, mỗi phần được nghe 2 lần, mở đầu và kết thúc mỗi phần nghe có tín
hiệu.
 Mở đầu và kết thúc phần nghe có tín hiệu nhạc.
 Mọi hướng dẫn cho thí sinh ( bằng tiếng Anh) đã có trong bài nghe.

Part 1 (10pts): You will hear a conversation between a staff member from Gaea's Guardians and a
man who wants to do something to protect the environment.
Listen and complete the form below.
Write NO MORE THAN THREE WORDS AND/ OR A NUMBER for each answer.
(Part1 – Test 4 – Ielts Listening Recent Actual Tests)

Example Answer

Aim: protecting environment through recycling

Type of group: non-profit

Frequency of newspaper collection: 1

Name: 2

Address: 3

E-mail: 4

Postcode: 5

Your anwsers:

1.

2.

3.
4.

5.

Part 2 (10pts): You will hear an interview with a yoga teacher. For question 6 – 10, choose the
answer (A,B,C or D), which fits best according to what you hear.
( Advanced CAE practice Tests – Part 3- Test 6)
6. The interviewer
A. tried yoga once but found it impossible to do.
B. is finding yoga hard to do but is improving with practice.
C. has only a vague idea about yoga.
D. has quite a good understanding of yoga.
7. According to Sarah,
A. yoga demands control of all aspects of being.
B. you need to be highly intelligent to practise yoga well.
C. you need to empty your mind completely when practising yoga.
D. meditation is like being hypnotised.
8. Which of the following does Sarah not say is necessary in order to practise yoga?
A. an empty stomach
B. comfortable, flexible clothing
C. a lot of confidence and a fit body
D. a place where you won't be disturbed
9. The interviewer seems concerned about
A. people paying a lot of money for public classes with unqualified teachers.
B. people getting stuck because the teacher is not supervising the class properly.
C. people buying too many yoga guides.
D. beginners practising yoga unsupervised.
10. Sarah sums up by saying that
A. you need to learn more about your own character before attempting to do yoga.
B. yoga can solve any problem you have in life.
C. yoga is better than conventional medicine.
D. in order to fulfill your potential you need to have a positive outlook.

Your answers:

6. 7. 8. 9. 10.

Part 3 (10pts): Listen to the recording and decide whether the following sentences (11-15) are true
(T) or false (F). (IELTS Test Buider – Test 1 Adapted)
11. The purpose of the data collection was to test people’s reaction to different buildings.
12. The initial plan to use a questionnaire was abandoned, because the questions were too difficult to
write.
13. To make sure people could see the detail in the images better only daylight images were used.
14. Among the people who formed part of the sample were tourists from various places.
15. They appointed a leader for the group to help hold the team together.
Your answers:

11. 12. 13. 14. 15.

Part 4 (20pts): Listen to a scientist talking about recent weather patterns. For question 16 – 25,
Complete the sentences. (FCE Practice Test Extra – Part 2- Test 2)

Mark Pontin works at an organisation in ___________________________ (16)


The USA has recently experienced a record number of ___________________(17)
It has been especially hot in _______________________________ (18)
In Sri Lanka there has been unusually ________________________________(19)
There are concerns about what effect this will have on the ________________ (20) there.
There have been very ____________________ (21) in England and Wales.
The last time Switzeland experienced such heat was in the _________________ (22)
Pontin draws some conclusions with regard to climate change in the __________(23)
The trend toward hotter temperatures has been noticed for _______________ (24) years.
In general the weather is becoming more ________________ (25) and extreme than it used to be.
Your answers:

16. 21.

17. 22.

18. 23.

19. 24.

20. 25.

B. LEXICO-GRAMMAR
Part 1 (20pts). Choose the best option A, B, C, or D to complete the following sentences and write
your answers in the corresponding numbered boxes.
1. After having children, the woman starts to __________.
A. go to seed B. come up smelling of roses
C. push up the daisies D. gild the lily
2. Reliable salesmen do not make ________ claims for their product.
A. unbridled B. extravagant C. madding D. unruly
3. Eva likes to have a dictionary _____ hand when she's writing.
A. at B. on C. off D. by
4. With the blistering pace of technological advancements in recent years, both workers
and employers have a myriad of choices, and ________ access to education.
A. exhaustive B. profuse C. bottomless D. unfettered
5. It’s amazing how Jenny acts as though she and Darren ___________ serious problems at
the moment.
A. aren’t having B. hadn’t had C. weren’t having D. hadn’t been having
6. The winning team were ________ criticized by the local media for the way in which they
had gloated over the losing team.
A. shrilly B. decidedly C. roundly D. cheaply
7. He's a(n) __________ piece of nothing at the office; no one even knows he exists.
A. two-bit B. abbreviated C. venial D. tinpot
8. I _________ his telephone number while he was still talking.
A. jotted down B. wrote up C. set down D. dashed off
9. I feel __________ to inform the head teacher that a number of students and parents are
annoyed with the decision.
A. my duty B. this my duty C. it my duty D. that my duty
10. A cute kid is _________ a lot of attention on the internet thanks to his uncanny
resemblance to a famous Kpop idol.
A. amassing B. raking C. garnering D. gathering
11. Government becomes no longer the servant of the people but in the thrall of big money,
lobbyists and a media happy to live off its fancy leftovers in a ___________ of gossip and
shallow speculation.
A. feeding frenzy B. second wind C. fever pitch D. buffer zone
12. Not until Kentucky’s Mammoth Cave had been completely explored in 1972 _________.
A. when was its full extent realized B. that its full extent was realized
C. was its full extent realized D. the realization of its full extent
13. James didn’t take ________ to your suggestion that he was mean with money.
A. kindly B. pleasantly C. agreeably D. cheerfully
14. Even though it’s still easy to _________ digital copy protection, most users can’t do it.
A. outflank B. deflect C. circumvent D. oscillate
15. Sending a reporter against the families' wishes only ensures that we will ______those
families ______.
A. walk/over B. tick/off C. push/around D. turn/on
16. They say that even the police hesitate before entering some of the city’s most
lawless_____.
A. suburbs B. crowds C. ghettoes D. allotments
17. Rising temperatures are said to be _________ the habitats and feeding patterns of
penguins.
A. marring B. dotting C. shifting D. altering
18. Some _________ left the door open overnight. How stupid!
A. scum of the earth B. bright spark C. live wire D. nasty piece of work
19. The school of fish, hiding among the jagged coral, _____________ by the hammerhead
shark, despite the gloomy camouflage of the surroundings.
A. were easily seen B. was easily seen C. seen D. being seen
20. Supermarkets were _________ one supplier _________ against another to get the
lowest price.
A. playing/off B. lousing/up C. gnawing/away D. fouling/up
Your answers:

1. 2. 3. 4. 5.

6. 7. 8. 9. 10.

11. 12. 13. 14. 15.

16. 17. 18. 19. 20.

Part 2:(10pts) The passage below contains 10 errors. IDENTIFY and CORRECT them. (0) has been
done as an example.

(0) concerning à concerned


People have been concerning with their hair since ancient times. In 1500 B.C., the Assyrians,
inhabiting the area known today as Northern Iraq, were the World’s first truly hair stylists. Their skills
at cutting, curling, layering and dyeing hair were known through the Middle East. In fact, they were
obsessing with their hair, which was oiled, perfumed, and tinted. A fashionable courtier wore his hair
cut in neat geometric layers. Kings, soldiers and noblemen had
their hair curl with a fire-heated iron bar, probably the world’s first curling iron. So important was hair
styling in Assyria which law dictated certain types of hair styles according to a person’s position and
employment. Facial hair was also important. Men grew beards down from their chests and had them
clipped in layers. High-rank women in both Egypt and Assyria wore fake beards during official court
business to show their equal authority with men.
As the Assyrians, the early Greeks liked long, scented, curly hair. Fair hair was favored over dark, so
those who were not “ natural blonds” lightened or reddened their hair with soaps and bleaches. The
Romans, on the other hand, favored dark hair for men for high social or politics rank. Early Saxon men
were neither blonds nor brunets but dyed their hair and beards blue, red, green, and orange.
Since the centuries, societies have combed, curled, waved, powdered, dyed, cut, coiffed, and sculpted
their hair, or someone else’s during times of wig crazes. Churches and lawmakers have sometimes
tried to put a stop to the humans obsession with hair, but with few success. It seems hair styling is
here to stay, and the future will likely prove no exception.
Your answers:

Line Mistakes Corrections

1.

2.

3.

4.

5.

6.

7.

8.

9.

10.

Part 3. (10pts) Write the correct form of the words given in the brackets. Write your answers in the
spaces provided below.
THE RNLI
The RNLI (Royal National Lifeboat Institution) is an organization (1)…………(DEPEND) of the
government control, dedicated to saving lives at sea. The RNLI has an active fleet of several hundred
of lifeboats around the length and (2)…………..(BROAD) of Britain’s coastline.
Since it was founded in 1824, the RNLI has had an extremely (3)…………….(SIGNIFY) impact on maritime
safety, saving over 130, 000 lives.
Because the RNLI is not funded by the government, it relies on the (4)……….(GOOD) of the public to
cover its costs, its income coming from membership fees and (5)…………….(CHARITY) donations. Some
people have raised (6)…………..(OBJECT) to this, saying that such (7)……………(VALUE) service should be
government - funded.
The lifeboat crews are almost all volunteers. They are generally (8)…………..(KNOW) by the British
people as being (9)………….(EXAMPLE) in their (10)……………(SELF), frequently putting their lives at risk
to save others.
Your answers:

1. 6.

2. 7.

3. 8.

4. 9.

5. 10.

C. READING (60pts)
Part 1: Read the following passage and mark the letter A, B, C, or D on your answer sheet to
indicate the correct answer to each of the questions.(10pts)
Do you ever wish you were more optimistic, someone who always (1) ............. to be successful?
Having someone around who always fears the worst isn't really a lot of (2) …...... - we all know
someone who sees a single cloud on a sunny day and says, 'It looks like rain.' But if you catch yourself
thinking such things, it's important to do something about it.
You can change your view of life, according to psychologist. It only takes a little…(3)….., and you'll
find life more rewarding as a..(4)................ . .Optimism, they say, is partly about self-respect and
confidence but it's also a more positive way of looking at life and all it has to (5) ………….. . Optimists
are more (6) ............. to start new projects and are generally more prepared to take risks.
Upbringing is obviously very important in forming your (7) ………..... to the world. Some people are
brought up to depend too much on others and grow up forever blaming other people when anything
(8)……..... wrong. Most optimists, on the (9) ……….. hand, have been brought up not to (10)….... failure
as the end of the world - they just get on with their lives.
1. A. counted B. expected C. felt D. waited
2. A. amusement B. play C. enjoyment D. fun
3. A. energy B. effort C. work D. effect
4. A. result B. reason C. purpose D. product
5. A. supply B. suggest C. offer D. propose
6. A. possible B. likely C. hopeful D. welcome
7. A. opinion B. attitude C. view D. position
8 A. goes B. falls C. comes D. turns
9. A. opposite B. others C. other D. far
10 A. regard B. respect C. suppose D. think
Part 2: Fill each blank with ONE suitable word. Write your answers in the numbered blanks provided
below the passage.(15 pts)
TEENAGERS AND TELEVISION
Until recently, teenagers have been hooked on television. Parents have worried that their children are
becoming fat, (1)…………potatoes, and teenagers seem to have preferred watching TV (2)………….almost
any other activity in the home. Except perhaps sleeping. But no more! According to the latest
statistics, teenagers have (3)………..off TV and are turning off in droves. Given the choice (4)………….TV
and the internet, it is clear what most teens prefer. The internet (5)…………an interactive, social need
that TV doesn’t. Teenagers at a loose (6)………..in their bedrooms can hang (7)………..with their mates in
cyberspace. As websites such as My space have (8)…………off, teenagers have been only (9)……….eager
to join in their millions and spend hours a day - and night online. We’re witnessing the birth of the
generation of the “Keyboard potato”, for (10)…………of a better expression.
Your answers:

1. 2. 3. 4. 5.

6. 7. 8. 9. 10.

Part3:(10pts) You are going to read a passage and choose the answer (A, B, C or D) which you think
fits best accordng to the text.
LAKE DISTRICT WATER PLAN SCRUTINISED
A public inquiry opened yesterday into plans to pump extra water from the Lake District National Park
to refill reservoirs drained by drought. United Utilities has submitted its proposals to take water from
two lakes, Windermere and Ullswater, to public scrutiny because of concerns about the potential
damage to wildlife.
Anglers are concerned that spawning sites for salmon and trout could dry out if water is drained from
the rivers that flow from the lakes. But the utilities company insists that removing and treating the
extra water will not hurt local flora and fauna. The hearing, headed by the government inspector,
Stuart Nixon, is being held in the Cumbrian town of Windermere and will last for two days. A final
decision is expected to be taken later this month by Margaret Beckett, the Environment Secretary.
United Utilities insists that the drought orders are necessary to prevent the further depletion of water
from two of its reservoirs in the Lake District, Haweswater and Thirlmere. Because of the unusually dry
summer this year, Haweswater has only 53 per cent of its capacity compared with 68 percent at the
same time last year. Thirlmere has just 47 percent, whereas last year it had 79 percent.
If United Utilities is given the go-ahead, it would be able to take extra water from the Windermere
and Ullswater rivers – Leven and Eamont respectively – this winter, rather than having to take
emergency measures next year. Water from Ullswater would be piped into Haweswater reservoir;
water from Windermere would enter the local supply, and prevent further depletion of Thirlmere.
Water supplies would reach two million people in Manchester, Lancashire, south Cumbria and parts of
Cheshire.
Cumbria Wildlife Trust said taking extra water could pose problems for wildlife if not properly
managed. But a spokesman said it was not opposing United Utilities' plans because it was better for
water to be drained in the wet winter months rather than in the summer. ‘If they don’t have to do it
now, they will have to do it in April,’ the spokesman said.
Dickon Knight, the agent for the landowner Holker Estates, said the proposed minimum flows on the
river Leven would harm efforts to protect salmon stocks. Alistair Maltby, the manager of the Eden
Rivers Trust, said taking water from the rivers during the winter was the best approach but urged
United Utilities to mend leaks in its pipes in the long run. Gary Dixon, customer service manager at
United Utilities, said, ‘We can’t predict what supplies will be like over the winter. Normally this is when
our reservoirs would refill but if the low rainfall continues we need to start planning ahead for next
summer. Taking action now will have a lower impact on the river environment than during the spring.’
The company said the deluge of rain in recent weeks was helping to top up the reservoirs. John
Carberry, a spokesman, said, ‘There is no crisis and no panic, but we are looking ahead for next year.
There is a potential impact on the environment which is why we are seeking permission to do what we
want to do.’ The Environment Agency said it would work with the utility company to ensure any
damage to fish stocks was kept to a minimum.
1. According to the opening paragraph, the controversy arose due to ________.
A a proposed scheme to top up water supplies.
B excessive water in local lakes.
C civilians protesting over a threat to the environment.
D a possibility that reservoirs could be contaminated.
2. The word “hearing” in paragraph 2 is closest in meaning to ________.
A. trial B. legal action C. lawsuit D. official meeting
3. Who does the outcome of the proposal ultimately rest with?
A the area’s fishermen B a utilities company
C Margaret Beckett D Stuart Nixon
4. What have high temperatures resulted in?
A several bodies of water in the Lake District completely drying up
B United Utilities wanting to take certain measures
C two lakes being left with less than 50% of their water
D a large decrease in profits for United Utilities
5. The word “go-ahead” in paragraph 2 is closest in meaning to ________.
A. leave B. consensus C. authorization D. permit
6. If the plan is implemented, _______.
A it will inevitably avert a national crisis.
B its impact will be felt nationwide.
C it could cause irreversible damage.
D it should preferably be done in winter.
7. Which of the following statements is NOT true, according to the passage?.
A the water depletion rate of all reservoirs is not the same.
B Wildlife might be affected by excessive water taking
C It’s inevitable that water should be drained sooner or later.
D Water from two rivers would be pumped into the local supply
8. What is Alistair Maltby’s opinion on refilling reservoirs?.
A It will be detrimental to certain species of fish.
B It’s the best solution as long as a technical problem is sorted out.
C It will have a negative effect on the environment if left until spring.
D Its success depends on how well United Utilities manage the project.
9. It can be inferred from the passage that ________.
A it’s spring at the moment the passage was written
B salmon stocks are being kept to a minimum
C there will be an active collaboration for the common good
D water depletion has reached epidemic proportions
10. What is the overall tone of the passage?
A alarmed B sarcastic C neutral D dismissive
Your answers:

1. 2. 3. 4. 5.

6. 7. 8. 9. 10.
Part 4: (15pts) Read the following passage and answer the questions.
A. After hours of driving south in the pitch-black darkness of the Nevada desert, a dome of hazy gold
suddenly appears on the horizon. Soon, a road sign confirms the obvious: Las Vegas 30 miles. Looking
skyward, you notice that the Big Dipper is harder to find than it was an hour ago.
B. Light pollution—the artificial light that illuminates more than its intended target area—has become
a problem of increasing concern across the country over the past 15 years. In the suburbs, where
over-lit shopping mall parking lots are the norm, only 200 of the Milky Way’s 2,500 stars are visible on
a clear night. Even fewer can be seen from large cities. In almost every town, big and small, street
lights beam just as much light up and out as they do down, illuminating much more than just the
street. Almost 50 percent of the light emanating from street lamps misses its intended target, and
billboards, shopping centres, private homes and skyscrapers are similarly over-illuminated.
C. America has become so bright that in a satellite image of the United States at night, the outline of
the country is visible from its lights alone. The major cities are all there, in bright clusters: New York,
Boston, Miami, Houston, Los Angeles, Seattle, Chicago, and, of course, Las Vegas. Mark Adams,
superintendent of the McDonald Observatory in west Texas, says that the very fact that city lights are
visible from on high is proof of their wastefulness. “When you’re up in an airplane, all that light you
see on the ground from the city is wasted. It’s going up into the night sky. That’s why you can see it.”
D. But don’t we need all those lights to ensure our safety? The answer from light engineers, light
pollution control advocates and astronomers is an emphatic “no.” Elizabeth Alvarez of the
International Dark Sky Association (IDA), a non-profit organization in Tucson, Arizona, says that overly
bright security lights can actually force neighbours to close the shutters, which means that if any
criminal activity does occur on the street, no one will see it. And the old assumption that bright lights
deter crime appears to have been a false one: A new Department of Justice report concludes that
there is no documented correlation between the level of lighting and the level of crime in an area. And
contrary to popular belief, more crimes occur in broad daylight than at night.
E. For drivers, light can actually create a safety hazard. Glaring lights can temporarily blind drivers,
increasing the likelihood of an accident. To help prevent such accidents, some cities and states
prohibit the use of lights that impair night-time vision. For instance, New Hampshire law forbids the
use of “any light along a highway so positioned as to blind or dazzle the vision of travellers on the
adjacent highway.”
F. Badly designed lighting can pose a threat to wildlife as well as people. Newly hatched turtles in
Florida move toward beach lights instead of the more muted silver shimmer of the ocean. Migrating
birds, confused by lights on skyscrapers, broadcast towers and lighthouses, are injured, sometimes
fatally, after colliding with high, lighted structures. And light pollution harms air quality as well:
Because most of the country’s power plants are still powered by fossil fuels, more light means more
air pollution.
G. So what can be done? Tucson, Arizona is taking back the night. The city has one of the best lighting
ordinances in the country, and, not coincidentally, the highest concentration of observatories in the
world. Kitt Peak National Optical Astronomy Observatory has 24 telescopes aimed skyward around the
city’s perimeter, and its cadre of astronomers needs a dark sky to work with.
H. For a while, that darkness was threatened. “We were totally losing the night sky,” Jim Singleton of
Tucson’s Lighting Committee told Tulsa, Oklahoma’s KOTV last March. Now, after retrofitting inefficient
mercury lighting with low-sodium lights that block light from “trespassing” into unwanted areas like
bedroom windows, and by doing away with some unnecessary lights altogether, the city is softly
glowing rather than brightly beaming. The same thing is happening in a handful of other states,
including Texas, which just passed a light pollution bill last summer. “Astronomers can get what they
need at the same time that citizens get what they need: safety, security and good visibility at night,”
says McDonald Observatory’s Mark Adams, who provided testimony at the hearings for the bill.
I. And in the long run, everyone benefits from reduced energy costs. Wasted energy from inefficient
lighting costs us between $1 and $2 billion a year, according to IDA. The city of San Diego, which
installed new, high-efficiency street lights after passing a light pollution law in 1985, now saves about
$3 million a year in energy costs.
J. Legislation isn’t the only answer to light pollution problems. Brian Greer, Central Ohio
representative for the Ohio Light Pollution Advisory Council, says that education is just as important, if
not more so. “There are some special situations where regulation is the only fix,” he says. “But the
vast majority of bad lighting is simply the result of not knowing any better.” Simple actions like
replacing old bulbs and fixtures with more efficient and better-designed ones can make a big
difference in preserving the night sky.
Questions 1- 6
The first six paragraphs of the reading passage are lettered A-F. Choose the most suitable headings for
paragraphs A-F from the list of headings below.
NB There are more headings than paragraphs, so you will not use them all.

List of Headings

I .Why lights are needed

ii. Lighting discourages law breakers

iii. The environmental dangers

iv. People at risk from bright lights

v. Illuminating space

vi. A problem lights do not solve

vii. Seen from above


viii. More light than is necessary

ix. Approaching the city

1) Paragraph A
2) Paragraph B
3) Paragraph C
4) Paragraph D
5) Paragraph E
6) Paragraph F
Questions 7-10
Complete each of the following statements with words taken from the passage.
Write ONE or TWO WORDS for each answer.
7) According to a recent study, well-lit streets do not .................... or make neighbourhoods safer to
live in.
8) Inefficient lighting increases .................... because most electricity is produced from coal, gas or oil.
9) Efficient lights .................... from going into areas where it is not needed.
10) In dealing with light pollution .................... is at least as important as passing new laws.
D. WRITING (50 pts)
Part 1: Rewrite the sentences with the given words or beginning in such way that their meanings
remain unchanged.(20pts)
1. Turning down that job was very foolish of you.
à You.............................................................................................................................
2. Many species of wild life are threatened with extinction.
à Many species .............................................................................................................
3. She didn’t shed a tear when the story ended in tragedy
à Not a tear ...................................................................................................................
4. You’d be wasting your time trying to make him change his mind
à It................................................................................................................................
5. We don’t seem to have much sugar left!
à We.................................................................................................................................
6. He seems to be more active because he has won a scholarship to study overseas.
(lease)
à That he has won ...........................................................................................................
7. It wasn’t until last week that the minister admitted he was wrong. (error)
à Only ..........................................................................................................................
8. The effects of the gale were felt mainly along the south coast. (brunt)
à The south coast……………………………………………………………………….the gale.
9. Getting upset over Michael’s departure is pointless. (tears)
à There is no………………………………………………… over Michael’s departure.

10. I have become extremely good at missing the rush hour over the last few weeks (fine)
à I have got……………………………………………………………………….…last few weeks.
TEST 7
A. LISTENING (50 points)
Part 1: Listen to the recording twice. Complete the table below. Write ONE WORD ONLY for each
answer. (10 points)

URBAN vs SUBURBAN LIVING

Advantages Disadvantages

Urban Action – clubs, cafẻs etc. Noise


1.__________ More 2. __________ and pollution
Arts & Culture – museums etc. Price of housing
Close to work

Suburban Peace & quiet Cost of 4. __________


Greener environment Time lost in daily 5. __________
Better for children
Homes larger with more
3.__________. – back yard etc.

1. 2. 3. 4. 5.
Part 2:You will hear an interview with a man called Mike O’Toole, who works as a teacher trainer. For
questions 1-5, choose the answer (A, B, C or D) which fits best according to what you hear. (10 points)
1. Why are many teachers leaving the profession, according to Mike?
A. They don’t feel it is financially rewarding any more.
B. They are not being given the respect they once were.
C. They are investing too much in it without getting enough back.
D. They find the subject matter they have to teach too difficult.
2. Mike believes that without radical changes
A. education in the UK will begin to get worse.
B. UK schools will lose their ability to compete with one another.
C. the UK will soon no longer be a leader in education.
D. the educational system in the UK can be transformed.
3. The main failing of the UK education system is
A. the inability of students to use computers.
B. the resistance within schools to the introduction of technology.
C. the failure of government to invest in hardware for schools.
D. the lack of training for teachers in the use of technology in the classroom.
4. Why is teacher training failing teachers, according to Mike?
A. It is not helping them to keep pace with developments.
B. It does not use technology in its training classrooms.
C. It uses hardware and software that are out of date.
D. It is suffering from a lack of trainers with specialist knowledge.
5. Teachers can only benefit from technology if
A. they cease to see it as a threat.
B. they can combine it with traditional methods.
C. they are allowed to use it in their own way.
D. they are willing to research its possibilities on their own.

1. 2. 3. 4. 5.
Part 3: Listen to the recording twice. Decide whether the following statements are true (T) or false (F).
Write T or F in the space provided. (10 points)
1. The speaker has come from the Theosophical Society. _____
2. One of the main points of the talk is to save money. _____
3. The woman thinks students should do more home work. _____
4. The woman argues that plastic containers won’t biodegrade quickly. _____
5. The woman warns that asthma sufferers should be careful with her recipes. _____

1. 2. 3. 4. 5.

Part 4: Listen to the recording twice. You will hear part of a radio talk for young people about
animals communicating with each other. Complete the sentences with NO MORE THAN THREE
WORDS. (20 points)
Bees do a (1.__________) to communicate where to find food.
Although parrots seem to speak, they are only (2.__________) the human sounds.
Primates can communicate a few (3.__________) using simple sounds.
Monkeys have not been observed to use any kind of (4.__________).
Although dolphins can make vowel sounds, they cannot accurately imitate our (5.__________)
Amazingly, dolphins demonstrate an (6.__________) of when to use phrases.
The sounds made by whales contain (7.__________) than human speech.
The songs of the bottle-nosed whale have many of the (8.__________) of human speech.
The unique grammatical nature of human language arose due to life in (9.__________).
Indeed, a young child needs enough (10.__________) with other people to develop speech.

1. 2. 3. 4. 5.

6. 7. 8. 9. 10.

B. LEXICO-GRAMMAR (50 points)


Part 1 : Choose the correct option for each gap in the following sentences. (20 points)
1. Please accept our _____ congratulations!
A. finest B. warmest C. dearest D. deepest
2. You should _____ at least three days for the journey.
A. expect B. permit C. accept D. allow
3. I had no chance to defend myself: the dog _____ for me as soon as I opened the door.
A. went B. ran C. fell D. stood
4. Please _____ a copy of your application form for at least six months.
A. return B. revise C. retain D. refer
5. _____ , dolphins have no sense of smell.
A. As known as far B. It is known as far
C. Known thus far as D. As far as is known
6. Alone in a deserted house, he was so busy with his research work that he felt _____ lonely.
A. nothing but B. everything but C. anything but D. all but
7. Good health enables people to enjoy life and have the _____ to achieve their gold.
A. chance B. opportunity C. occasion D. situation
8. The current political _____ of our country is favorable for foreign investments.
A. climate B. weather C. temperature D. state
9. They drove _____ the home when their car run out of oil.
A. in a mile to B. for a mile to C. within a mile to D. within a mile by
10. If the size of the workforce can be easily and rapidly altered _____ market fluctuations, profits will
be maximized.
A. in terms of B. in response to C. in reply to D. with respect to
11. Leo was ________ from the meeting after he began objecting loudly to other people’s ideas.
A. ejected B. rejected C. dejected D. injected
12. He was completely ________, sleeping on the streets and begging for money.
A. down-and-out B. grin and bear C. prim and proper D. fits and starts
13. It was ________ whether the operation would go ahead because so many staff were on sick leave
that week.
A. safe and sound B. touch-and-go C. pros and cons D. grin and bear
14. Everything was at ________ when we arrive, as they had only moved into the house the day
before.
A. fits and starts B. safe and sound C. sixes and sevens D. song and dance
15. My toothache is worse but until I can get to the dentist, I’ll just have to ________ it.
A. grin and bear B. prim and proper C. get rid of D. cast a spell on
16. The play is simply a vehicle for its stars and falls ________ of having a decent plot.
A. fast B. short C. quick D. thin
17. Anna’s friend knew the casting director, so she pulled a few ________ to arrange an audition.
A. ropes B. wires C. threads D. strings
18. Only a few companies were found to be in ________ with the new laws.
A. submission B. obedience C. compliance D. fulfillment
19. Miguel felt he was being overlooked, which is why he ________ back rudely when his manager
finally asked him what he thought.
A. answered B. responded C. replied D. uttered
20. As Mary was an ________ member of the team, everyone was disappointed when she was
announced her resignation.
A. intensive B. interior C. internal D. integral

1. 2. 3. 4. 5. 6. 7. 8. 9. 10.

11. 12. 13. 14. 15. 16. 17. 18. 19. 20.

Part 2: The passage below contains 10 mistakes. Underline the mistakes and correct them in the
space provided in the column on the right. (0) has been done as an example. (10 points)

Why I dislike computers Line 1……..


Most everyone says that computers are wonderful and that they are changing Line 3……..
our lives for the better by making everything faster and more reliable, but I’m
Line 4……..
not so sure that this is a case.
Line 5……..
The another day I was standing in a large department store waiting to pay for a
couple of films for my camera while the assistant announced that the computer Line 7……..
which controlled the till had stopped working. I didn’t think this was a big
Line 9……..
problem and I set up to find another counter, but of course, all the machines are
part of the same system. So there we were: a shop full of customers, money at Line 11……..
the ready, waiting to do our purchases, but it was quite clear that none of the Line 13……..
assistants knew what to do. They weren’t allowed to take our money and give
customers a written receipt, so that the sales wouldn’t then have been recorded Line 14……..
on the computer system.
Line 17……..
At the end, like many other people, I left my shopping on the counter and
walked out. Don’t you think that’s ridiculous? It will never have happened before
computers, and that, for me, is the problem: we are beginning to depend on
these machines so completely that we simply can’t manage without them no
more.

1. 2. 3. 4. 5.

6. 7. 8. 9. 10.

Part three: Fill each gap in the following sentences with only one suitable preposition particle. (10
points)
1. His honesty _____ question; nobody can doubt it.
2. The building of the new road has been held _____ by bad weather.
3. We can get _____ with eight computers in the lab at the moment, but we’ll need a couple more
when the new staff arrive.
4. She established the school in 1960 and since then tens of thousands of children have passed _____
her school.
5. Judging _____ the time of the day when something is done, one can decide how important an
event is.
6. Her business has gone _____, and she has lost everything.
7. I can’t concentrate on my work with the prospect of the court case hanging _____ me.
8. Peter was an inspirational politician, who put _____ his ideas with clarity.
9. His ball control skills really set _____ him from the rest of the players.
10. Can you check _____ these figures against last year’s figures. I’d like to know which year was more
successful.

1. 2. 3. 4. 5.

6. 7. 8. 9. 10.
Part 4 : Use the correct form of the words in brackets to complete the passage. (10 points)
The upper layers of Earth’s oceans have cooled (1. SIGNIFY)__________ over the past two years, even
though the planet as a whole is warming up. While this may just be part of the natural (2. VARY)
__________ of oceans, (3. CLIMATE) __________ are still confounded by the massive uncountable
loss of heat. Scientists have been (4. INCREASE) __________ concerned by rising sea temperatures
over the last 50 years but these new (5. FIND) __________ tell a different story. Generally speaking,
the (6. ABSORB) of heat by the oceans reduces atmospheric warming. Now (7. MEASURE)
__________ taken by the National Oceanic and Atmospheric Administration have put a wrinkle in the
trend. The researchers used data from 3000 floating buoys which monitor the oceans (8. WORLD)
__________.They found that the oceans dropped in temperature by an (9. BELIEVE) __________ 0.02
degrees centigrade between 2011 and 2013. Now, that may not seem like much, but trying to account
for the missing energy is proving to be enormously (10. PROBLEM) __________. It is possible that
volcanic eruptions are one main cause of the phenomenon, but no firm answers have yet been
provided.

1. 2. 3. 4. 5.

6. 7. 8. 9. 10.

C. READING (50 points)


Part 1. Read the following passage and decide which answer (A, B, C, or D) best fits each gap. Write
your answers in corresponding numbered boxes. (10 pts)
The money that some professional sportsmen earn shouldn’t impress anyone when you take into
consideration the fact that only a few of them manage to (1) ____________ immorality and
everlasting fame. And once they reach their prime and display their talent at their best, they are (2)
____________ conscious that their brilliant careers won’t last forever. They live under a constant
pressure of being (3) ____________ and subsequently replaced by someone who is younger, faster
and more accomplished. For that reason, objectives like retirement benefits and pensions are of great
concern to all professional athletes.
Some of the retired competitors go as far as to organize strikes and rallies to (4) ____________ their
protest against any policy unresponsive to their demands whereas the younger professionals seek
more upgrading solutions to the problem as more and more of them attach a proper significance to
(5) ____________ a solid education, even at university level. Such an approach should help them find
interesting and well-paid jobs (6) ____________ their sports career is over.
A completely new strategy has been (7) ____________ by the schools priding themselves (8)
____________ supporting their own teams. Their authorities insist that the sports club members
achieve high academic standards or else they are debarred from partaking in certain sports events,
which may lead to further (9) ____________ in their professional careers.
By these practical and most effective means, combining education with sports activity, the (10)
____________ of the professional athlete as being brainless and unintelligent may eventually be
changing to the sportsmen’s benefit.
1. A. acquire B. fulfill C. attain D. succeed
2. A. fully B. extremely C. terribly D. very
3. A. outcast B. outshone C. outstayed D. outgrown
4. A. voice B. claim C. insist D. speak
5. A. mastering B. learning C. receiving D. attending
6. A. right away B. promptly C. barely D. once
7. A. assembled B. installed C. devised D. emerged
8. A. with B. on C. for D. in
9. A. disruption B. disturbance C. detachment D. damage
10. A. vision B. outlook C. image D. judgment
Your answers

1. 2. 3. 4. 5.

6. 7. 8. 9. 10.

Part 2. Read the text below and think of the word which best fits each space. Use only ONE word in
each space. Write your answers in the corresponding numbered boxes. (15 points)
Throughout history people have worn clothing of one description or another. Apart from protection
against the weather, clothes were also often used to show the wearer’s status and wealth. Over the
years, numerous fashions in clothing have come and gone. (1) ____________ some of these have
been popular for relatively short periods, others have lasted longer.
Until the first half of the 20th century, the ability to follow fashion was limited to those
(2) ____________ had the money to (3) ____________ so. But following fashion did not only demand
money, it also required large amounts of leisure time. Wealthy people took fashion very seriously and
close attention had to (4) ____________ paid to detail. Wearing the correct clothes for different
occasions was very important, despite the (5) ____________ that this often meant changing clothes
five or six (6) ____________ day.
More recently, fashionable clothes have come within the reach of ordinary people. The traditional
craft of dressmaking, (7) ____________ usually involved sewing (8) ____________ hand, was both
costly and slow. But today, large-scale manufacturing has made it easier for people to keep
(9) ____________ with changes in fashion (10) ____________ having to spend a great deal of money.
Your answers

1. 2. 3. 4. 5.

6. 7. 8. 9. 10.

Part 3. Read the passage and choose the best option A, B, C, or D to answer the questions. Write
your answers in the corresponding numbered boxes. (10 points)
Tides are the periodic rise and fall of the Earth’s waters that are caused by the Moon’s and Sun’s
forces of gravity acting on the Earth. It is important to distinguish natural tidal phenomena from huge
tsunamis, with the later being caused by earthquakes and undersea volcanic eruptions.
The Moon is a main factor controlling ordinary tides. At the location on the Earth closest to the
Moon, it exerts a powerful gravitational pull on the water. The resulting rise in the water produces
higher tides. The water on the side of the Earth farthest away from the Moon also gets pulled by this
lunar gravity, but not as strongly. The Earth itself has its own gravitational force that is constantly
pulling waters downward, which is why the oceans do not simply bulge out toward the Moon.
Ordinary tides usually feature high and low waters alternating in relation to the Earth’s rotation. Most
shores around the world have two high waters and two low waters for each day, which last about 24
hours and 50 minutes. The difference in height between high water and low water is called the range
of tide, and it can be quite dramatic in narrower bays. Canada’s bays of Fundy, for example, commonly
experiences the world’s most extreme tidal ranges, with daily differences of 16 meters.
Two other types of tides are influenced by the Sun, which is much farther away from the Earth
and exerts less than half of the Moon’s gravitational force. When the Sun, the Moon and the Earth are
directly in line, the solar and lunar gravitational forces and up to produce higher spring tides. The
range of spring tides is intensified, with higher high water marks and lower low water marks. However,
when the Moon is in the first or third quarter, it is at a 90-degree angle with the Sun in relation to the
Earth. The opposing solar and lunar forces partially cancel each other out, and the result is a lower
tide. This is called a neap tide, which comes twice a month and has lower high water marks and higher
low water marks. The range of neap tides is minimum.
Some tides do not occur over water at all. The solid body of the Earth has slight elasticity, so lunar
and solar gravity causes it to stretch very subtly. These changes in the Earth’s shape,
although imperceptible to humans, are known as Earth tides. Another tidal phenomenon,
atmospheric tides, is caused by the Sun’s heating of the Earth’s atmosphere. Like ordinary tides, they
usually occur over 12- hour periods.
Question 1. What is the main idea of the passage?
A. Tides are influenced more by the Moon than by the Sun.
B. Tides are a natural phenomenon but are not a natural disaster.
C. Tides always occur over water and are usually predictable.
D. Tides are a phenomenon mainly caused by forces of gravity.
Question 2. Why does the author mention tsunamis in the passage?
A. To explain that not all tides are caused by gravity
B. To give an example of an extreme tidal phenomenon p
C. To show that they are not related to natural tides
D. To suggest that more categories for tides area needed
Question 3. The word “exerts” in paragraph 2 is closest in meaning to
A. mixes with B. brings into use
C. infers from D. connects with
Question 4. The word “it” in paragraph 2 refers to
A. height B. the high water
C. the low water D. the range of tide
Question 5. What can be inferred about Canada’s Bay of Fundy?
A. It may experience tsunamis because of its extreme tides.
B. It may have a longer tide cycle because of its wide variations.
C. It may be very narrow because it has wide tidal variations.
D. It may be influenced more by gravity than other places.
Question 6. The word “imperceptible” in the last paragraph could best be replaced by
A. not noticeable B. difficult to explain
C. not generally known D. not able to be said
Question 7. According to paragraph 3, it is true that
A. neap tides are characterized by lower water during the low tide period.
B. spring tides occur when the Moon is in the first and the Third quarter.
C. the Sun has more gravitational force than the Moon because of its larger size.
D. the range of tide is greater in spring tides than in ordinary tides.
Question 8. What is true about the Moon’s gravitational force?
A. It pulls water on the far side of the Earth more strongly
C. It is more than twice as powerful as that of the Sun.
B. It has reduced gravity when it is lined up with the Sun and the Earth.
D. Its force is strongest when it is located at 90 degrees to the Earth.
Question 9. Besides ordinary tides, how many other types of tide are mentioned in the passage?
A. two B. three C. four D. five
Question 10. Which of the following does not relate to ordinary tides?
A. receiving greatest influence from the Moon
B. influenced by the Sun’s position relative to the Moon
C. taking turns the higher and lower water relating to the Earth’s rotation
D. having the striking range of tide in narrower bays
Your answers

1. 2. 3. 4. 5.

6. 7. 8. 9. 10.

Part 4. Read the following passage and do the tasks that follow. (15 points)
CLONING
A. The ethics of human cloning has become a great issue over the past few years. The advocates for
both sides of the issue have many reasons to clone or not to clone. A recent poll has shown the
differences in opinions with half as many women as men approving of the process. Many people find
it strange to see such a clear difference between men and women with twenty-six percent of men
favouring cloning.
B. So, what is cloning? It has been defined as “the production of genetically identical organisms via
somatic cell nuclear transfer”. You take an egg and remove its nucleus, which contains the DNA/genes.
Then you take the DNA from an adult cell and insert it into the egg, either by fusing the adult cell
with the enucleated egg, or by a sophisticated nuclear transfer. You then stimulate the reconstructed
egg electrically or chemically and try to make it start to divide and become an embryo. You then use
the same process to implant the egg into a surrogate mother that you would use with artificial
insemination. What cloning does is that it copies the DNA/genes of the person and creates a genetic
duplicate. The person will not be a Xerox copy. He or she will grow up in a different environment than
the clone, with different experiences and different opportunities. Genetics does not wholly define a
person and the personality.
C. In February 1997, when embryologist Ian Wilmut and his colleagues at Roslin Institute in Scotland
were able to clone a lamb named Dolly, the world was introduced to a new possibility and will never
be the same again. Before this, cloning was thought to be impossible, but now there is living proof
that the technology and knowledge to clone animals exist. Questions began to arise within
governments and scientific organizations and they began to respond. Are humans next? Is it possible
to use this procedure to clone humans also? Would anyone actually try? What can we learn if we
clone humans? How will this affect the world? These are only a few of the questions that have
surfaced and need answering. A whole new concept in ethics was created when the birth of Dolly was
announced.
D. When the cells used for cloning are stem cells, we are talking about cells that are pluripotential.
This means that they have the capacity to develop into any of the numerous differentiated cell types
that make up the body. Early embryonic cells are pluripotent and a limited number of stem cells are
also found in adults, in bone marrow for instance. There is an important distinction to be made
between therapeutic cloning and reproductive cloning. Reproductive cloning would be exactly like
Dolly; it would involve the creation of a cloned embryo which would then be implanted into a womb
to develop to term and the birth of a clone. On the other hand, therapeutic cloning involves the use of
pluripotent cells to repair damaged tissue, such as found after strokes, Parkinson’s disease and spinal
cord injuries.
E. There is evidence for the effectiveness of therapeutic cloning as shown by work involving the
introduction of stem cells into the brain of patients suffering from brain diseases, when the cells which
have been added differentiate to form nerve cells which can in turn then lead to recovery of the lost
function. In the US, foetal human cells have been similarly used though recent reports indicate that
the results so far are disappointing. However, apart from the ethical problems associated with the use
of foetal cells in this way, there are simply not enough cells available for it to be an effective
treatment, since it needs the cells from three foetuses to treat one patient.
F. After Dolly, governments began to take control and make laws before anything drastic could ever
happen. Several ethics committees were asked to decide whether scientists should be allowed to try
to clone humans. In the United States, the Bioethics Advisory Commission recommended a five-year
moratorium on cloning a child through somatic cell nuclear transfer. In the United Kingdom, the
Human Fertilization and Embryology Authority and the Human Genetics Advisory Commission have
approved human cloning for therapeutic purposes, but not to clone children. Many organizations have
come out and stated their opinions also. Amongst all this ethical defining, many people are being
ignored by the governments. People are speaking out about what they want done.
G. Historically, we find that many a great medical breakthrough, now rightly seen as a blessing, was in
its own time condemned by bio-conservative moralists. Such was the case with anesthesia during
surgery and childbirth. People argued that it was unnatural and that it would weaken our moral fibre.
Such was also the case with heart transplantations and with in vitro fertilization. It was said children
created by IVF would be dehumanized and would suffer grave psychological harm. Today, of course,
anesthesia is taken for granted; heart transplantation is seen as one of medicine’s glories and the
public approval rate of IVF is up from 15% in the early seventies to over 70% today.
Task one: The passage has 7 paragraphs (A - G). Which paragraph offers information on the following
ideas? Write the appropriate letters (A - G) in the gaps for questions 1 - 6. One paragraph is used more
than once and two are not used at all.
1. Different types of cloning. _______
2. Protective legislation. _______
3. Similar situations _______
4. A survey on attitudes towards cloning. _______
5. Scientific reasons why cloning is currently not viable for medical cures. _______
6. Illness examples that cloning could help treat. _______
Task two: Choose the correct answers.
7. Which of the following is a feature of cloning used for possible medical treatment?
A. A genetic duplicate is born through use of a surrogate mother.
B. The creation and implantation of an embryo.
C. Implantation of differentiated pluripotent cells.
D. Genetic mutation of pluripotent cells.
8. Which of the following is NOT a feature of cloning?
A. Reproduction of a genetic copy of the subject cloned.
B. Reproduction of exact personality characteristics of the subject cloned.
C. Reproduction of the pluripotential stem cells of the subject cloned.
D. Reproduction of the DNA of the subject cloned.
Task three: Use NO MORE THAN THREE WORDS from the passage to answer the questions
9. In what part of an egg is found the DNA used for cloning? ________________
10. According to the text, what body wants to wait before cloning a human? _______________

Your answers

1. 2. 3. 4. 5.

6. 7. 8. 9. 10.

D. WRITING (50 points)


Part 1. Finish each of the following sentences in such a way that it means the
same as the one printed before it. Write your answers in the space provided (10
points)
1. Two men stole the old lady's handbag.
The old lady was ……………………………………………………………
2. The minister gave no precise figures about the casualties.
The minister didn't go ………………………………………………………
3. You just can’t compare the quality of her work with his.
There’s no …………………………………………………………………...
4. Fares will be very likely to go up again this autumn.
It looks ………………………………………………………………………
5. For further information, please send a self-addressed envelope to the above address.
Further information can ………………………………………………………
Part II. Rewrite the sentences below in such a way that their meanings stay the same. You must use
the words in capital without changing their forms. Write your answers in the space provided (10
points)
1. One of our philosophers is supposed to have said this. (ATTRIBUTED)

………………………………………………………………………………………

2. A government official leaked the story to the world press. (WIND)


………………………………………………………………………………………
3. He's becoming very successful. (PLACES)
………………………………………………………………………………………
4. They're faced with the choice of two alternatives. (HORNS)
………………………………………………………………………………………
5. Since the advertisement, we’ve had more applications than we can deal with. (SWAMPED)
………………………………………………………………………………………
TEST 8
A. LISTENING
Part 1: You will hear a conversation between a Scottish student called John and a Finish student
called Pirkko about the Tampere Student Games in Finland. For questions 1-5, complete the notes
below. Write NO MORE THAN THREE WORDS AND/OR A NUMBER for each answer in the
corresponding numbered boxes.
Tampere Student Games
 Dates of the games: (1) ______________
 Cost of taking part (2) ______________ euros per day each
 Entry fee includes competition entrance, meals and (3) ______________
 Hotel (4) ______________ has a special rate during the games
 Hotel is close to (5) ______________
 Website address: www.sellgames.com
Your answers:

1. 2. 3. 4. 5.

Part 2: You will hear a radio programme called Future world and decide if the following sentences
are True (T) or False (F). Write your answers in the corresponding numbered boxes.
1. V2V involves cars’ talking to each other' through a computer system.
2. The new mobile phone will tell you how nervous or confident you look.
3. The new mobile phone was invented to help people during ‘speed dates’.
4. The memory device is not just one machine.
5. According to Gordon Bell, recording your life is rather dull, but may be important in the future.
Your answers:

1. 2. 3. 4. 5.

Part 3: You will hear two psychologists talking about modern childhood. For each of the following
questions, choose the option which fits best to what you hear.
1. What does Daniel imply about past images of childhood?
A. They are entirely fictional.
B. They are diverse.
C. They represent the innocence of childhood.
2. When mentioning the children throwing bags on the bus-stop, Louise is
A. critical. B. angry. C. sarcastic.
3. According to Daniel,
A. children are failing to learn adequate social skills.
B. children do not eat a balanced diet.
C. children are far more sociable than they used to be.
4. What does Louise say about the media?
A. Manipulative actors have a negative effect on children.
B. Adverts are aimed more at young people than adults.
C. It glorifies unrealistic ideals.
5. Daniel implies that
A. children would be happier if their parents taught them at home.
B. machines are more of a menace to children than people are.
C. teachers aren’t helping children to be competitive enough.
Your answers:

1. 2. 3. 4. 5.

Part 4: You will hear part of a radio talk for young people about animals communicating with each
other. For questions 1-10, complete the notes below. Write NO MORE THAN THREE WORDS AND/OR
A NUMBER for each answer in the corresponding numbered boxes.
 Bees do a (1) ______________ to communicate where to find food.
 Although parrots seem to speak, they are only (2) ______________ the human sounds.
 Primates can communicate a few (3) ______________ using simple sounds.
 Monkeys have not been observered to use any kind of (4)______________
 Although dolphins can make vowel sounds, they cannot accurately imitate our (5)
______________
 Amazingly, dolphins demonstrate an (6) ______________ of when to use phrases.
 The sounds made by whales contain (7) ______________ than human speech.
 The songs of the bottle-nosed whale have many of the (8) ______________ of human speech.
 The unique grammatical nature of human language arose due to life in (9) ______________ .
 Indeed, a young child needs enough (10) ______________ with other people to develop
speech.
B. LEXICO-GRAMMAR
Part 1. Choose the best option A, B, C, or D to complete the following sentences and write your
answers in the corresponding numbered boxes.
1. Business leaders predict a hard year ahead with the economy _________.
A. on the rocks B. on the cards C. in the black D. in the doldrums
2. A punctual man himself by nature, he detested the thought of any _______ delays, and
so roused Peter as early as he dared.
A. groundless B. unfounded C. improper D. untoward
3. In preparation for a lucky New Year, my parents decorate the living room, while my sister
and I __________ the attic.
A. conk out B. clam up C. turn out D. back into
4. Prosperity and happiness arrive to reward her confidence in a _______ economy.
A. floating B. buoyant C. dynamic D. fluid
5. Global warming has progressed ________ glaciers everywhere are shrinking.
A. too much that B. enough to cause
C. to such an extent that D. so great an extent that
6. The authority is going to great lengths to _________ war on dangerous driving.
A. launch B. stage C. boot D. wage
7. During the rush hour, the traffic in big cities is _________.
A. devil B. crime C. fire D. murder
8. The problem with losing weight is that, if you succeed, all your clothes need to be
________.
A. taken in B. cut down C. decked out D. made down
9. In bas-relief sculpture, a design projects very slightly from its background, ______ some
coins.
A. as on B. with which C. outwith D. similarly
10. Shell decoration is an effective _________ for demonstrating artistic skills, especially in
Easter.
A. method B. means C. vehicle D. drive
11. It’s so unfortunate to have a boss who ________ all the time.
A. follows your nose B. breathes down your neck
C. keeps your temper D. draws your eyes
12. ______ are considered humorous is mainly due to his characters’ use of slang.
A. That Damon Ruyan’s stories B. Damon Ruyan’s stories, which
C. Damon Ruyan’s stories D. Because Damon Ruyan’s stories
13. A(n) ________ love of country lays the foundation for sustainable development and
complete harmony.
A. ingrained B. indelible C. abiding D. established
14. It is necessary that they _________ a bit and examine the history of the problem.
A. backfire B. backtrack C. backlash D. backwash
15. Demand for the product is expected to peak five years from now and then to _________.
A. taper off B. fall down C. set back D. drift away
16. Her comments cast a _______ on the integrity of his employees.
A. blot B. slur C. stain D. drag
17. To apply for this position, each candidate has to submit a ________ photo besides other
required documents.
A. full-bodied B. full-scale C. full-length D. full-fledged
18. During the early period of ocean navigation, __________ any need for sophisticated
instruments and techniques.
A. so that hardly B. when there hardly was
C. hardly was D. there was hardly
19. Although the task is basically a no-brainer, she’s making such a ________ weather of it.
A. heavy B. weighty C. stormy D. rough
20. Although colouring books for adults may raise a few eyebrows, more and more people
are seeking them ________ as a way to relax and de-stress.
A. up B. on C. by D. out
Your answers:

1. 2. 3. 4. 5.
6. 7. 8. 9. 10.

11. 12. 13. 14. 15.

16. 17. 18. 19. 20.

Part 2. Read the passage below which contains 10 mistakes. Identify the mistakes and write the
corrections in the corresponding numbered boxes.
LINE TEXT
1 Sociologists believe groups form in two basic ways. One is through social
cohesion, in which is when people come together base on interpersonal
2
attraction, or in other words, when they admire one another’s
3 personality traits. On contrast, other groups form through social identity,
which is centred round a person’s social category. This involves
4
economical status, profession, ethnicity, and other such factors.
5 Furthermore, with social identity, it is important for the individual to
6 include certain people from their group. First of all, they feel distinctly
different from other groups, which strengthens their own identity. A
7 third, less common group, known as an emergent group, forms as
8 opposed to some type of sudden event, such as a disaster. Accordingly,
these individuals had no prior knowledge of each other and may not
9 possess mutually attractive personnel traits or sharing identities, they can
form strong lasting bonds nonetheless.
Your answers:

Line Mistakes Corrections


1.

2.

3.

4.

5.

6.

7.

8.

9.
10
.

Part 3. Write the correct form of the words given in the brackets. Write your answers in the spaces
provided below.
Dealing with phobias
Do you feel hysterical at the thought of spiders? Do you start to shake if you think that you might have
to touch cotton wool? It is estimated that between three to five million Britons suffer from such
phobias, and the majority of these people do not (l. GO) ___________ form of treatment. Most (2.
RATION) ___________ fears begin in the first instance as mild forms of (3. ANXIOUS) ___________ and
only develop gradually into (4.BLOWN) ___________ phobias. They seem to be becoming increasingly
frequent in all sections of society - perhaps because with the expansion of technology, people who are
(5. CUSTOM) ___________ to controlling their (6.STYLE) ___________ with the push of a button panic
when things go wrong. Men are less likely to suffer from such fears than women, but attempts by
either men or women simply to (7. REGARD) ___________ them can exacerbate the problem.
Nowadays, however, phobias can be treated. The easiest option is prescription drugs, which effectively
control the physical symptoms, but may have (8. DESIRE) ___________ side-effects. The other option
is behavioural therapy, in which you gradually learn to (9. COME) ___________ your fear through
facing up to it. This is a safe and lasting (10. ALTERNATE) ___________ to drug treatment.
Your answers:

1. 2. 3. 4. 5.

6. 7. 8. 9. 10.

C. READING
Part 1. Read the following passage and decide which answer (A, B, C, or D) best fits each gap. Write
your answers in corresponding numbered boxes.
There can be no (1) ___________ that online shopping is of huge benefit to the consumer. Far from
becoming (2) ___________, online shoppers are very demanding. Overpriced merchants with poor
services should beware. Gone are the days when stores could charge what they liked for goods and
get away with it. The same, too, for shady manufacturers: smarter consumers know which products
have a good (3) ___________ and which do not, because online they now read not only the sales (4)
___________ but also reviews from previous purchasers. And if customers are disappointed, a few
(5) ___________ of the mouse will take them to places where they can let the world know.
Nowadays there is nothing more damning than a flood of negative comments on the internet.
However, the big boys, as always, are ahead of the game. Some companies are already adjusting their
business models to take account of these trends. The stores run by Sony and Apple, for instance, are
more like brand showrooms than shops. They are there for people to try out (6) ___________ and to
ask questions to knowledgeable staff. Whether the products are ultimately bought online or offline is
of secondary importance.
Online traders must also adjust. Amazon, for one, is (7) ___________ turning from being primarily a
bookseller to becoming a (8) ___________ retailer by letting other companies sell products on its site,
rather like a marketplace. During America’s Thanksgiving weekend last November, Amazon's sales of
consumer electronics in the United States (9) ___________ its book sales for the first time in its
history. Other transformations in the retail business are (10) ___________ to follow.
1. A. query B. examination C. question D. proposal
2. A. complacent B. dissatisfied C. competent D. compassionate
3. A. distinction B. resolution C. opinion D. reputation
4. A. bubble B. message C. blare D. blurb
5. A. taps B. clucks C. clicks D. prods
6. A. devices B. tools C. emblems D. schemes
7. A. mistakenly B. rapidly C. unreasonably D. secretly
8. A. mass B. block C. lump D. chunk
9. A. receded B. excluded C. repressed D. exceeded
10. A. tied B. secured C. bound D. fastened
Your answer:

1. 2. 3. 4. 5.

6. 7. 8. 9. 10.

Part 2. Read the following text and fill in the blank with ONE suitable word. Write your answers in
corresponding numbered boxes.
Graffiti: Art or Vandalifm?
(1) __________ recently, spray-painting a wall would land you in jail, but these days even politicians
are associating with graffiti artists in an effort to gain popularity and internationally acclaimed artist
Banksy, (2) __________ works of art make millions has transformed the way the community views
street art. However, many still see (3) __________ as a crime, especially as the cost of removing
grafitti from walls runs (4) __________ millions of euros every year. Last May, members of a gang
which had left a six-year trail of destruction on trains as (5) __________ apart as Australia and Japan
were jailed for eight months (6) __________ pleading guilty to conspiracy to commit criminal damage.
(7) __________ other form of art has ever divided people so strongly, even (8) __________ the
custom of leaving paintings on walls goes back to the days of cave art. No one would imagine scraping
cave drawings (9) __________ the walls of a cave, and a thousand years from now children may find
(10) __________ studying street artists in school.
adapted from The Olive Press
Your answers:

1. 2. 3. 4. 5.

6. 7. 8. 9. 10.

Part 3. Read the following passage and circle the best answer to each of the following questions.
Write your answers in corresponding numbered boxes.
How birds navigate during migration
Bird migration is one of the most interesting yet least understood natural phenomena. Every fall birds
from northern latitudes fly in groups to the warmer southern latitudes and then return north in the
spring. Scientists agree on the main reasons for migration: to follow the food supply and to avoid
harsh climate conditions. For example, insects disappear during the cold months, prompting insect-
dependent birds to fly south to warm areas where insects breed. No similar consensus has emerged,
however, about how birds are able to navigate. Despite many recent experiments, bird experts still do
not know how birds arrive at the same destination every year and then find their way back home in
the spring.
Some have suggested that birds find their way by following landmarks, such as rivers and mountain
ranges. Experiments have confirmed that some species do follow such topographic features. But that
method cannot explain how some birds travel at night. Other studies show that some nocturnal birds
navigate by the stars. But that explanation cannot explain daytime migration or travel when the skies
are cloudy.
The most popular explanation currently is that birds are guided by Earth’s magnetic poles. The
mechanism by which that works has not yet been proved. One theory points to the fact that some
birds’ brains contain magnetite, a naturally occurring magnetic compound consisting of iron oxide.
Magnetite has been found in many animals, including beds. With magnets embedded their brains,
birds would be able to sense the magnetic fields of the North and South Poles
A recent experiment with homing pigeons provided some evidence that magnetite does play a crucial
role in migration. Homing pigeons are known to have the ability to return to their homes after being
taken hundreds of miles away. Researchers found that they could train homing pigeons to recognize
changes in a magnetic field. When a surrounding magnetic field was normal, the birds would gather at
one end of a cage. But when the field’s polarity was altered, they hoped to the other end, suggesting
that they were detecting and responding to changes in the magnetic field.
Another theory has been offered to explain this sensitivity to magnetic poles, a theory that draws
upon quantum mechanics, which is the study of how particles move inside an atom. It relies on that
fact that electrons come in pairs that orbit the nucleus of an atom. The two electrons spin in opposite
directions, creating two magnets that neutralize each other. But when molecules split and react with
other molecules to form compounds, the electron pairs may no longer spin in opposite directions.
Instead, they may repel each other, as when two north ends of magnets are pressed together. The
electrons struggle to change direction in order to achieve a stable state in which the two electrons
again neutralize each other, giving off no magnetic field.
The theory is that these disturbed electron pairs are created in birds when they are exposed to
changes in light. The birds can sense the efforts of the electrons in trying to reach a condition of
stability because of the slight changes in the pull of the North and South Poles. In this way, the birds
can detect the direction of the poles while they are in flight.
In one experiment to confirm this effect, a group of European robins were tricked by artificial light to
believe that it was time for spring migration. The birds became eager to fly north. The changes in light
triggered the electron-pair movement described above exposing the robins to the magnetic field,
accompanying the electron pairs. The birds became disoriented and flew in all directions. The
simulated magnetic fields were much too weak to be detected by the birds' natural magnetite,
suggesting to the experimenters that the electron pairs, not the magnetite, were responsible for the
birds' confused flying.
The current view, therefore, is that light plays an important role in guiding bird migration. This may be
why birds turn their heads from side to side before flying off. Their eyes are collecting the surrounding
light, which in turn allows them to process and analyze the existing magnetic fields and to keep
themselves pointed in the right direction.
1. According to paragraph 1, insects influence bird migration in which of the following ways?
A. Insects generate a magnetic field that birds can detect.
B. Insects provide a food supply that exists only in warm climates.
C. Birds follow the paths taken by flying insects.
D. Birds know when to migrate by a sudden increase in insect population.
2. The word some in the passage refers to
A. insects B. recent experiments C. bird experts D. birds
3. According to paragraph 3, birds can detect the magnetic fields of the North and South Poles
because
A. they sense the motion of electron pairs
B. they can locate the poles by following landmarks
C. they ingest metal particles that are attracted by the poles
D. they have magnetite in their brains
4. The word embedded in the passage is closest in meaning to
A. implanted B. attached C. attracted D. activated
5. The author discusses homing pigeons in paragraph 4 in order to
A. provide an example of how humans can train birds
B. describe an experiment showing the importance of magnetite
C. show that homing pigeons return home by following landmarks
D. report homing pigeons’ behavior inside a cage
6. According to the passage, all of the following are theories about how birds navigate EXCEPT:
A. They follow landmarks like rivers and mountains.
B. They are guided by their position relative to the stars.
C. They feel vibrations in nerve endings in their brains.
D. They respond to changes in light
7. According to paragraph 4, the pigeons moved to the opposite end of a cage because
A. the magnetic field was normal
B. the magnetic field was stronger at one end
C. the magnetic field changed its polarity
D. the magnetic field was removed
8. The word altered in the passage is closest in meaning to
A. reversed B.canceled C. strengthened D. detected
9. Which of the following can be inferred about an electron pair in two north ends of magnets?
A. The two electrons spin in opposite directions.
B. One electron will move to the south end
C. One electron will be captured by the nucleus.
D. The two electrons spin in the same direction.
10. Which of the following best expresses the essential information in the highlighted sentence?
Incorrect answer choices change the meaning in important ways or leave out essential information.
A. The bird’s failure to detect the magnetic fields led researchers to conclude that the electron pairs
caused the birds' confusion.
B. The birds’ failure to detect the electron pairs showed that their magnetite was the cause of their
disorientation.
C. Experimenters found that the electron pairs were stronger than the birds’ magnetite and helped
them find their destinations.
D. Magnetic fields that are triggered by artificial light are detected by the birds’ magnetite causing
them to fly in the right direction
Your answers:

1. 2. 3. 4. 5.

6. 7. 8. 9. 10.

Part 4. Read the following text and do the tasks that follow.
Playing psychological games
A.
‘Psychological games’ is an approach to relationships that developed around the 1960s. It is a way of
looking at the interaction between people – identifying what seem to be fixed scripts in a seemingly
spontaneous conversation. One person says something which seems to elicit a certain type of
response from the other person, and the response seems to demand yet another particular response
from the first person. And on it goes, as if the two people were following a script that someone had
written.
B
Games fall into a number of categories, ranging from the harmless to the destructive. Some harmless
games are even essential to social interaction - such as the 'Greeting Game' (‘Hello, how are you?’ “I'm
fine, how are you?”) and the “Thanks Game” (“Thank you for inviting me. I had a great time.”). Game
playing is expected in some situations. Everyone involved knows that it is a game, and what is
expected. A sales person plays a game of pleasing the prospective customer. Children play games with
parents. In cases like these, the game player creates an impression, saying things which are not sincere
but are ways and means of getting what they want. Other games, however, keep a relationship from
developing to a more real and important level. Still others can actually be destructive, as they are
played by people with deeper psychological needs and motivations for power, control or
manipulation.
C
Some people set out to manipulate others for their own reasons. But others may not realize that they
are being manipulative. They are acting rather from an emotional script. Like a child that wants
something, and does all sorts of things to get it, some game players act from their own internal
desires, not realizing the effect their words and actions have on others.
D
A number of potentially damaging games have been identified. In the ‘Corner Game’ the manipulator
backs the other person into a corner – places them in a situation where anything they do is wrong. A
parent complains that their son or daughter’s room is never clean. Yet when the child tidies the room,
the parent says, “Why did it take you so long?” or “You haven’t tidied up inside the cupboard.” The
‘It’s Your Decision Game’ is played by people who want to escape the responsibility of making a
decision: “I don’t mind. You decide.” Although actually very much concerned about the outcome of
the decision, by insisting they are not the game player forces the other person to take all responsibility
for the consequences of the decision.
E
Games may indicate a lack of confidence in the other person, an unwillingness to communicate with
them directly. In the most innocent cases, they are played in an attempt at politeness, or genuine
concern for the other's feelings (trying not to hurt them). However, even these well-intentioned
games don't always have a good end. They can make it impossible for an atmosphere of trust to be
created.
F
At their worst, games are a way for an individual to retain power in a relationship, because their own
personal feelings are not revealed. The person who uses games to their own advantage needs to win a
game in order to have a sense of self-esteem - by harming someone else's self-confidence.
Manipulators range from Dictator (who always has to be in charge) to Nice Guy (who exaggerates care
and love for others, in order to get what he or she wants), to Protector (who is over- supportive or
over-protective)
G
Some game players have so many psychological needs that fulfilling their desires overshadows
everything else in a relationship. For example, a person who needs to be the centre of attention may
play games in which they consistently take the role of someone who needs help, someone who is
dependent Sometimes people fall into games in a relationship because of the roles that they think
they should be playing. A young couple that accepts the traditional roles for men and women may
assume that the husband needs to defend his wife against criticism by his family, or that he will
automatically make the decisions about minor repairs on her car, even though in both cases the wife is
perfectly capable of looking after herself. Their exchange will fall into a kind of game, because they
have restricted themselves by their concept of the roles that they should play.
Task 1. The Reading Passage above has seven paragraphs A-G. From the list of headings below,
choose the most suitable heading for each paragraph. Write the appropriate numbers (i-x) in boxes
1-6. Paragraph A has been done for you.

LIST OF HEADINGS
i. Towards a classification of games
ii. How the theory of games was developed
iii. Feeling good by making others have doubts about themselves
iv. Being cautious towards other people
v. Games that create permanent relationships between people
vi. Game-playing - conscious or unconscious
vii. How a relationship can be dominated by games
viii. The type of people that game players look for
ix. Some examples of harmful games
x. A tool for understanding communication

Example:
0. Paragraph A ____ x________
1. Paragraph B ______________ 2. Paragraph C _____________
3. Paragraph D ______________ 4. Paragraph E _____________
5. Paragraph F ______________ 6. Paragraph G _____________

Task 2: Do the following statements reflect the claims of the writer in the reading passage? Write
YES if the statement reflects the claims of the writer
NO if the statement contradicts the claims of the writer
NOT GIVEN if it is impossible to say what the writer thinks about this
8. When people want a certain decision they will always make it themselves.
9. Games that show regard for other people can prevent trust from developing in the relationship.
10. Giving another person too much help may be a form of manipulation.
11. Avoiding social pressure to behave in certain ways is difficult for a young married couple.
Your answers:

1. 2. 3. 4. 5. 6.

7. 8. 9. 10.

D. WRITING
Part 1. Rewrite each sentence using the word in brackets so that the meaning stays the same. You
must use between TWO and SIX words, including the word given.
1. When they started their trek, they had no idea how bad the weather would become. (OUTSET)
à Nobody realized how bad the weather would become.
2. The footballer injured his knee, so that was the end of his hopes of a first team place. (PAID)
à The footballer’s knee his hopes of a first team place.
3. I inherited this clock from my father and it belonged to his grandfather before that. (DOWN)
à This clock grandfather to my father and, in turn, to me.
4. It has been difficult for the children to accept their parents’ separation. (COME)
à The children are finding with their parents’ separation.
5. If Marc hadn’t taken up politics, he might have become a famous art historian. (NAME)
à If Marc hadn’t taken up politics, he might have himself as an art historian.
Part 2. Finish each of the following sentences in such a way that it means exactly the same as the
sentence printed before it.
1. Many creatures still survive and thrive in the harsh conditions of the deserts.
àHarsh ___________________________________________________________________.
2. She is prohibited from importing animal products for fear of spreading infectious diseases.
àLest .
3. We had to go home early from our holiday because of a strike threat from airport workers.
à We had to cut .
4. The thought passed through his mind and the decision was taken a moment later.
à The thought had no .
5. Nowadays I consider taking up a hobby to be far less important than I used to.
à Nowadays I don’t attach nearly .
TEST 9
1. LISTENING
I. Read the sentences below. You are going to hear two women talking about a holiday in France.
Read the sentences, and choose the best option: A, B, or C, to complete the statements about the
recording.

1. Paula's friend says that


A. she has been ill.
B. Paula doesn't look very well.
C. she's pleased to see Paula.

2. Before the trip, Paula


A. was enthusiastic about It.
B. wanted to go to the Lake District.
C. didn't tell anybody she was going.

3. Before Mark and Paula went to Paris,


A. Mark's boss didn't want him to go.
B. Paula arranged for somebody to look after the hamster.
C. Paula's sister promised to look after the children.

4. The journey across the Channel


A. was very smooth.
B. was unpleasant for Paula.
C. lasted eight hours.

5. The return trip from Paris was


A. disturbed by a flood.
B. an enjoyable experience.
C. earlier than planned.

II. Read the statements and decide whether they are true (T) or false (F).
1. The speaker has come from the Theosophical Society.
2. One of the main points of the talk is to save money.
3. She thinks students should do more housework.
4. She argues that plastic containers won't biodegrade quickly.
5. She warns that asthma sufferers should be careful with her recipes.
III. Part 1
You are going to hear two separate recordings about the uses of plastics.
Question 1-5
Complete the sentences below. Write no more than three words or number for each space
1. According to the speaker, the three factors contributing to the success of plastics today are
safety, __________ and ______________.
2. _______________ and other food products are wrapped in plastic to protect them from
contamination.
3. The use of plastics in major appliances ensures that they will _________________ corrosion.
4. Experts have calculated that a kilo of plastic packaging can lead to a reduction of ___________
in wasted food.
5. Plastic parts have made air conditioners as much as ___________________ more efficient since
the 1970s.
Question 6-10
Complete the notes. Write no more than three words of numbers in each space.
A history of plastic bag

1957 introduction of (6) _______________

1966 Around one third of packaging in (7) ___________


consists of plastic bags

1969 ‘New York City Experiment’:


(8) __________________ collected in plastic bags

(9) _______________ Plastic grocery bags first manufactured commercially

2002 New Irish tax of (10) ______________ per carrier bag

II. LEXICO-GRAMMAR
Exercise 1: Choose the best answer to complete each sentence.
1. I forgot my glasses so I'm as blind as a(n)_______.
A. bat B. owl C. racoon D. beaver
2. When a major disaster _________, the local Red Cross society coordinates the response.
A. hits B. strikes C. rises D. arrives
3. The university should not lay ______ its employees on the grounds of financial stringency.
A. out B. down C. away D. off
4. Graduates are expected to _____ a contribution to our society to promote democracy.
A. make B. take C. give D. hand
5. ______ the operation costs were rising so fast, the UN decided to reduce the personnel
by half.
A. Because B. If C. As if D. As a result
6. Their jealousy led them to commit deeds that they would come to regret _________.
A. painfully B. harshly C. heavily D. deeply
7. Jane was born with a ________ spoon in her mouth.
A. gold B. silver C. diamond D. bronze
8. We used to share a room at college, but we ______ apart over the years.
A. ran B. broke C. drifted D. distanced
9. It costs about thirty dollars to have a tooth ______.
A. filling B. to fill C. filled D. fill
10. Technical knowledge is of ______ importance for technical translation.
A. lively B. vibrant C. essential D. vital
11. The team turned ______ trumps in the final game and won the championship.
A. up B. out C. above D. over
12. I’d say let’s meet on Saturday but I’m none _____ sure what’s happening at the
weekend.
A. so B. very C. that D. too
13. We’ve seen it effects in many experiments, but there’s still a ________ chance it’s just
an illusion.
A. thin B. slim C. narrow D. restricted
14. The purpose of the survey was to ________ the inspectors with local conditions.
A. inform B. acquaint C. instruct D. notify
15. If you’d like to take a seat in the waiting room till the doctor can see you, you’ll find
plenty of magazines to browse _______.
A. over B. through C. in D. round
16. Peregrine _______ the rewards of his hard work when he was given the chair of classics.
A. grasped B. got C. reaped D. collected
17. The film traces the _______ of fascism in Europe.
A. debut B. rise C. introduction D. invention
18. She thinks that you started that rumor about her - that’s why she’s been giving you the
cold _______ all day.
A. shoulder B. look C. hand D. back
19. These measures have been taken _______ increasing the company’s profits.
A. Thanks to B. with a view to C. with a reason for D. for fear of
20. When we _______, we totally stop acting and moving.
A. fall down B. die out C. pass away D. go down
Exercise 2: The passage below contains 10 mistakes. Underline the mistakes and write their correct
forms in the space provided in the column on the right. (0) has been done as an example.

One rain day in 2012, I ducked into my local library of shelter and, for 1. rain => rainy
some reason, started looking at wills. I found one written by a woman
2. ……………
from Derby who had disguised her as a man and gone off to fight –
and die – in the first world war. She had written her will phonetic, 3. ……………
which made it easier for me to read as I’m dyslexic. That was one
4. ……………
reason why I connected with it. The other was the phrase: “Blood
swept lands and seas of red where angels fear to tread.” It leapt out at 5. ……………
me, made me to think about the war’s death toll. With a bit of 6. ……………
research, I discovered that there had been 888,246 British and
Colonial military fatality. 7. ……………

I decided to represent that deaths through ceramic flowers, anything I 8. ……………


had first started making at university. Ceramics are transient and 9. ……………
fragile, like we are. They feel part of our very humane – societies have
always been carbon-dated by their ceramics and pottery. I considered 10. ……………
making roses, which symbolise sacrifice in Victorian times, but I 11. …………..
settled on the poppy because of its links to war and remembrance.

Exercise 3: Write the correct FORM of each bracketed word in the numbered space provided in the
column on the right.

Putting some fun into the workplace MAJOR


In a study of 737 chief executives working in large corporations, the vast PREFER
(1)___________ gave the same answer when asked what kind of person
they like to employ. Ninety-eight per cent said they would hire someone LOYAL
with a cheerful attitude, (2)___________ with a good sense of humour.
SIGNIFY
Having fun at work apparently inspires (3)___________ in employees. A
LONG
survey of 1,000 workers showed how (4)___________ their manager’s
sense of humour or lack of it was to the (5)___________ of time they LIKELY
stayed in a job. If they worked for a boss whose sense of humour they
BENEFIT
described as ‘below average’, the employee’s (6)___________ of staying
dropped to seventy-seven per cent compared to ninety per cent for a NECESSARY
boss who had an ‘above average’ sense of humour. CONTINUE
Laughter may be both (7)___________ and good for business but it isn’t SATISFY
(8)___________ a positive aspect of all jobs. Some people working in
retail jobs are required to smile (9)___________. Such enforced
happiness can cause (10)___________ at work and also result in
emotional stress.

III. READING
Exercise 1: Read the following passage and decide which answer (A, B, C, or D) best fits each gap.
Write your answer in the numbered box.
Frescoes painting is the age-old technique which 1________ painting on a damp plaster
walls. It is known to have been used to decorate homes for at least 5000 years and has
probably existed in one form or another for 2________ longer. Its very nature makes it
3_________ for adorning large surfaces, which explains its 4_________in churches and
government buildings. 5___________ oil paintings, which are glossy and reflect light,
making it difficult to view them from certain angles, frescoes have a pleasing matte finish.
As fresco plaster bleaches relatively easily, artists do not use as wide a range of pigments as
in other types of painting, sticking mainly to pale earth 6_______.
Fresco painting is considered to have reached its 7__________ in Italy during the
Renaissance, with Michelangelo being perhaps the most famous artist to 8__________ this
technique. A great many churches in Europe 9__________ wonderful frescoes, while more
recently celebrated artists such as the renowned Mexican muralist Diego Rivera have used
the technique to great 10_________.
1. A. composes B. comprises C. involves D. consists
2. A. greatly B. noticeably C. perceptibly D. considerably
3. A. ideal B. pertinent C. relevant D. absolute
4. A. ubiquity B. universality C. preoccupation D. all-presence
5. A. On the contrary B. Unlike C. Conversely D. In reverse
6. A. tones B. tints C. shades D. hues
7. A. summit B. crest C. height D. top
8. A. operate B. enlist C. engage D. employ
9. A. demonstrate B. boost C. expose D. boast
10. A. result B. success C. effect D. reaction
Exercise 2: Read the following passage and fill in each numbered blank with ONE suitable word.
Write your answer in the numbered box.
Over the last century the world has become increasingly smaller. Not geographically, of
course, but in the sense that media, technology and the opening of borders has enabled the
world’s citizens to view, share and gain 1_________ to a much wider range of cultures,
societies and world views. In this 2________ pot that the world has become, today’s child
is privy 3_________ facets of the human experience that his immediate predecessors had
no inkling even existed. It 4____________ to reason that in order to absorb, configure and
finally form opinions about this information-laden planet, children must be supplied with
certain tools. 5____________ in this list of ‘tools’ are: education, social skills, cultural
awareness and the acquisition of languages, the most important of these being the latter.
Until recently, a child who had the ability to speak more than one language would have
been considered a very rare entity. 6___________ one-language phenomenon could be
attributed to a combination of factors. One of them is that the monolingual environment in
which a child was raised played a strong role, 7___________ did the limited, biased
education of the past. With regards to immigrants, the sad fact was that non-native parents
tended to withhold the teaching of the mother tongue so that the child would acquire the
more ‘prestigious’ language of the adopted country. Nowadays, the situation has
8___________ an almost complete reversal. In the majority of North American and
European countries, most children are given the opportunity to learn a second or
9__________ a third language. Children acquire these foreign languages through various
and diverse means. In many countries, learning a foreign language is a compulsory subject
in the state school curriculum. Other children rely on language schools or private tuition to
achieve their goal. In other instances, children are 10_________ to bilingual parents, who, if
they so desire, may teach the children two languages.
Exercise 3: Read the following passage and decide which option (A, B, C, or D) best answers each
question. Write your answer in the numbered box.
Deep in the Sierra Nevada, the famous General Grant giant sequoia tree is suffering its loss of stature
in silence. What once was the world's No. 2 biggest tree has been supplanted thanks to the most
comprehensive measurements taken of the largest living things on Earth.
The new No. 2 is The President, a 54,000-cubic-foot gargantuan not far from the Grant in Sequoia
National Park. After 3,240 years, the giant sequoia still is growing wider at a consistent rate, which
may be what most surprised the scientists examining how the sequoias and coastal redwoods will be
affected by climate change and whether these trees have a role to play in combating it.
"I consider it to be the greatest tree in all of the mountains of the world," said Stephen Sillett, a
redwood researcher whose team from Humboldt State University is seeking to mathematically assess
the potential of California's iconic trees to absorb planet-warming carbon dioxide.
The researchers are a part of the 10-year Redwoods and Climate Change Initiative funded by the Save
the Redwoods League in San Francisco. The measurements of The President, reported in the current
National Geographic, dispelled the previous notion that the big trees grow more slowly in old age.
It means, the experts say, the amount of carbon dioxide they absorb during photosynthesis continues
to increase over their lifetimes.
In addition to painstaking measurements of every branch and twig, the team took 15 half-centimeter-
wide core samples of The President to determine its growth rate, which they learned was stunted in
the abnormally cold year of 1580 when temperatures in the Sierra hovered near freezing even in the
summer and the trees remained dormant.
But that was an anomaly, Sillett said. The President adds about one cubic meter of wood a year during
its short six-month growing season, making it one of the fastest-growing trees in the world. Its 2 billion
leaves are thought to be the most of any tree on the planet, which would also make it one of the most
efficient at transforming carbon dioxide into nourishing sugars during photosynthesis.
"We're not going to save the world with any one strategy, but part of the value of these great trees is
this contribution and we're trying to get a handle on the math behind that," Sillett said.
After the equivalent of 32 working days dangling from ropes in The President, Sillett's team is closer to
having a mathematical equation to determine its carbon conversion potential, as it has done with
some less famous coastal redwoods. The team has analyzed a representative sample that can be used
to model the capacity of the state's signature trees.
More immediately, however, the new measurements could lead to a changing of the guard in the land
of giant sequoias. The park would have to update signs and brochures - and someone is going to have
to correct the Wikipedia entry for "List of largest giant sequoias," which still has The President at No.
3.
Now at 93 feet in circumference and with 45,000 cubic feet of trunk volume and another 9,000 cubic
feet in its branches, the tree named for President Warren G. Harding is about 15 percent larger than
Grant, also known as America's Christmas Tree. Sliced into one-foot by one-foot cubes, The President
would cover a football field.
Giant sequoias grow so big and for so long because their wood is resistant to the pests and disease
that dwarf the lifespan of other trees, and their thick bark makes them impervious to fast-moving fire.
It's that resiliency that makes sequoias and their taller coastal redwood cousin worthy of intensive
protections and even candidates for cultivation to pull carbon from an increasingly warming
atmosphere, Sillett said. Unlike white firs, which easily die and decay to send decomposing carbon
back into the air, rot-resistant redwoods stay solid for hundreds of years after they fall.
Though sequoias are native to California, early settlers traveled with seedlings back to the British Isles
and New Zealand, where a 15-foot diameter sequoia that is the world's biggest planted tree took root
in 1850. Part of Sillett's studies involves modeling the potential growth rate of cultivated sequoia
forests to determine over time how much carbon sequestering might increase.
All of that led him to a spot 7,000 feet high in the Sierra and to The President, which he calls "the
ultimate example of a giant sequoia." Compared to the other giants whose silhouettes are bedraggled
by lightning strikes, The President's crown is large with burly branches that are themselves as large as
tree trunks.
The world's biggest tree is still the nearby General Sherman with about 2,000 cubic feet more volume
than the President, but to Sillett it's not a contest.
"They're all superlative in their own way," Sillett said.

1. The word "supplanted" in paragraph 1


A. means inquisitive
B. Has a double-meaning both as a pun on the topic of plants and a literal meaning of "to replace"
C. Is a synonym for "to plant again"
D. Has the same meaning as "to plant," with extra emphasis
2. One common myth about trees that The President helps disprove is
A. that giant sequoias are more resilient than other tree species
B. that old trees are as productive at photosynthesis as younger ones
C. that only giant sequoias may be named after historical figures
D. that large trees grow more slowly as they age
3. What is the primary benefit that Sillett and other researchers suggest that giant sequoias may have?
A. Their natural beauty can have health benefits for those who travel to wildlife preserves to see them
B. They represent centuries of natural history that no other living things do
C. Because of their size, they are able to process more carbon dioxide than other trees, which can
have significant benefits for the atmosphere
D. Their resilient bark may have eventual uses in human medicine.
4. The giant sequoias are compared to white firs to demonstrate that?
A. Even when the sequoias fall, they do not decay and so send less carbon into the air
B. White firs are more plentiful because they grow and decay more quickly than sequoias
C. The giant sequoias are completely resistant to death
D. White firs are essential because when they decompose they emit necessary nutrients
5. The President has grown every year EXCEPT
A. 1850
B. 2012
C.1580
D. The President has grown every year of its life
6. All of the following contribute to the lifespan of the giant sequoia EXCEPT
A. They are resistant to diseases that can affect other tree species
B. Their size makes them less vulnerable to animal attacks
C. They are resistant to pests that commonly inhabit trees
D. Their thick bark protects them from wildfires.
7. The term "changing of the guard" in Paragraph 10 means
A. The size rankings of various large sequoias is being reevaluated
B. Human security will be employed to protect these valuable trees
C. Wildlife parks will bring in new equipment to ensure the safety of the trees
D. A new schedule of shifts will be made for studying the trees
8. What does the term "cultivated sequoia forests" in Paragraph 14 imply?
A. Current sequoia reserves will be altered to grow in particular patterns
B. That sequoias may be specially grown in the future for the sole purpose of filtering carbon from the
air
C. New forests may be grown globally to promote the beauty of the species
D. Wildlife parks will make more of an effort in the future to direct visitors to the sequoia forests
9. Giant sequoias are native to California, but can also be found in
A. New Zealand B. France C. South America D. Australia
10. In the final sentence, the word "superlative" is closest in meaning to
A. Best of a species B. Most beautiful
C. The winner of a contest D. Having individual, unique merit
Exercise 4: Read the following passage and do the tasks that follow.
TOURISM
A Tourism, holidaymaking and travel are these days more significant social phenomena than most
commentators have considered. On the face of it there could not be a more trivial subject for a book.
And indeed since social scientists have had considerable difficulty explaining weightier topics, such as
work or politics, it might be thought that they would have great difficulties in accounting for more
trivial phenomena such as holidaymaking. However, there are interesting parallels with the study of
deviance. This involves the investigation of bizarre and idiosyncratic social practices which happen to
be defined as deviant in some societies but not necessarily in others. The assumption is that the
investigation of deviance can reveal interesting and significant aspects of normal societies. It could be
said that a similar analysis can be applied to tourism.
B Tourism is a leisure activity which presupposes its opposite, namely regulated and organised work. It
is one manifestation of how work and leisure are organised as separate and regulated spheres of
social practice in modern societies. Indeed acting as a tourist is one of the defining characteristics of
being ‘modern’ and the popular concept of tourism is that it is organised within particular places and
occurs for regularised periods of time. Tourist relationships arise from a movement of people to, and
their stay in, various destinations. This necessarily involves some movement, that is the journey, and a
period of stay in a new place or places. ‘The journey and the stay’ are by definition outside the normal
places of residence and work and are of a short term and temporary nature and there is a clear
intention to return ‘home’ within a relatively short period of time.
C A substantial proportion of the population of modern societies engages in such tourist practices new
socialised forms of provision have developed in order to cope with the mass character of the gazes of
tourists as opposed to the individual character of travel. Places are chosen to be visited and be gazed
upon because there is an anticipation especially through daydreaming and fantasy of intense
pleasures, either on a different scale or involving different senses from those customarily
encountered. Such anticipation is constructed and sustained through a variety of non-tourist practices
such as films, TV literature, magazines records and videos which construct and reinforce this
daydreaming.
D Tourists tend to visit features of landscape and townscape which separate them off from everyday
experience. Such aspects are viewed because they are taken to be in some sense out of the ordinary.
The viewing of these tourist sights often involves different forms of social patterning with a much
greater sensitivity to visual elements of landscape or townscape than is normally found in everyday
life. People linger over these sights in a way that they would not normally do in their home
environment and the vision is objectified or captured through photographs postcards films and so on
which enable the memory to be endlessly reproduced and recaptured.
E One of the earliest dissertations on the subject of tourism is Boorstins analysis of the pseudo event
(1964) where he argues that contemporary. Americans cannot experience reality directly but thrive on
pseudo events. Isolated from the host environment and the local people the mass tourist travels in
guided groups and finds pleasure in inauthentic contrived attractions gullibly enjoying the pseudo
events and disregarding the real world outside. Over time the images generated of different tourist
sights come to constitute a closed self-perpetuating system of illusions which provide the tourist with
the basis for selecting and evaluating potential places to visit. Such visits are made says Boorstin,
within the environmental bubble of the familiar American style hotel which insulates the tourist from
the strangeness of the host environment.
F To service the burgeoning tourist industry, an array of professionals has developed who attempt to
reproduce ever-new objects for the tourist to look at. These objects or places are located in a complex
and changing hierarchy. This depends upon the interplay between, on the one hand, competition
between interests involved in the provision of such objects and, on the other hand changing class,
gender, and generational distinctions of taste within the potential population of visitors. It has been
said that to be a tourist is one of the characteristics of the modern experience. Not to go away is like
not possessing a car or a nice house. Travel is a marker of status in modern societies and is also
thought to be necessary for good health. The role of the professional, therefore, is to cater for the
needs and tastes of the tourists in accordance with their class and overall expectations.
Questions 1-5
Choose the most suitable heading for each paragraph from the list of headings below Write the
appropriate numbers (i-ix) in boxes 1-5 on your answer sheet.
Paragraph D has been done for you as an example.
NB. There are more headings than paragraphs so you will not use all of them You may use any heading
more than once.

List of Headings
i The politics of tourism
ii The cost of tourism
iii Justifying the study of tourism
iv Tourism contrasted with travel
v The essence of modern tourism
vi Tourism versus leisure
vii The artificiality of modern tourism
viii The role of modern tour guides
ix Creating an alternative to the everyday experience

1. Paragraph A
2. Paragraph B
3. Paragraph C

Example Answer
Paragraph D ix

4. Paragraph E
5. Paragraph F
Questions 6-10
Do the following statements agree with the views of the writer in the reading passage? In boxes 6-10
write :
YES if the statement agrees with the writer
NO if the statement contradicts the writer
NOT GIVEN if it is impossible to say what the writer thinks about this
6. Tourism is a trivial subject.
7. An analysis of deviance can act as a model for the analysis of tourism.
8. Tourists usually choose to travel overseas.
9. Tourists focus more on places they visit than those at home.
10. Tour operators try to cheat tourists.
IV. WRITING
Exercise 1: Rewrite the sentences so that they have the same meaning as the ones before them.
1. We get on very well with our next door neighbor.
=> We are on ______________________________.
2. The last time we met each other was a fortnight ago.
=> It has been two ______________________________.
That makes me think of something that happened to me.
=> That brings ______________________________.
4. Apart from Philip, everyone else at the meeting was a Party member.
=> With the ____________________________, everyone else at the meeting was a Party member.
5. His arrival was completely unexpected.
=> His arrival took ______________________________.
Exercise 2: Rewrite the sentences using the words given so that they have the same meaning as the
ones before them. Do not change the words given.
1. Bill reckoned that his success was due to incredible luck. PUT
=> Bill ______________________________.
2. They’re faced with the choice of two alternatives. HORNS
=> They’re ______________________________.
3. I suddenly realized the meaning of a “freebie”. DAWNED
=> It ______________________________.
4. I feel certain that there will be a new government after the election. BOUND
=> In my opinion, there is ______________________________.
5. I didn’t realize how much she was influenced by her mother. EXTENT
=> I didn’t realize the ______________________________.
TEST 10
A. LISTENING (50 points):
HƯỚNG DẪN PHẦN THI NGHE HIỂU
 Bài nghe gồm 4 phần, mỗi phần được nghe 2 lần, thí sinh có 30-40 giây giữa mỗi phần nghe
để đọc bài.
 Mọi hướng dẫn cho thí sinh ( bằng tiếng Anh) đã có trong bài nghe.
Part 1. Listen and complete the notes below. Write NO MORE THAN TWO WORDS OR A NUMBER
for each answer. Write your answers in the corresponding numbered boxes. (10 points) (IELTS
PRACTICE EXAM- PRACTICE TEST 4)

Example:

Order taken by: Ms. Jones


Names: Harold (1) _________________

Address 58 Fulton Avenue, apartment 12

Type of service (2) ___________________

Employer Wrightsville Medical Group

Occupation (3)____________________

Work phone (4) ___________________

Time at current job 9 years

Special service (5) ________________ Internet.

Installation scheduled for Day: Friday - Time of the day: Morning

Your answers:

1. 2. 3. 4. 5.

Part 2. You are going to listen to a radio interview the Channel Tunnel and choose the correct
answer A, B, C or D for each question. Write your answers in the corresponding numbered boxes.
(10 points)
1. Why did Mrs Jamieson first start campaigning against the tunnel ?
1. She realized at the start that the tunnel was unsafe.
2. She knew the tunnel would disrupt the village life
3. She thinks it is important to fight for what is right.
4. The construction work was interfering with normal life.
2. Why was it difficult to choose a suitable route for the tunnel limk?
1. The first route selected was considered too unsafe.
2. Most possible routes were inconvenient to passengers
3. Much of the south-east is covered by dense forest.
4. All possible routes passed through residential areas
3. Why are there still problems with the tunnel, according to Mrs Jamieson?
1. The construction work was never properly finished.
2. The construction company became short of money.
3. The normal safety checks were never carried out.
4. The operators did not spend enough money on the tunnel.
4. What does Mr Ashton say about the problems reported in newspapers?
1. They were problems that have already been solve
2. Those responsible for the problems have been dismissed.
3. The reports do not affect his confidence in the tunnel.
4. The reports were untrue and designed to scare people.
5. Why does Mr Ashton find the idea of a rabies epidemic in Brita in “silly”?
1. It would be impossible for a rabid animal to enter the tunnel.
2. It is unlikely that a rabid animal could get to Britain via the tunnel.
3. No rabid animals have ever crossed the River Seine in France.
4. It would be impossible for one animal to cause a epidemic.
Your answers:

1. 2. 3. 4. 5.

Part 3. You will hear a radio interview with Ryan Patterson, the inventor of a new device. Decide
whether the statements are true (T) or false (F). Write your answers in the corresponding numbered
boxes. (10 points) (IELTS MASTERLCLASS)

1. The idea for the invention occurred to Ryan while waiting at a Burger King restaurant.
2. A cell phone is used as the reciever when using the Sign Language Translator.
3. The invention brought Ryan money to cover the costs of his further education.
4. Ryan had no previous experience of building electronic devices
5. Ryan has sold this invention to a deaf community centre.
Your answers:

1. 2. 3. 4. 5.

Part 4: You will hear a part of an interview with a man called Ewan Richardson , who is trying to
persuade people to use less paper. For questions 1-10, complete the sentences with NO MORE THAN
THREE WORDS for each gap. Write your answers in the corresponding numbered boxes. (20 points)
(FCE TRAINER)

1. Every year, the average UK citizen uses about ________ of paper.


2. Most of the world’s paper comes from very________forests.
3. The production of paper causes terrible ________ in some places.
4. The destruction of the forests is a much bigger cause of global warming than ________
5. Ewan says that there are already paper recycling bins in many ________
6. You can use less paper by avoiding unnecessary ________ when you are studying or working.
7. You can often reuse________ that you have received.
8. To receive less junk mail, don’t ask for________when you buy something.
9. Stop receiving any magazines you don’t always read, or________ them with others.
10. Most________published in Britain are now printed on recycled paper.
Your answers:

1. 3. 5. 7. 9.

2. 4. 6. 8. 10.

B. LEXICO - GRAMMAR (50 points)


Part 1. Choose one of the words marked A, B, C, or D which best completes each of the following
sentences. Write your answers in the corresponding numbered boxes. (20 points)
1. Choose whatever you like from the menu. It’s your birthday so money is no ______.
A. brain B. barrier C. factor D. object
2. Students should be encouraged to _______ critically the information that they are given.
A. debate B. discuss C. comment D. argue
3. When I joined the army, I found it difficult to ________ orders from my superiors, but I
soon got used to it.
A. finish off B. wrap up C. carry out D. follow through
4. On Christmas Eve, the _________ family gathers for dinner, usually at my grandmother’s
house.
A. mere B. entire C. total D. complete
5. ____________, helped shape the history of American industry.
A. Joseph Wharton, along with his many successful business ventures, has
B. Joseph Wharton, including his many successful business ventures, have
C. Joseph Wharton, along with his many successful business ventures, have
D. Joseph Wharton and his many successful business ventures has
6. I’d like to _______ the crossword puzzle in the newspapers everyday.
A. answer B. fill C. do D. make
7. Her living room, with its delicate furniture and knickknacks, made him feel like a bull in a
__________ shop.
A. China B. French C. Swedish D. German
8. The thing I don’t like about my present job is that we often have to work up to ten hours
________ a stretch.
A. to B. at C. over D. through
9. Cyclone Althea devastated portions of northern Australia in December 1971; the strong
winds ripped roofs off houses, hail damaged vehicles and windows, and __________ cost
the Australian federal government close to a billion dollars in modern-day adjusted values.
A. it B. they C. those D. the storm
10. Some of the members were so _______ by the way Jack opened the meeting that they
walked right out.
A. joyous B. incensed C. unaffected D . tired
11. He pressed on, ______, planning the largest electric furnace in the world.
A. thinking better of it B. thinking out loud C. thinking back D. thinking big
12. Mad Max was a low-budget film. It received a lot of good reviews ________.
A. however B. although C. whatever D. though
13. After receiving the result of the latest exam, Sally was in _______ .
A. rigour B. anguish C. jeopardy D. offence
14. He will never ________ his pride and try to bring his ex back.
A. drop B. shed C. overlook D. swallow
15. Yesterday, I ________ my homeroom teacher when I was having coffee with Dan.
A. dropped in B. got on C. bumped into D. came over
16. Cooks should remember that some raw foods are very ________ and should be
refrigerated or chilled until ready to be eaten or cooked.
A. peripheral B. perishable C. periodic D. permanent
17. He had no ________ about lying to her mother despite her efforts to lift up his spirits .
A. qualms B. doubts C. queries D. thoughts
18. Due to the blatant plagiarism of his classmate, Tim is now emitting smoke from ______
orifices.
A. nine B. seven C. nil D. double
19. Would you mind if I _________ the door?
A. closed B. closing C. close D. am closing
20. I’ll _________ your suggestions and reply to you next week.
A. glance through B. mull over C. look up D. refer to
Your answers:

1. 2. 3. 4. 5.

6. 7. 8. 9. 10.

11. 12. 13. 14. 15.

16. 17. 18. 19. 20.

Part 2. Read the following text which contains 10 mistakes. Identify the mistakes and write the
correction in the corresponding numbered boxed (10 points)
While the internet opens up a whole new world of knowledge and information for this and future
generations to explore, it also poses a number of serious concerns as parents with young, net-savvied
children. For starters, it is exceptionally difficult to monitor your children's net activity and make
abreast of whom they are interacting with online. Secondly, there is little if any censorship of the
internet, so parents must be willing to do the censorship themselves or rely on software products to
do it for them. Even still, there are ways around the best-intentioned of such programmes, and,
besides, the alarming level of growth in cyber-bullying is indication of a trend parents should, perhaps,
be far more concerning about. It used to be that children were protecting from the bullies one they
returned to the safe confine of their home, having escaped their schoolyard tormentors, but not
anymore. There is nowhere to run thanks to social networks like Face book, which, if anything, make
the spreading far and away of malicious rumours and the like easier than ever before given the
virulence nature of the internet.
Your answers:

Line Mistakes Corrections

1.
2.

3.

4.

5.

6.

7.

8.

9.

10.

Part 3. Give the correct form of the words in brackets. Write your answers in the corresponding
numbered boxes. (10 points)
Meditation
People are often put off meditation by what they see as its many mystical associations. Yet
meditation is a (1) ______ (STRAIGHT) technique which merely involves sitting and resting the mind. In
addition to its (2) ______ (SIMPLE), meditation offers powerful help in the battle against stress.
Hundreds of studies have shown that meditation, when (3) ______ (TAKE) in a principled way, can
reduce hypertension which is related to stress in the body. Research has proved that certain types of
meditation can (4) ______ (SUBSTANCE) decrease key stress symptoms such as anxiety and (5) ______
(IRRITABLE). In fact, those who practice meditation with any (6) ______ (REGULAR) see their doctors
less and spend, on average, seventy percent fewer days in hospital. They are said to have more
stamina, a happier (7) ______ (DISPOSE) and even enjoy better relationships.
When you learn to meditate, your teacher will give you a personal ‘mantra’ or word which you
use every time you practice the technique and which is (8) ______ (SUPPOSE) chosen according to
your needs. Initial classes are taught individually but (9) ______ (SEQUENCE) classes usually consist of
a group of students and take place over a period of about four days. The aim is to learn how to slip
into a deeper state of (10) ______ (CONSCIOUS) for twenty minutes a day. The rewards speak for
themselves.
Your answers:

1. 2. 3. 4. 5.

6. 7. 8. 9. 10.

C. READING (50 points)


Part 1. Read the following passage and decide which answer (A, B, C, or D) best fits each gap. Write
your answers in corresponding numbered boxes. (10 pts)
TURN OFF THE HEATING
By 1_________ your hand into a freezing cold bucket of water, you can discover how quickly your
body burns calories. This basic experiment is the starting point for a pioneering study into the effects
of brown- or good - fat in the body.
Everyone is born with brown fat around the shoulder 2________. It is central to keeping a baby’s body
temperature on an even 3________ by using up this 4__________ of fat in order to keep babies
warm. Scientists, though, have long believed that this brown fat vanishes as babies grow out of
5___________ and it is no longer needed.
However, a few years ago, researchers were carrying out scans 6________ adults during the winter
and realized there were 7__________ of fat that seemed to have been 8________ by the cold
weather. This discovery has encouraged scientists to 9_________ further research in the hope it is the
10__________ gun that will help solve weight problems amongst the obese. They have already
determined that the cold, certain foods and exercise can activate this brown fat to people’s benefit.
1. A. prodding B. thrusting C. propelling D. heaving
2. A. blades B. joints C. bones D. sockets
3. A. path B. track C. course D. keel
4. A. store B. storation C. storage D. storing
5. A. development B. infancy C. adolescent D. childhood
6. A. over B. for C. on D. off
7. A. stretches B. areas C. tracts D. pockets
8. A. developed B. formed C. provoked D. triggered
9. A. make B. conduct C. perform D. absorb
10. A. grease B. zip C. smoking D. light
Your answers

1. 2. 3. 4. 5.

6. 7. 8. 9. 10.

Part 2. Read the text below and think of the word which best fits each space. Use only ONE word in
each space. Write your answers in the corresponding numbered boxes. (15 points)
A prodigy is defined as a person with a great natural ability which shows itself at an early age; they
may become expert musicians or be (1)______ of doing complex mathematics. Some youngsters
develop (2) _____ remarkable abilities that they attract the (3) _____ of the media, like Arran
Fernandez, a five-year old who became the youngest person to pass a GCSE, an exam for school
leavers. He had obviously benefited intellectually (4) ______ being taught at home by his parents,
who said that their son was still a happy and normal child (5) ______ never having been to any form of
school. Arran could add up at the age of two and a half, so he obviously had a natural gift, but without
a great deal of effort on the part of his parents, it’s unlikely that he would have applied (6)______ to
serious study. The role parents (7) ______ in such cases is highly controversial. Many people believe
that the more you push young children, the greater the chances are that the child will have social and
emotional problems (8) ______ in life. The story of another young person, Sufiah Yusof, who entered
university at the age of 13, is often quoted as proof of this. She (9) _____ out of her studies at Oxford
two years later, claiming that her parents attitude towards her constituted emotional abuse. It seems
that being a prodigy is (10) _______ substitute for a normal childhood.

Your answers

1. 2. 3. 4. 5.

6. 7. 8. 9. 10.

Part 3. Read the passage and choose the best option A, B, C, or D to answer the questions. Write
your answers in the corresponding numbered boxes. (10 points)
POPULATION ECOLOGY
Population ecology is the science that measures changes in population size and composition and
identifies the causes of these fluctuations. Population ecology is not concerned solely with the
human population. In ecological terms, a population consists of the individuals of one species that
simultaneously occupy the same general area, rely on the same resources, and are affected by similar
environmental factors. The characteristics of a population are shaped by its size and by the
interactions among individuals and between individuals and their environment.
Population size is a balance between factors that increase numbers and factors that decrease
numbers. Some factors that increase population are favorable light and temperature, adequate food
supply, suitable habitat ability to compete for resources, and ability to adapt to environmental change.
Factors that decrease populations are insufficient or excessive light and temperature, inadequate food
supply, unsuitable or destroyed habitat, too many competitors for resources, and inability to adapt to
environmental change.
Another important characteristics of any population is its density. Population density is the
number of individuals per unit, such as the number of maple trees per square kilometer in a country.
Ecologists can rarely determine population size by actually counting all individuals within geographical
boundaries. Instead, they often use a variety of sampling techniques to estimate densities and total
population sizes. For example, they might estimate the number of black bears in a national park by
counting individuals in a few sample plots representative of the whole park. In some cases, they
estimate population size through indirect indicators, such as the number of nests or burrows, or signs
such as tracks or droppings.
Another important population characteristics, dispersion, is the pattern of spacing among
individuals within the population’s geographical boundaries. Various species are distributed in their
habitats in different ways to take better advantage of food supplies and shelter, and to avoid predators
or find prey. Within a population’s range, densities may vary greatly because not all areas provide
equally suitable habitat, and also because individuals space themselves in relation to other members
of the population.
Three possible patterns of dispersion are clumped, uniform, and random. A clumped dispersion
pattern means that individuals are gathered in patches throughout their habitat. Clumping often
results from the irregular distribution of resources needed for survival and reproduction. For example,
fallen trees keep the forest floor moist, and many forest insects are clumped under logs where the
humidity is to their liking. Clumping may also be associated with mating, safety, or other social
behavior. Crane flies, for example, swarm in great numbers, a behavior that increases mating chances,
and some fish swim in large schools so they are less likely to be eaten by predators.
A uniform or evenly spaced distribution results from direct interactions among individuals in the
population. For example, regular spacing of plants may result from shading and competition for water.
In animal populations, uniform distribution is usually caused by competition for some resource or by
social interactions that set up individual territories for feeding, breeding, or resting.
Random spacing occurs in the absence of strong attraction or repulsion among individuals in a
population. Overall, random patterns are rare in nature, with most populations showing a tendency
toward either clumped or uniform distribution.
Population change in size, structure, and distribution as they respond to changes in
environmental conditions. Four main variable – births, deaths, immigration and emigration –
determine he rate of change in the size of the population overtime. A change in the birth rate or death
is the major way that most populations respond to changes in resource availability. Members of some
animal species can avoid or reduce the effects of environmental stress by immigrating to another with
more favourable environmental conditions, thus altering to population’s dispersion.
1. Which sentence below best expresses the essential information in the highlighted sentence in
par.1 ?
A. Any species of life can be studied in population ecology.
B. Population ecologists care about the future of humanity.
C. The growth of the human population is a major concern.
D. Population ecology does not consider humans worthy of study.
2. According to the passage, which factor might cause the population of a species to decrease in size?
A. A favorable amount of light and water
B. An ability to hide from or defend against predators
C. A large number of other species competing for food
D. A greater number of births than deaths
3. Which of the following is an indirect indicator of a population’s density?
A. The distribution of food in a given area
B. The number of nests in a given area
C. The number of births in a given period of time
D. The number of individuals counted in a given area
4. The distribution pattern of individuals within a population’s geographical boundaries is known as
______.
A. population ecology B. population density
C. population change D. population dispersion
5. The word “range” in paragraph 4 is closest in meaning to ______.
A. territory B. control C. history D. shelter
6. The word “patches” in paragraph 5 is closest in meaning to
A. dark places B. family groups C. warm spots D. small areas
7. The word “their” in paragraph 5 refers to ______.
A. resources B. trees C. insects D. logs
8. All of the following are given as reasons for clumping EXCEPT _____.
A. uneven resource distribution B. territorial disputes
C. mating behavior D. safety from predators
9. Which of the following situations would be most likely to result in a uniform dispersion pattern?
A. Birds compete for a place to build their nests.
B. Fish swim in large schools to avoid predators.
C. Whales develop strong bonds among relatives.
D. Elephants form a circle to protect their young.
10. Why does the author mention immigration and emigration in paragraph 8?
A. To identify factors affecting population dispersion
B. To give examples of territorial behavior in animals
C. To show that populations balance themselves over time
D. To explain why animal populations are uniformly dispersed
Your answers

1. 2. 3. 4. 5.

6. 7. 8. 9. 10.

Part 4. Read the text and do the following tasks. (15 points)
For questions 1-6, choose the correct heading for paragraphs A-G from the list of headings below.
Write your answers in the corresponding numbered boxes.

List of headings
i. Why some early social science methods lost popularity
ii. The cost implications of research
iii. Looking ahead to an unbiased assessment of research
iv. A range of social issues that have been usefully studied
v. An example of a poor decision that was made too quickly
vi. What happens when the figures are wrong
vii. One area of research that is rigorously carried out
viii. The changing nature of medical trials
ix. An investigative study that may lead to a new system
x. Why some scientists’ theories are considered second- rate

Example Paragraph A: x
1. Paragraph B ________
2. Paragraph C ________
3. Paragraph D _______
4. Paragraph E ________
5. Paragraph F ________
6. Paragraph G ________
Try it and see
In the social sciences, it is often supposed that there can be no such thing as a controlled
experiment. Think again.
1. In the scientific pecking order, social scientific are usually looked down on by their peers in the
nature sciences. Natural scientists do experiments to test their theories or, if they cannot, they
try to look for natural phenomena that can act in lieu of experiments. Social scientists, it is
widely thought, do not subject their own hypotheses to any such rigorous treatment. Worse,
they peddle their untested hypotheses to government and try to get them turned into policies.
2. Governments require sellers of new medicines to demonstrate their safety and effectiveness.
The accepted gold standard of evidence is a randomized control trial, in which a new drug is
compared with the best existing therapy (or with a placebo, if no treatment is available).
Patients are assigned to one arm or the other of such a study at random, ensuring that the only
difference between the two groups is the new treatment. The best studies also ensure that
neither patient nor physician knows which patient is allocated to which therapy. Drug trials
must also include enough patients to make it unlikely that chance alone may determine the
result.
3. But few education programmes or social initiatives are evaluated in carefully conducted
studies prior to their introduction. A case in point is the ‘whole-language’ approach to reading,
which swept much of the English-speaking world in the 1970s and 1980s. The whole-language
theory holds that children learn to read best by absorbing contextual clues from texts, not by
breaking individual words into their component parts and reassembling them (a method known
as phonics). Unfortunately, the educational theorists who pushed the whole-language notion
so successfully did not wait for evidence from controlled randomized trials before advancing
their claims. Had they done so, they might have concluded, as did an analysis of 52 randomized
studies carried out by the US National Reading Panel in 2000, that effective reading instruction
requires phonics.
4. To avoid the widespread adoption of misguided ideas, the sensible thing is to experiment first
and make policy later. This is the idea behind a trial of restorative justice which is taking place in
the English courts. The experiment will include criminals who plead guilty to robbery. Those
who agree to participate will be assigned randomly either to sentencing as normal or to
participation in a conference in which the offender comes face-to-face with his victim and
discusses how he may make emotional and material restitution. The purpose of the trial is to
assess whether such restorative justice limits re-offending. If it does, it might be adopted more
widely.
5. The idea of experimental evidence is not quite new to the social science as sneering natural
scientists might believe. In fact, randomized trials and systematic reviews of evidence were
introduced into the social sciences long before they became common in medicine. An apparent
example of random allocations is a study carried out in 1927 of how to persuade people to vote
in elections. And randomized trials in social work were begun in the 1930s and 1940s. But
enthusiasm later waned. This loss of interest can be attributed, at least in part, to the fact that
early experiments produced little evidence of positive outcomes. Others suggest that much of
the opposition to experimental evaluation stems from a common philosophical malaise among
social scientists, who doubt the validity of the natural sciences, and therefore reject the
potential of knowledge derived from controlled experiments. A more pragmatic factor limiting
the growth of evidence-based education and social services may be limitations on the funds
available for research.
6. Nevertheless, some 11,000 experimental studies are known in the social sciences
(compared with over 250,000 in the medical literature). Randomised trials have been used to evaluate
the effectiveness of driver-education programmes, job-training schemes, classroom size, psychological
counseling for post-traumatic stress disorder and increased investment in public housing. And where
they are carried out, they seem to have a health dampening effect on otherwise rosy interpretations
of the observations.
7. The problem for policymakers is often not too few data, but what to make of multiple and
conflicting studies. This is where a body called the Campbell Collaboration comes into its own.
This independent non-profit organization is designed to evaluate existing studies, in a process
known as a systematic review. This means attempting to identify every relevant trial of a give
question (including studies that have never been published choosing the best ones using clearly
defined criteria for quality, and combining the results in a statistically valid way. An equivalent
body, the Cochrane Collaboration, has produced more than 1,000 such reviews in medical
fields. The hope is that rigorous review standards will allow Campbell, like Cochrane, to become
a trusted and authoritative source of information.
For question 7-10, choose NO MORE THAN TWO WORDS from the passage for each answer.
Fighting Crime
Some criminals in England are agreeing to take part in a trial designed to help reduce their chance of
7.________. The idea is that while one group of randomly selected criminals undergoes the usual
8._________, the other group will discuss the possibility of making some repayment for the crime by
meeting the 9.___________. It is yet to be seen whether this system, known as 10.________, will
work.
Your answers

1. 2. 3. 4. 5.

6. 7. 8. 9. 10.

D. WRITING (50 points)


Part 1. Finish each of the following sentences in such a way that it means the same as the one
printed before it. Write your answers in the space provided. (10 points)
1. He assumed that she would brush up her English after this course.
He took it _________________________________________.
2. Passengers can only board the plane when all bags have been checked.
Only after _________________________________________.
3. My boss has got to persuade the investor to sign the contract.
My boss has got to ___________________________________.
4. Reports say that police have arrested a number of people since the robbery.
A number of people ___________________________________.
5. Josh does a very good imitation of the French teacher.
Josh takes ___________________________________________.
Part 2. Complete the second sentence so that it has a similar meaning to the first sentence, using
the word given. Do not change the word given. You must use between three and eight words,
including the word given. (10 points)
1. Most people seem to think that I will be next to be promoted. LINE
Most people seem to think that __________________________a promotion..
2. Paul wanted to be famous so much that he participated in a reality show. DESIRE
So great _____________________________ that he participated in a reality show.
3.The police have put a barrier around the building to keep people out. CORDONED
The building ___________________________ police.
4.He was unable to take part in the tournament because of nagging injury. RULED
A nagging injury ____________________________ the tournament.
5. Troops are not allowed to go into the town’s pub and bars. BOUNDS
The town’s pubs and bars __________________________ troops.
TEST 11
1. LISTENING (50 pts)
PART 1.
You are going to listen a journalist called Max Wilson talking about a book about luck in sport by
Matthew Syed. Listen to the whole interview and choose the best answer A, B, or C.(10 pts)
1. Max says that top sportspeople usually believe their success is due to
A. good fortune
B. hard work
C. natural skill
2. According to Max, the examples of recent sporting achievements prove
A. that people in general have become stronger and fitter
B. that standards are getting higher
C. that technology is responsible for improved performance
3. In the book Matthew Syed says he had a greater chance of success because of
A. his parents’ love of table tennis.
B. his competitive brother.
C. his own ambition.
4. That advantage is mentioned of the Omega Club when Matthew joined?
A. It was open all the time.
B. It had a lot of good players.
C. It had great facilities.
5. Max says that a ten-year investigation has shown that lucky people
A. believe they will succeed.
B. look for good opportunities.
C. depend less on talent.

🖎Write your answers here:

1. 2. 3. 4. 5.

PART 2.
Listen to a nutritionist called Penny Flack talking about the effects of health and diet in some
countries around the world. Are these statements True (T) or False (F)? (10 pts)
EATING FOR HEALTH
1 A quarter of Europeans and Americans are now said to be obese.
2 American politicians have been discussing how to tackle the causes and
consequences of obesity.
3 High-fat cheese and meat is causing the French to become obese.
4 Heart disease is becoming more common in Japan and Greenland.
5 Scientists have discovered that a number of spices used in Indian cooking can improve brain
health.

🖎Write your answers here:

1. 2. 3. 4. 5.

PART 3.
Listen to a talk on insomnia – the inability to sleep properly. Complete the following notes on the
talk about insomnia. Write NO MORE THAN TWO WORDS AND/OR A NUMBER for each answer. (20
pts)

 CAUSES
People may have trouble falling asleep due to worries about exams or a (1) ………………..
…………
It can be due to (2) ………………………………….factors: noise, light, no privacy.
It can be due to occupational factors: working irregular hours, overworking, too much (3)
…………………………………., high stress.
 SOLUTIONS
In situations where the patient is suffering from illness and physical discomfort, a doctor
may give them (4) …………………………………. or (5) ………………………………….
(6) …………………………………. before going to bed.
Watch your diet. Don’t eat a large meal in the evening. Avoid alcohol, cola and coffee.
Drink herbal tea (e.g. camomile) or (7) ………………………………….
Don’t take naps during (8) ………………………………….
Take a (9) ………………………………….before bed or after exercise.
Cut down on (10) …………………………………. in the evening.

🖎Write your answers here:


1. ………………………………… 6. ………………………………………
2. ………………………………… 7. ………………………………………
3. ………………………………… 8. ………………………………………
4. ………………………………… 9. ………………………………………
5. ………………………………… 10. ………………………………………
B. LEXICO AND GRAMMAR (50 pts)
Part I. Choose the best option A, B, C, or D to complete the following sentences. (20pts)
1. As it’s not out on DVD yet, we might ______ get it on video, I suppose.
A. also B. as well C. too D. and
2. I’m ______ my brother is.
A. nowhere like so ambitious B. nothing near as ambitious as
C. nothing as ambitious than D. nowhere near as ambitious as
3. The government was finally______ down by a minor scandal.
A. brought B. taken C. come D. pulled
4. It appears that the hostages were not______ to any unnecessary suffering.
A. subjugated B. subjected C. subsumed D. subverted
5. The incoming administration ______ to clean up corruption in the city.
A. pledged B. contemplated C. suggested D. resumed
6. The company has just got a big order and the workers are working round the ------.
A. day B. clock C. hour D. night
7. My brother Ted is a high school dropout who joined a circus; he is the black ------ in the family.
A. sheep B. dog C. cat D. goat
8. When James came home at three in the morning, his father hit the ------.
A. door B. chair C. table D. ceiling
9. Forget it. It is no use crying over ------ milk.
A. spoiled B. spilt C. lost D. stolen
10. Coin collecting is interesting, but you find a valuable coin only once in a blue ------.
A. moon B. sky C. star D. space
11. John got sick, then his brothers and sisters all got sick. It never rains but it ------.
A. falls B. drops C. pours D. dries
12. Can you __________ to it that no one uses this entrance?
A ensure B guarantee C assure D see
13. Next year I hope to __________ my ambition of climbing Mont Blanc.
A complete B follow C realise D impose
14. We should all ___________ when advertisers attempt to use unfair practices.
A. make a stand B. make a deal C. make amends D. make a comeback
15. We were not _________ convinced by his arguments.
A. closely B. extremely C. entirely D. widely
16. I’m sure that never happened. It’s just a ______ of your imagination.
A. figment B. piece C. picture D. fantasy
17. Did you see Jonathan this morning? He looked like ______. It must have been quite a party last
night.
A. a wet blanket B. a dead duck C. death warmed up D. a bear with a sore head
18. In the ______ of security, personnel must wear their identity badges at all time.
A. requirement B. demands C. assistance D. interests
19. I thought I had made it ______ that I didn’t wish to discuss this matter.
A. distinct B. plain C. frank D. straight
20. It's freezing outside so ______ to stay home and be warm.
A. we'd do well B. we might as well C. we'd better D. we're better off

Part II. Find out ten mistakes in the following passage then correct them. (10pts)
Water scarcity is fast becoming one of major limiting factors in world crop production. In many areas,
poor agricultural practices have led to increased desertification and the loss of former arable lands.
Consequently, those plant species are well adapted to survive in dry climates are being looked at for
an answer in developing more efficient crops to grow in marginally arable lands.
Plants use several purely mechanical and physical adaptations, such as the shape of the
Planet’s surface, small leaf size, and extensive root system. Some of adaptations are related with
chemical mechanism. Many plants such as cactuses, have internal gums which give them water
retaining properties. Another chemical mechanism is that of the epicuticular wax layer. This was layer
acts like a cover to protect the plant which prevents loss of internal moisture.
Part III. Read the text below. Use the word given in capitals at the end of some of the lines to form a
word that fits in the space in the same line. There is an example at the beginning (0). (10pts)

A recent poll on the use of animals in circuses showed that the (0. MAJOR) … MAJORITY… of people in
the UK now disapproves of it. Circuses which employ animals are no longer seen as a form of
(1. HARM) ……………….entertainment; in fact, most people think they should be banned outright.
Eighty percent of those interviewed (2. EQUIVOCAL) ……………………. declared that the use of
endangered wild animals such as elephants and tigers should be prohibited, while sixty-five percent
said no animals (3. WHAT) ……………………. should be used in circuses. A large proportion also claimed
they were opposed to the inevitable (4. BRUTAL) ……………..involved in training animals to perform
tricks. Animals in the wild do not juggle balls, ride monocycles, leap through (5. FIRE)…………………
hoops or wear clown costumes. Furthermore, besides being kept in (6. CONFINE) …………………….,
circus animals travel for most of the year, living a life of (7. DEPRIVE) ……………………. Unfortunately,
there is evidence to indicate that most animals face (8. TREAT) …………………….on a daily basis. The
number of people who visit animal free circuses these days is over twice the number of those who
visit traditional circuses. Animal free circuses are growing in number as well as (9. POPULAR)
……………………, and many say that the quality of the acts performed by humans far (10. EXCESS)
……………………. those acts that use animals.

C. READING COMPREHENSION (50 pts)


Part I. Read the text below and decide which answer best fits each space. (10 pts)
(1) ___________ popular belief, one does not have to be a trained programmer to work online. Of
course, there are plenty of jobs available for people with high-tech computer skills, but the growth of
new media has (2) ___________up a wide range of Internet career opportunities requiring only a
minimal level of technical (3) ___________. Probably one of the most well-known online job
opportunities is the job of webmaster. However, it is hard to define one basic job description for this
position. The qualifications and responsibilities depend on what tasks a particular organization needs
a webmaster to (4) ___________.
To specify the job description of a webmaster, one needs to identify the hardware and software that
the website will manage to run (5) ___________. Different types of hardware and software require
different skill sets to manage them. Another key factor is whether the website will be running
internally or externally. Finally, the responsibilities of a webmaster also depend on whether he or she
will be working independently, or whether the firm will provide people to help. All of these factors
need to be considered before one can create requiring (6) ___________ knowledge of the latest
computer applications. (7) ___________, there are also online jobs available for which traditional skills
remain in high (8) ___________. Content jobs require excellent writing skills and a good sense of the
web as a "new media".
The term "new media" is difficult to define because it encompasses a (9) ___________ growing set of
new technologies and skills. Specifically, it includes websites, email, Internet technology, CD-ROM,
DVD, streaming audio and video, interactive multimedia presentations, e-books, digital music,
computer illustration, video games, (10) ___________ reality, and computer artistry.
1. A. Apart from B. Contrary to C. Prior to D. In contrast to
2. A. taken B. sped C. set D. opened
3. A. expertise B. master C. efficiency D. excellency
4. A. conduct B. perform C. undergone D. overtake
5. A. on B. over C. in D. with
6. A. built-in B. up-market C. in-service D. in-depth
7. A. However B. Therefore C. Moreover D. Then
8. A. content B. demand C. reference D. requirement
9. A. constantly B. continually C. increasingly D. invariably
10. A. fancy B. imaginative C. illusive D. virtual

Part II. Read the passage and fill in each gap with ONE suitable word. (15 pts)
My stay among the indigenous people of Africa turned out to be a mixture of both success and failure.
I was able to observe firsthand (1) __________ the people lived instead of relying on books or films. I
was pleasantly surprised to find out that they were genial people. They were friendly (2) ___________
me from the very first day and they did not display any signs of hostility throughout my month long
stay. I felt indignant (3)____________ I thought of the terrible way they had always been (4)
__________ in films and books. I felt (5)____________ for them as they did not know how much
others (6) __________ them.
However, my trip was not a (7) __________ of roses. I had been warned many times by my colleagues
that when I stay with a tribe, I had to do as the Romans do. I should not be afraid to try new things
especially the food that they (8) __________ delicacies. These were highly prized and only eaten on
special (9)__________. Unfortunately, I did not (10) __________ their advice and I gagged on a piece
of meat that they offered me. They looked offended and from that day, many of them were not as
friendly as they used to be.

Part III. Read the passage and choose the best answer to each question. (10 pts)
HERBS AND DRUGS
Herbs are different kinds of plants and plant parts that can be used for medicinal purposes. This can
include the leaves, stems, roots, or seeds of the plant. Herbs have been important in traditional
medicine for centuries, in both of the East and the West. In Western medicine, they have largely been
replaced by drugs. Herbal treatments are, however, still an integral part of Eastern medicine. In recent
years, interest in traditional medicine has increased in the West. Many people are either using modern
methods combined with traditional treatments or are turning to these treatments entirely. While
traditional medicine can be helpful, they are not without their share of problems.
The main difference between herbs and drugs is that, while herbs are simply parts of plants, drugs are
specific chemicals in a pure form. Many modern drugs are derived from chemicals found in plants.
One example is aspirin, which is made from a chemical extracted from the back of the willow tree.
Other drugs are entirely synthetic. Even those drugs that are derived from natural sources are heavily
processed in order to purify and concentrate them. This allows drugs to be administered in very
precise amounts. Different kinds and degrees of illnesses often require dosages that differ only
slightly. A little too much or not enough of a certain drug can have negative effects on the patient.
Many drugs also produce negative effects even when taken in the recommended dosage. These
undesired negative effects are called “side-effects”
The possibility of dangerous and unexpected side-effects from drugs has led many people back to
traditional medicine. Herbal treatments seem more natural than modern drugs. Many feel that
traditional medicine is more concerned with treating the underlying causes of disease instead of just
the symptoms, through the truth of this claim is not yet clear. In any case, there is a certain comfort in
taking natural herbal treatments instead of the processed, synthetic chemicals derived from them.
There are, however, disadvantages to herbal treatments, some of them are serious. Few herbal
treatments have been scientifically studied. While the active chemicals in the herb may be known, it
may not be clear what they really do, or if they are really effective at all. Because the active chemicals
are not used alone, it is very difficult to determine the proper amount for treatment, since the levels
of the chemical are not constant throughout the plant. Therefore, the risk of under- and overdose is
higher than with drugs. A more serious problem is that certain herbal treatments may have no real
medicinal effect, thus giving the patient a false sense of security. This is particularly harmful when the
patient refuses treatment with modern drugs that could be effective.
There are even certain herbs that can cause side-effects, just like a drug. (A) One of these is ma-huang,
also called ephedra, which is taken to increase energy. (B) It has been known to cause damage to the
heart and nervous system. Garlic and ginger are common elements in food that are also taken as
herbal treatments, but they can be dangerous for people with diabetes. (C) In general, herbs are most
dangerous when they are taken along with common drugs. This may happen in two ways: a patient
decides to supplement his or her regular treatment with herbs, or a dishonest manufacturer adds
modern drugs to an herbal treatment. (D) In both cases, the results can be very serious. The herb St.
John’s wort is often used to treat depression, but if it is used along with conventional antidepressants,
such as Zoloft, the combination can cause confusion, headaches, allergic reactions, and other
problems.
Finally, because production of herbal treatments is seldom regulated, harmful substances can be
present in herbal preparations. Herbs grown in polluted soil may contain lead, arsenic, or mercury.
They may also be tainted with pesticides. It is for these reasons that herbs should not be treated as
the perfect substitute for drugs. Although herbs appear to be quite distinct from modern drugs, it is
important to use them with the same sort of care.
1. The word “integral” in the first paragraph is closest in meaning to
A. harmful. B. important. C. famous. D. controversial.
2. Which of the following is closest in meaning to “tained” in paragraph 6?
A. Accompanied. B. Augmented. C. Substituted. D. Contaminated.
3. The word “this” in paragraph 4 line 8 refers to
A. patients taking ineffective herbs instead of drugs.
B. patients having a sense of security.
C. the risk of under and overdose of an herbal treatment.
D. the patient’s belief that modern drugs are bad for you.
4. Why does the author mention that drugs are heavily processed?
A. To show that herbs cannot compete with modern medical techniques.
B. Because many mistakes can occur in this processing.
C. To illustrate that drugs are not natural.
D. To point out that many countries cannot afford to produce drugs.
5. What is the main difference between herbs and drugs according to the passage?
A. Drugs can cause side-effects, whereas herbs do not.
B. Drugs are at least partially synthetic, whereas herbs are natural.
C. Herbs are dangerous when taken in large amounts, drugs are safer.
D. Herbs can produce a false sense of security, whereas drugs do not.
6. According to the passage, when are herbs most dangerous?
A. When not taken under a doctor’s supervision.
B. When the patient refuses modern medicine.
C. When the dosage is not administered precisely.
D. When taken in combination with drugs.
7. According to the passage, who should not take ginger or garlic as herbal treatments?
A. Patients with diabetes.
B. Patients with liver damage.
C. Patients taking antidepressants.
D. Patients with problems of the heart or nervous system.
8. Which of the following is NOT mentioned in the passage?
A. Some herbal treatments are not effective in fighting disease.
B. Modern doctors often do not approve of using herbs.
C. Herbal treatments can also produce side-effects.
D. Some manufacturers add drugs to herbs.
9. It can be inferred from the passage that.
A. The drug industry is better regulated than the herb industry.
B. People who use drugs instead of herbs recover more quickly.
C. The popularity of herbal treatments will decrease in the future.
D. The side-effects of drugs are more serious than those of herbs.
10. Where the following sentence could be added to paragraph 5:
“Certain herbs have also been known to be harmful for people suffering from asthma.”
A. (A) B. (B) C. (C) D . (D)
Part IV. Read the passage and do the tasks that follow. (15 pts)
Secrets of the Forest
A. In 1942 Allan R.Holmberg, a doctoral student in anthropology from Yale University, USA, ventured
deep into the jungle of Bolivian Amazonia and searched out an isolated band of Siriono Indians. The
Siriono, Holmberg later wrote, led a "strikingly backward" existence. Their villages were little more
than clusters of thatched huts. Life itself was a perpetual and punishing search for food: some families
grew manioc and other starchy crops in small garden plots cleared from the forest, while other
members of the tribe scoured the country for small game and promising fish holes. When local
resources became depleted, the tribe moved on. As for technology, Holmberg noted, the Siriono "may
be classified among the most handicapped peoples of the world". Other than bows, arrows and crude
digging sticks, the only tools the Siriono seemed to possess were "two machetes worn to the size of
pocket knives".
B. Although the lives of Siriono have changed in the intervening decades, the image of them as Stone
Age relics has endured. Indeed, in many respects the Siriono epitomize the popular conception of life
in Amazonia. To casual observers, as well as to influential natural scientists and regional planners, the
luxuriant forests of Amazonia seem ageless, unconquerable, a habitat totally hostile to human
civilization. The apparent simplicity of Indian ways of life has been judged an evolutionary adaptation
to forest ecology, living proof that Amazonial could not - and cannot - sustain a more complex society.
Archaeological traces of far more elaborate cultures have been dismissed as the ruins of invaders from
outside the region, abandoned to decay in the uncompromising tropical environment.
C. The popular conception of Amazonia and its native residents would be enormously consequential if
it were true. But the human history of Amazonia in the past 11,000 years betrays that view as myth.
Evidence gathered in recent years from anthropology and archaeology indicates that the region has
supported series of indigenous cultures for eleven thousand years; an extensive network of complex
societies - some with populations perhaps as large as 100,000 - thrived there for more than 1,000
years before the arrival of Europeans. (Indeed, some contemporary tribes, including the Siriono, still
live among the earthworks of earlier cultures.) Far from being evolutionarily retarded, prehistoric
Amazonian people developed technologies and cultures that were advanced for their time. If the lives
of Indians today seem "primitive", the appearance is not the result of some environmental adaptation
of ecological barrier; rather it is a comparatively recent adaptation to centuries of economic and
political pressure. Investigators who argue otherwise have unwillingly projected the present onto the
past.
D. The evidence for a revised view of Amazonia will take many people by surprise. Ecologists have
assumed that tropical ecosystems were shaped entirely by natural forces and they have focused their
research on habitats they believe have escaped human influence. But as the University of Florida
ecologist, peter Feinsinger, has noted, an approach that leaves people out the equation is no longer
tenable. The archaeological evidence shows that the natural history of Amzonia is to a surprising
extent tied to the activities of its prehistoric inhabitants.
The realization comes none too soon. In June 1992 political and environmental leaders from across
the world met in Rio de Janeiro to discuss how developing countries can advance their economies
without destroying their natural resources. The challenge is especially difficult in Amazonia. Because
the tropical forest has been depicted as ecologically unfit for large-scale human occupation, some
environmentalists have opposed development of any kind. Ironically, one major casualty of that
extreme position has been the environment itself. While policy makers struggle to define and
implement appropriate legislation, development of the most destructive kind has continued apace
over vast areas.
F. The other major casualty of the "naturalism" of environmental scientists has been the indigenous
Amazonians, whose habits of hunting, fishing, and slash-and-burn cultivation often have been
represented as harmful to the habitat. In the clash between environmentalists and developers, the
Indians, whose presence is in fact crucial to the survival of the forest, have suffered the most. The new
understanding of the pre-history of Amazonia, however, points toward a middle ground. Archaeology
makes clear that with judicious management selected parts of the region could support more people
than anyone thought before. The long-buried past, it seems, offers hope for the future.
Questions 1-4
Choose the most suitable headings (i-vi) for sections A, B, C and D
List of Headings
i. Amazonia as unable to sustain complex societies
ii. The role of recent technology in ecological research in Amazonia.
iii. The hostility of the indigenous population to North America influences
iv. Recent evidence
v. The influence of prehistoric inhabitants on Amazonian natural history.
1. Section A_______
2. Section B_______
3. Section C_______
4. Section D_______
Question 5-10
Do the following statements agree with the view of the writer of the passage? Please write
Yes If the statement agrees with the view of the writer
No If the statement contradicts the view of the writer
Not given If it is impossible to say what the writer think about this
5. The reason for the simplicity of the Indian way of life is that Amazonian has always been unable to
support a more complex society. _________________
6. There is a crucial popular misconception about the human history of Amazonia. ______________
7. There are lessons to be learned from similar ecosystem in the other parts of the world.
___________
8. Most ecologists were aware that the areas of Amazonia they were working in had been shaped by
human settlement. _____________
9. The indigenous Amazonian Indians are necessary to the well-being of the forest. ______________
10. It would be possible for certain parts of Amazonia to support a higher population. _____________

D. WRITING (50 pts)


Part I. Rewrite each of the following sentences in such a way that it stays the same meaning to the
first one, using the word given. Do not change the word given. (10pts)
1. Don't panic about something so trivial. MOUNTAIN
_______________________________________________________________________
2. I suddenly realized the meaning of a "freebie". DAWNED
_______________________________________________________________________
3. They arrived at their destination alive and kicking. SOUND
_______________________________________________________________________
4. My jewellery has been stolen. OFF
_______________________________________________________________________
5. This is a subject that Jack and I disagree about. EYE
_______________________________________________________________________

Part II. Complete the second sentence so that it has a similar meaning to the first sentence, using
the word given. Do not change the word given. You must use between three and six words,
including the word given. There is an example at the beginning (0). (10pts)
0. He paid no attention to our warning.
NOTICE
He ……… took no notice of …………. our warning.
1. I’m not sure which year saw the abolition of capital punishment in this country.
DID
I’m not sure when they ……………………………………………………………… capital punishment in this country.
2. I wish I hadn’t said that to her.
TAKE
If only ………………………………………………………………I said to her.
3. If children were allowed to do what they wanted, they would probably play computer games all
day.
OWN
If children were ……………………………………………………………… they would probably play computer games
all day.
4. If Tom hadn’t acted promptly to extinguish the fire, there might have been more damage to the
house.
IN
But ……………………………………………………………… out the fire, there might have been more damage to the
house.
5. Initially, everybody believed his story but now they think he was lying.
UP
He is now ……………………………………………………………… the story.
TEST 12
I. LISTENING:
PART 1: You will hear part of a radio interview with a psychologist, about friendship. For questions
1-5, choose the answer (A, B, C or D), which fits best according to what you hear. (10 points)
1. According to the presenter _____.
A we have to adjust our friendships as our lives change.
B new books are being written to teach people how to manage friendships.
C it’s almost impossible to keep childhood friends for life.
D the most important friends are childhood friends.
2. The danger of a trophy friend is that _____.
A he or she will inhibit your social life.
B you may develop unrealistic expectations.
C he or she will push you to be a higher achiever.
D he or she will expect you to become more popular.
3 A sisterly friend _____.
A can be relied on but may be too involved in your life.
B will resent other close friendships in your life.
C will be as close to your family as she is to you.
D will want to go out on dates with you and your new partner.
4. According to Barbara, a good friend _____.
A probably needs to be someone that you see very often.
B tends to ask for your help slightly more than you would like but you tolerate it.
C doesn’t expend too much of your time or energy.
D never puts you in a serious mood.
5. When making new friends _____.
A try to be as funny as you can.
B don’t mention serious issues before you have got to know them well.
C be sensitive to their need for space.
D spend as much time with them as you can

🖎Write your answers here:

1. 2. 3. 4. 5.

PART 2: You will hear part of a radio interview about fox hunting. For questions 1-5, decide
whether the following statements are true (T) or false (F) .Write your answers in the corresponding
numbered boxes (10 points)
1. Nothing seems to have changed after the new law on hunting.
2. According to Brian Hook, it is impossible for them to monitor all the hunts.
3. The new law allows dogs to kill a fox as long as the kill is monitored.
4. Hook believes that traditional hunting methods are the only way to control the fox population.
5. The interviewer seems to think that the issue is too emotive to be settled easily.

🖎Write your answers here:

1. 2. 3. 4. 5.

PART 3: You will hear part of a radio programme presented by author and foodie, Pat
Chapman. For questions 1-10, complete the sentences with a word or short phrase. (20 points)
Pat says that the British were a .................................................... (1) according to Napoleon.
Pat likens curry in Britain nowadays to a ............................................... (2)
Britain suffered from a ................................................. (3) in the period after the Second World War.
Immigrants to Britain had to arrange for their prized ................................................ (4) to be imported.
Eating curry became compulsive as the dish was ................................................. (5) for most people.
The majority of curry restaurants in the UK are not ..................................................... (6)
Indian dishes prepared in their own containers need .................................................... (7) to be
authentic.
Nowadays, additional ................................................... (8) are added to pre-cooked ingredients.
A ........................................................ (9) is responsible for cooking breads and tandoori items.
This is still the formula of the ...................................................... (10) house.

🖎Write your answers here:

1. 6.

2. 7.
3. 8.

4. 9.

5 10.

II: Grammar& Vocabulary


Part 1: Choose the best option to complete each of the following questions.
1. I’m sure there’s a definite __________ of envy in her nasty comments about you.
A. factor B. ingredient C. component D. element
2. In the ___________ of just two days, her whole life changed.
A. interval B. space C. spell D. duration
3. The thieves took ___________ when they heard a police car approaching.
A. retreat B. flight C. escape D. getaway
4. Afterwards, when I ___________ on the events of that day, I could hardly believe what had
happened.
A. contemplated B. reviewed C. reflected D. weighed
5. There was a huge ___________ of applause when the star of the show appeared.
A. bout B. stint C. round D. spate
6. As he accepted the award, his voice ___________ with emotion.
A. quivered B. flinched C. cringed D. winced
7. Rose has always had a ___________ interest in matters to do with the environment.
A. fierce B. sharp C. grave D. keen
8. By the ___________ of it, the economy will improve over the next few months.
A. face B. impression C. evidence D. look
9. Tim and Alan have never got on well and there is a lot of ___________ feeling between them.
A. cross B. adverse C. ill D. vile
10. Peter has now arrived late for work three days in a ___________ .
A. line B. sequence C. series D. row
11. Harry doesn’t ___________ to great fame and fortune; he just wants to make a decent living.
A. crave B. hanker C. yearn D. aspire
12. A lot of people are ___________ to the damage that is being done to the environment.
A. blind B. unacquainted C. uninitiated D. ignorant
13. She showed little ___________ of the problems we were facing.
A. affinity B. appreciation C. regard D. sensitivity
14. This is the ___________ timetable for the conference – it may change later.
A. conditional B. provisional C. indeterminate D. indefinite
15. They worked for six ___________ hours without a break.
A. solid B. incessant C. perpetual D. constant
16. I wrote to them a fortnight ago but ___________ I haven’t had a reply.
A. as yet B. so long C. just now D. these days
17. I couldn’t stop myself from ___________ with boredom during the lecture.
A. sighing B. panting C. blowing D. gasping
18. She didn’t show even a ___________ of emotion when the court found her guilty.
A. wink B. flicker C. flash D. gleam
19. The manager’s future ___________ whether the team wins or loses this one game.
A. stems from B. derives from C. counts on D. rests on
20. Sam has always taken the ___________ that there is more to life than money.
A. view B. outlook C. belief D. opinion
Part 2: Write the correct form of each bracketed word in the numbered space provided in the
column on the right.
When Bryan Ferry recorded his solo album, Another 1. __________________
Time, Another Place, in 1974, he was an apparently
2. __________________
unstoppable, 1. ________ (EXHAUST) creative force. His
band, Roxy Music, was barely two years old. During a brief 3. __________________
and 2. ________ (METEOR) ascent, the band had released
4. __________________
three albums and, under Ferry’s close 3. ________
(ART) guidance, refashioned the rock’n roll experience as a 5. __________________
4. ________ (WEIRD) costumed trip around some 5. 6. __________________
________ (FUTURE) archive. 6. ________ (WHERE)
between global engagements with Roxy music, Ferry had 7. __________________
found time to launch a solo career seemingly dedicated to 8. __________________
7. ________ (HONOR) the songs he grew up listening to.
9. __________________
Nowadays, albums of old hits (cover versions) are a
standard career ploy, but back in 1973 such retrospective 10. __________________
dalliance was simply not the done thing amongst rock
musicians. Neither was posing for your sleeve photo in full
evening dress, like a posh matinee idol from the 1940s.
But 8. ________ (LOOSE) the iron grip of 9. ________
(CONFORM) rock behavior was 10. ________ (PRECISION)
Ferry’s point, and he had more than enough musical wit
and wisdom to back up these outlandish postures.
Part 3: Correcting mistakes: The passage below contains 10 errors. Underline and correct them.
Write your answers in the space provided on the right.

Tourism is now among the world’s most important 1.___________________


industries, generate jobs and profits worth billions of
2. __________________
pounds. At the same time, however, mass tourism can have
dire effects on the people and places they embraces – both 3.___________________
tourists and the societies and human environments they
4. __________________
visit. We are increasingly familiar with some of the worst
effect of unthinking, unmanaged, sustainable tourism: 5. __________________
previous undeveloped coastal villages that have become 6. __________________
sprawling, charmless towns, their seas poisonous by sewage,
denuded of wildlife, their beaches stained by litter and 7. __________________
empty tubes of sun cream. Historic towns, their streets now 8. __________________
choking with traffic, their temples, churches and cathedrals
seemingly reduced to backdrop for holiday snaps that 9. __________________
proclaim, “Been there, Done that”. Some of the world’s 10. _________________
richest environments were bruised by the tourist onslaught,
their most distinctive wildlife driven to near-extinction, with
wider environmental impacts caused by the fuel-hungry
transport systems used to take holidaying travelers around
the world and back again.

III. READING.
Part 1. For each gap, choose the correct answer A, B, C or D which best fits the context.
Nature’s clock
Our biological clocks govern almost every aspect of our life. Our sensitivity to stimuli (1) _______ over
the course of the day, and our ability to perform certain functions is subject to fluctuations.
Consequently, there is an (2) _______ time for tasks such as making decisions: around the middle of
the day. Anything that (3) _______ physical co-ordination, on the other hand, is best attempted in the
early evening. What is more, there is a dramatic drop in performance if these activities are carried out
at other times. The risk of accident in a factory, for example, is 20% higher during the night (4)
_______.
Primitive humans lived their lives in tune with the daily cycle of light and dark. Today we are firmly
convinced that we can (5) _______ schedules on our life at (6) _______. Sooner or later, however, we
pay a (7) _______ for ignoring our natural rhythms. A good example is jet lag, caused when we
confuse our body’s biological clocks by crossing several time (8) _______. People suffering from jet lag
can take several days to adjust to new environments, and have a reduced ability to make decisions,
which is a worrying thought, as serious (9) _______ of judgement can be made. And this may be just
the (10) _______ of the iceberg.
Question 1: A. modifies B. ranges C. varies D. wavers
Question 2: A. peak B. optimum C. maximum D. summit
Question 3: A. requests B. demands C. dictates D. stipulates
Question 4: A. shift B. labor C. duty D. work
Question 5: A. blame B. base C. emphasize D. impose
Question 6: A. stake B. best C. most D. will
Question 7: A. price B. fine C. fee D. cost
Question 8: A. warps B. trials C. spans D. zones
Question 9: A. errors B. inaccuracies C. mistakes D. fallacies
Question 10: A. peak B. pinnacle C. top D. tip
Part 2: Fill in each blank in the following passage with ONE suitable word.
Evolution and children
If we are asked to envisage an archetypal human 1. ________, the picture that comes into our minds
may be male or female. It may be black, white or yellow, but it will almost certainly be an adult. We
take it for granted that 2. ________ is the meaningful part of our existence, and everything prior to it
is merely preparation. The old adage quoted by Samuel Butler is often 3. ________ but has not yet
been fully assimilated: “A hen is an egg's way of making another 4. ________.” It is very difficult for
any of us to think of 5. ________ as a baby's way of making another baby.
So there is a tendency in discussions about human evolution to overlook the fact that at every step of
the journey there were not only males and females, but also babies, infants and children, and 6.
________ selection would never have favoured one age group at too great a cost to 7. ________ of
the others.
8. ________ children as smaller, imperfect copies of ourselves, we explain much of their behaviour in
the way we explain the rough-and-tumble play of cubs and kittens, calling it “preparation for adult
life” or “developing the skills that they will later need.” That is strange, because it is one of the
inviolable tenets of evolutionary theory that what an animal is or 9. ________ is governed by events
that have happened, 10. ________ events that are going to happen. Only in describing the young is it
acceptable to believe that a mammal's behaviour is governed by the future that awaits it, rather than
the history that lies behind it.
Part 3: Read the following passage and choose the best answer to each question.
When one hears the expression “role models”, one’s mind naturally jumps to celebrities, especially as
far as young people are concerned. Therefore, it would be more than natural to assume that
teenagers, heavily influenced by the media, are dazzled by well-known Hollywood stars, famous
musicians and internationally renowned athletes. However, nothing could be further from the truth.
In reality, according to a recent survey, over 75% of teens who filled out an online questionnaire
claimed that the role model for whom they had the greatest respect was not a famous personality, but
a family member. It seems that the qualities that make a good role model are more complex than
researchers first assumed. For example, Nancy L, a teenage girl from Wisconsin, described her role
model as a woman who had a clear sense of what was important to her, making the effort to create
things that would make a real difference in the world. The woman she was referring to was her
favorite aunt, who was a painter and sculptor.
Role models come into young people’s lives in various ways. They are family members, educators,
peers and ordinary people encountered in their daily lives. Students emphasized that being a role
model is not confined to those with international fame or unbelievable wealth. Instead, they said the
greatest attribute of a role model is the ability to inspire others. Teachers were often mentioned as
examples in this case, ones that are dedicated to encouraging students, helping them push their limits
and strengthen their characters.
Another quality high on the list was the ability to overcome obstacles. In addition to parents, peers
often made up a large percentage of such role models. Young people are at a point in their lives when
they are developing the skills of initiative and capability, so it is only natural that they admire people
who show them that success in the face of difficulty is possible.
A final and perhaps unexpected character trait that the youth of today admire is a clear set of values.
Children admire people whose actions are consistent with their beliefs; in other words, who practice
what they preach. Role models help them to understand the significance of honesty, motivation and
the desire to do general good. For example, local politicians who clearly struggle to improve living
conditions in their cities are high on their lists of role models.
Perhaps what should be understood from what young people consider important in a role model is
that each and every person around them affects them to a certain extent, perhaps much more than
most parents think. This makes it crucial for adults to be aware of their influence on the young and set
the best examples possible.
Question 1: Which of the following is closest in meaning to “dazzled”?
A. impressed B. disappointed C. confused D. frightened
Question 2: Which of the following is LEAST likely to be assumed as teens’ role model?
A. A handsome actor B. A talented footballer
C. A hot popstar D. A brilliant scientist
Question 3: What is surprising about the findings of the survey?
A. Celebrities are the most common role models to most teens.
B. The role models of the respondents are not quite influential.
C. The qualities that make up teens’ role models are not simple.
D. Most celebrities have their family members as role models.
Question 4: What does the passage tell us about Nancy L’s role model?
A. She was not related to her. B. She was famous for her talent.
C. She was a mysterious person. D. She had strong priorities.
Question 5: Which of the following is closest in meaning to “confined to”?
A. assisted by B. restricted to C. similar to D. influenced by
Question 6: According to the passage, what quality makes teachers good role models?
A. their ambition to succeed B. their wide knowledge
C. their ability as academic educators D. their positive effect on students
Question 7: The ability to overcome obstacles is important to young people because ______.
A. teens must have it to teach their peers
B. it is not something that one can easily find
C. obstacles make life more difficult
D. it is relevant to the stage of life they are in
Question 8: According to paragraph 5, children really look up to those who _______.
A. are as active as possible B. do what they say they will do
C. pay attention to the needs of the young D. are religious in their life
Question 9: According to the passage, some politicians are considered admirable ______.
A. because they are familiar to young people
B. because of the strong power they have
C. because of their concern for others
D. because they believe in themselves
Question 10: The passage suggests that adults should ______.
A. try to avoid imposing their influence on younger people
B. realize that they have a strong effect on young people
C. be careful of the role models their children may have
D. encourage children to reject celebrities as role models
Part 4: Read the following passage and answer the questions.
Questions 1-13 which are based on Reading Passage 1 below.
The Life of Sir Isaac Newton
A
Isaac Newton was born on January 4, 1643, in Lincolnshire, England. The son of a farmer, who died
three months before he was born, Newton spent most of his early years with his maternal
grandmother after his mother remarried. Following an education interrupted by a failed attempt to
turn him into a farmer, he attended the King’s School in Grantham before enrolling at the University of
Cambridge’s Trinity College in 1661, where he soon became fascinated by the works of modern
philosophers such as René Descartes. When the Great Plague shut Cambridge off from the rest of
England in 1665, Newton returned home and began formulating his theories on calculus, light and
color, his farm the setting for the supposed falling apple that inspired his work on gravity.
B
Newton returned to Cambridge in 1667. He constructed the first reflecting telescope in 1668, and the
following year he received his Master of Arts degree and took over as Cambridge’s Professor of
Mathematics. In 1671 he was asked to give a demonstration of his telescope to the Royal Society of
London in 1671, the same year he was elected to the prestigious Society. The following year,
fascinated with the study of light, he published his notes on optics for his peers. Through his
experiments, Newton determined that white light was a composite of all the colors on the spectrum,
and he asserted that light was composed of particles instead of waves. His methods were heavily
criticized by established Society member Robert Hooke, who was also unwilling to compromise again
with Newton’s follow-up paper in 1675. Known for his temperamental defense of his work, Newton
engaged in heated correspondence with Hooke before suffering a nervous breakdown and
withdrawing from the public eye in 1678. In the following years, he returned to his earlier studies on
the forces governing gravity.
C
In 1684, English astronomer Edmund Halley paid a visit to the reclusive Newton. Upon learning that
Newton had mathematically worked out the elliptical paths of celestial bodies, such as the movement
of the planets around the sun, Halley urged him to organize his notes. The result was the 1687
publication of “Philosophiae Naturalis Principia Mathematica” (Mathematical Principles of Natural
Philosophy), which established the three laws of motion and the law of universal gravity. Principia
made Newton a star in intellectual circles, eventually earning him widespread acclaim as one of the
most important figures in modern science.
D
As a now influential figure, Newton opposed King James II’s attempts to reinstate Catholic teachings at
English Universities, and was elected to represent Cambridge in Parliament in 1689. He moved to
London permanently after being named warden of the Royal Mint in 1696, earning a promotion to
master of the Mint three years later. Determined to prove his position wasn’t merely symbolic,
Newton moved the pound sterling from the silver to the gold standard and sought to punish forgers.
E
The death of Hooke in 1703 allowed Newton to take over as president of the Royal Society, and the
following year he published his second major work, “Opticks.” Composed largely from his earlier notes
on the subject, the book detailed Newton’s experiments with refraction and the color spectrum, and
also contained his conclusions on such matters as energy and electricity. In 1705, he was knighted by
Queen Anne of England.
F
Around this time, the debate over Newton’s claims to originating the field of calculus, the
mathematical study of change, exploded into a nasty dispute. Newton had developed his
mathematical concept of ‘fluxions’ (differentials) in the mid-1660s to account for celestial orbits,
though there was no public record of his work. In the meantime, German mathematician Gottfried
Leibniz formulated his own theories and published them in 1684. As president of the Royal Society,
Newton oversaw an investigation that ruled his work to be the founding basis of the field, but the
debate continued even after Leibniz’s death in 1716. Researchers later concluded that both men likely
arrived at their conclusions independent of one another.
G
Newton was also obsessed with history and religious doctrines, and his writings on those subjects
were collected into multiple books that were published after his death. Having never married, Newton
spent his later years living with his niece at Cranbury Park, near Winchester, England. He died on
March 31, 1727, and was buried in Westminster Abbey. A giant even among the brilliant minds that
drove the Scientific Revolution, Newton is remembered as an extraordinary scholar, inventor and
writer. His theories about the movement of bodies in the solar system transformed our understanding
of the universe and his precise methodology helped to give birth to what is known as the scientific
method. Although his theories of space-time and gravity were eventually superseded by those of
Einstein his work remains the foundation stone of modern physics was built.

Questions 1-6
The text has seven paragraphs labelled A–G.
Reading passage 1 has seven paragraphs, A-G.
Choose the correct headings for paragraphs B-G from the list of headings below.
Write the correct number, i-viii, in boxes 1-6 on your answer sheet.
List of Headings
i Continued breakthroughs in research
ii Competing claims of originality
iii The early years of Sir Isaac Newton
iv The legacy of an exceptional mind
v Routine life at a 17th century university
vi Heated academic disputes
vii A new venture
viii His crowning achievement
ix A controversial theory about planets
Answer Example
iii Paragraph A
1 Paragraph B
2 Paragraph C
3 Paragraph D
4 Paragraph E
5 Paragraph F
6 Paragraph G
Questions 7-8
Answer the questions below.
Choose NO MORE THAN TWO WORDS from the passage for each answer
Write your answers in boxes 7-8 on your answer sheet.
7 With which scientific organization was Newton associated for much of his career?.....
8 With whom did Newton live as he got older?......
Questions 9-13
Complete the notes below.
Choose ONE WORD from the passage for each answer.
Write your answers in boxes 9-13 on your answer sheet.
Sir Isaac Newton’s achievements
 Created first reflecting 9………………….., subsequently made a professor at Cambridge at the age
of 25.
 Helped develop the scientific method with his experiments in 10……………………, the study of
light; showed that it is 11………………….., not waves, that constitute light.
 Worked out the laws of the movement of bodies in space (planets etc.), published Principia
Mathematica with laws of gravity and 12……………………..
 Joint founder (with Leibniz) of 13……………………., a new branch of mathematics.
IV. WRITING:
Part 1: A. For each of the sentence below, write a new sentence as similar as possible in
meaning to the original sentence using the word given. This word must not be altered in any way.
1. David played the main role when the proposal was drafted. (instrumental)
David _____________________________ of the proposal.
2. If you hadn't changed our original agreement, everything would have been fine. (stuck)
Had _________________________________ agreed, everything would have been fine.
3. I think you should have some consideration for those who don't have lives as privileged as yours.
(spare)
I think you should ________________________ lives aren't as privileged as yours.
4. We decided to stay for longer because we were so thrilled by the place. (prolong)
We decided to __________________________________ we by the place.
5. Competitors were amazed by how shrewdly he conducted his business affairs. (marveled)
Competitors ____________________________ which he conducted his business affairs.
B. For each of the sentence below, write a new sentence as similar as possible in meaning to the
original sentence.
1. There are plans to close down the cinema due to poor attendance levels.
The cinema is under _____________________________ due to poor attendance levels.
2. The machine looks dangerous, but I'll believe it's safe if you say so.
The machine doesn't look safe, but I’ll take _____________________________ it.
3. Once signed, this contract is binding.
As soon as you sign, you ___________________________ this contract.
4. Looking back, I really believe I did everything I could to stop her.
I can honestly say, with hindsight, _________________________________ more to stop her.
5. She has become a famous novelist.
She has made ____________________________________ a novelist.
TEST 13
A. LISTENING (50 points):
Part 1: Complete the table below. Write NO MORE THAN THREE WORDS for each answer. Write your
answers in the corresponding numbered boxes. (10 points)

Date Event Importance for art

3000 BC rice farmers from 1. ________ built temples with wood and stone
carvings settled in Bali

14th century introduction of Hinduism artists employed by 2. ________ and


focused on epic narratives

1906 Dutch East Indies Company art became expression of opposition


established to 3. ________

1920s beginning of 4. ________ encouraged use of new materials,


techniques and subjects

1945 independence new art with scene of 5. _________


(e.g. harvests) reflecting national
identity

Your answers:

1. 2. 3. 4. 5.

Part 2: You will hear a radio discussion about children who invent imaginary friends. Choose the
answer (A, B, C or D) which fits best according to what you hear. Write your answers in the
corresponding numbered boxes. (10 points)
1. In the incident that Liz describes,
A. her daughter asked her to stop the car.
B. she had to interrupt the journey twice.
C. she got angry with her daughter.
D. her daughter wanted to get out of the car.
2. What does the presenter say about the latest research into imaginary friends?
A. It contradicts other research on the subject.
B. It shows that the number of children who have them is increasing.
C. It indicates that negative attitudes towards them are wrong.
D. It focuses on the effect they have on parents.
3. How did Liz feel when her daughter had an imaginary friend?
A. always confident that it was only a temporary situation
B. occasionally worried about the friend’s importance to her daughter
C. slightly confused as to how she should respond sometimes
D. highly impressed by her daughter’s inventiveness
4. Karen says that one reason why children have imaginary friends is that
A. they are having serious problems with their real friends.
B. they can tell imaginary friends what to do.
C. they want something that they cannot be given.
D. they want something that other children haven’t got.
5. Karen says that the teenager who had invented a superhero is an example of
A. a very untypical teenager.
B. a problem that imaginary friends can cause.
C. something she had not expected to discover.
D. how children change as they get older.

Your answers:

1. 2. 3. 4. 5.

Part 3: You will hear an interview which takes place on a show between the interviewer, Michaela
Robinson, a leading psychologist, Duncan Stone, and a former patient, Ian Smith. Write T (for True)
or F (for False). Write your answers in the corresponding numbered boxes. (10 points)
1. Mr. Stone thinks that psychiatrists don’t need any medical training.
2. Psychiatrists need to know how to make patients open up.
3. Ian Smith didn’t know what caused his nervousness.
4. Boys at school were friendly to Ian.
5. Mr. Stone found it difficult to get Ian’s life story.

Your answers:

1. 2. 3. 4. 5.

Part 4: You will hear part of an interview with an explorer. Complete the sentences and write your
answers in the corresponding numbered boxes. (20 points)
The Explorer
It could be said that his expeditions over the last five years have mostly (1) _________ but there have
been (2) _________ on any of them.
He thinks that (3) _________ of his expeditions has been good and that (4) _________ and (5)
_________ have been right. One of the aims of his expedition to Antarctica is to raise money for a (6)
_________.
The second aim is to find out what the human body can tolerate when working very hard and (7)
_________ in extreme temperatures.
These days he does not emphasise (8) _________ involved in exploring because people don’t take it
seriously.
He intends to go to (9) _________ this year.
On the next expedition to Antarctica they are taking special sledges and (10) _________ than on the
previous expedition there.

Your answers:

1. 2. 3. 4. 5.

6. 7. 8. 9. 10.

B. LEXICO-GRAMMAR (50 points)


Part 1: Choose the best option A, B, C, or D to complete the following sentences and write your
answers in the corresponding numbered boxes. (20 points)
1. I’m afraid that the herring we had for dinner has given me _______.
A. sickness B. indisposition C. infection D. indigestion
2. This lime juice needs _______ before you can drink it.
A. diluting B. dissolving C. softening D. watering
3. It must be true. I heard it straight from the _______ mouth.
A. dog’s B. horse’s C. camel’s D. cat’s
4. The facilities at many schools today are still _______ inadequate.
A. sadly B. woefully C. regrettably D. grimly
5. His laziness at work made him _______ with his workmate.
A. improper B. disliked C. unpopular D. unappealing
6. Although he’s shy, it certainly hasn’t _______ his career in any way.
A. restricted B. obstructed C. cramped D. impeded
7. She used the map to discover where she was in _______ to her surroundings.
A. connection B. affinity C. relation D. reference
8. You can’t always depend on _______ on time.
A. the trains’ arriving B. the trains to arrive
C. the arriving of trains D. the train that arriving
9. You’d _______ that clock fixed; it hasn’t worked for over a week now.
A. rather have B. better have C. rather had D. have
10. The picking of the fruit, _______, takes about a week.
A. whose work they receive no money.
B. as they receive no money for that work
C. for which work they receive no money
D. they receive no money for it
11. If I don’t write you a note to say you have a doctor’s appointment, the teacher will think you are
playing ________.
A. the fool B. truant C. for time D. hard to get
12. Exercise tends to _______ the effects of old age.
A. waylay B. run down C. set back D. offset
13. There’s an old saying, “It’s the double gins that cause the _______.”
A. crossed eyes B. double chins C. flat feet D. knock knees
14. It’s a good area for shopping. All the shops are within easy _______.
A. approach B. arrival C. neighbourhood D. reach
15. The big new supermarket on the outskirts of town does a _______ trade.
A. crying B. roaring C. screaming D. shouting
16. Sport provides an _______ for a teenager’s feeling of aggression or frustration.
A. overflow B. outlet C. exit D. exhaust
17. She was caught cheating in the race. _______, she was disqualified.
A. Explicitly B. Accordingly C. Equally D. Fundamentally
18. Alice was not sure which profession to enter, but finally _______ for medicine.
A. opted B. chose C. accepted D. selected
19. Jane handed in the test and awaited the results _______.
A. with bated breath B. out of breath
C. under her breath C. in the same breath
20. Burglar alarms on cars and houses may act as a _______ to the casual thief.
A. prevention B. precaution C. stopper D. deterrent

Your answers:

1. 2. 3. 4. 5.

6. 7. 8. 9. 10.

11. 12. 13. 14. 15.

16. 17. 18. 19. 20.

Part 2: Read the passage below which contains 10 mistakes. Identify the mistakes and write the
corrections in the corresponding numbered boxes. There is an example at the beginning. (10 points)
Example: Line 1: Originalà Originally
THE NEW BRITISH LIBRARY
Original commissioned 14 years ago, the new British Library was supposed to open in 1990. Thus, the
project has been delayed by political infighting, poor planning and financial problems. The most
recent setback came in June when inspectors discover that 60 miles of new metal shelving had started
to rust and needed to be replaced. That would postpone the opening of the project’s first phase for
yet other two years. “Things have gone from bad to worse’, said Brian Lake, secretary of the Regular
Readers, an association of writers and scholars who are not happy with plans to the new library. “It is
a grand nation project that has become a great scandal”.
It sounded like a splendid idea which the government unveiled its 164- million project in 1978.
Sophisticated electronic equipment would help keep the library’s irreplaceable stock at an optimal
temperature and humid. A computer-controlled delivery system would provide books to readers
within minutes of a request better than days. And to serve other needs of the reading public, the
library would also include exhibition gallery, a restaurant and a conference hall.
That was the plan, anyway. The start of construction delayed until 1982 by arguments about planning
and by a change of government. Four years later, members of the cabinet ordered a progress report
and discovered that the committee responsible for supervision the project hadn’t met in four
years.
Your answers:

Line Mistakes Corrections

1.

2.

3.

4.

5.

6.

7.

8.

9.

10.

Part 3: Write the correct form of the words given in the brackets. Write your answers in the spaces
provided below. (10 points)
Rebuilding Conventry
In the late 30s, (1) __________ knew that the centre of the ANALYSE
historic town of Conventry in the West Midlands needed to
SHELF
be redeveloped. Plans had to be (2) __________ when the
Second World War started in 1939. However, the architects’ MATERIAL
opportunity (3) __________ when the city centre was
REPAIR
practically destroyed during the war. Many buildings were
SPACE
(4) __________ damaged and demolition work began.
Aiming to create a much more (5) __________ area for RESIDE

(6) __________ to work and shop in, town planners came up INHABIT
with a radical idea. They would make the city more OBJECT
(7) __________ by pedestrianising the centre, preventing DENSE
cars entering. There were (8) __________ from local
shopkeepers, who thought that it would have an impact on ARCHITECT
trade, but the planners went ahead. What was once a (9)
__________ populated area became a pleasant, attractive
place to visit. It was a real (10) __________ achievement,
one that many British towns have emulated since.

Your answers:

1. 2. 3. 4. 5.

6. 7. 8. 9. 10.

C. READING (50 points)


Part 1: Read the following passage and decide which answer (A, B, C, or D) best fits each gap. Write
your answers in corresponding numbered boxes. (10 points)
All in the Stars
First-time visitors to India are likely to be impressed by how profoundly astrology influences almost
every (1) ______ of life on the subcontinent. In fact, the belief that the motions of remote heavenly
bodies can affect events on Earth is so (2) ______ that several Indian universities (3) ______ courses in
the subject. It is not, therefore, surprising that many people will (4) ______ an astrologer before they
take any important step. For example, Indian marriages are arranged with the aid of an astrologer,
who will cast the horoscopes of the bride and groom, and also work out the best date for the wedding
to take place. A few years ago in Delhi, thousands of couples rushed to get married on a particularly
auspicious day, with the (5) ______ that priests, brass bands and wedding photographers were in
short supply.
The role of astrology is not (6) ______ only to the social aspects of Indian life. Few people (7) ______
business without resorting to their astrologer. Major films are only released on suspicious dates. Even
(8) ______ of state are not exempt from its influence: when India (9) ______ her independence from
Britain in 1947, the (10) ______ of power was carefully timed to take place after a particularly
inauspicious period had passed.
1. A. division B. facet C. angle D. sector
2. A. widespread B. overwhelming C. intensive D. capacious
3. A. offer B. afford C. supply D. serve
4. A. interrogate B. confer C. interview D. consult
5. A. effect B. outcome C. upshot D. result
6. A. demarcated B. bound C. confined D. restrained
7. A. engage B. perform C. carry D. conduct
8. A. affairs B. cases C. issues D. topics
9. A. grabbed B. procured C. gained D. captured
10. A. delivery B. inheritance C. succession D. transfer

Your answer:

1. 2. 3. 4. 5.

6. 7. 8. 9. 10.

Part 2: Read the following text and fill in the blank with ONE suitable word. Write your answers in
corresponding numbered boxes. (15 points)
A sting in the tale
A scorpion stung Peter Marks on the back of his right leg, (1) ________ below the knee, then
continued up that leg and down the (2) ________, he believes, before getting him again in the shin. It
wasn’t (3) ________ he was expecting on a flight from Chicago to Vermont. Marks, a 46-year-old
builder, was abroad the United Airlines flight on the second leg of his trip home from San Francisco
where he and his wife Helena had been visiting their sons. He awoke (4) ________ a nap shortly
before landing and noticed something strange.
“My leg felt like it was asleep, but that was isolated to one spot, and it felt as (5) ________ it was
being jabbed with a sharp piece of plastic (6) ________ something. The second sting came after the
plane had landed and the Marks were waiting for their bags at the luggage carousel. Peter rolled up
his cuff to investigate, and the scorpion fell out.
“It felt like a shock, a tingly thing. Someone screamed, ‘It’s a scorpion’,” Peter recalled. Another
passenger stepped on the 5-centimetre arachnid, and (7) ________ else suggested Marks seek
medical help. “The airlines tell you that you can’t bring water on a plane”, Helena Marks said, “but the
scorpion did make it abroad”. A United spokesperson said the incident “is something that we will look
(8) ________. We’re very sorry for what happened. Our customers’ safety and security is our number
one priority.” (9) ________ incidents are not unheard of. An American Airlines flight was delayed for
an hour in Toronto on Sunday after a passenger was stung by a scorpion that had (10) ________ its
way on board. Paramedics treated the man when the flight landed.

Your answers:

1. 2. 3. 4. 5.

6. 7. 8. 9. 10.
Part 3: Read the following passage and circle the best answer to each of the following questions.
Write your answers in corresponding numbered boxes. (10 points)
The medieval artists didn’t know about perspective; they didn’t want to make their people look like
real, individual people in a real, individual scene. They wanted to show the truth, the eternal quality
of their religious stories. So these artists didn’t need to know about perspective.
In the European Renaissance period, artists wanted to show the importance of the individual person
and his or her possessions and surroundings. A flat medieval style couldn’t show this level of reality
and the artists needed a new technique. It was the Italian artist Brunelleschi who discovered the
technique of perspective drawing. At first the artists of the Renaissance only had single-point
perspective. Later they realized that they could have two-pointed perspectives and still later multi-
point perspective.
With two-point perspective they could turn an object like a building at an angle to the picture and
draw two sides of it. The technique of perspective which seems so natural to us now is an invented
technique, a part of the “grammar of painting”. Like all bits of grammar there are exceptions about
perspective. For example, only vertical and horizontal surfaces seem to meet on eye level. Sloping roof
tops don’t meet on eye level.
For 500 years, artists in Europe made use of perspective drawing in their pictures. Nevertheless, there
are a range of priorities that artists take in displaying individual styles. Crivelli wanted to show depth in
his picture and he used a simple single-point perspective. Cezanne always talked about space and
volume. Van Gogh, like some of the other painters of the Impressionist period, was interested in
Japanese prints. And Japanese artists until this century were always very strong designers of “flat”
pictures. Picasso certainly made pictures which have volume and depth. However, he wanted to keep
our eyes on the surface and to remind us that his paintings are paintings and not illusions.
It is technically easy to give an illusion of depth. However, a strong two dimensional design is just as a
feeling of depth, and perhaps more important.
1. The passage mainly discusses
A. the difference between medieval and Renaissance art.
B. how the technique of perspective influenced the modern art.
C. the discovery of the technique of perspective.
D. the contribution of Renaissance artists.
2. The word “eternal” in line 3 is closest in meaning to
A. timeless B. infinite C. frequent D. constant
3. According to the passage, which is the main concern for medieval artists?
A. The individual person and his/ her possessions and surroundings.
B. Real people, real scenes.
C. Eternal timeless truth of the earth.
D. Themes of religious stories.
4. The discovery of perspective was the result of
A. Renaissance artists’ to prove that the medieval artists could show level of reality.
B. the need to turn an object at an angle and draw more than one side of it.
C. the subject being shifted from religious stories to individual person and surroundings.
D. natural evolution of human senses.
5. The word “it” in bold in paragraph 3 refers to
A. the picture B. perspective C. angle D. the object
6. The word “grammar” in bold in paragraph 3 is closest in meaning to
A. construction B. grammatical rules
C. rules and regulations D. tones and volume
7. The author’s purpose to give the example in the last two sentences of paragraph 3 is to
A. explain how perspective work in painting.
B. support two-pointed perspective.
C. illustrate that there are exceptions about perspective.
D. point out that the technique of perspective though seems so natural is an invented technique.
8. The following artists’ priorities in style shift away from perspective EXCEPT
A. Crivelli B. Cezanne
C. Japanese artists D. Brunelleschi
9. The word “illusion” in bold in the last paragraph is closest in meaning to
A. deception B. photograph C. decoration D. illustration
10. It can be inferred from the passage that Renaissance artists
A. embraced the medieval style of eternal truth.
B. needed to develop a new approach towards painting to show a new level of reality.
C. were inspired by vertical and horizontal surfaces in inventing the technique of perspective.
D. saw two dimensional design more important than a feeling of depth.
Your answers:
1. 2. 3. 4. 5.

6. 7. 8. 9. 10.

Part 4: Read the following text and do the tasks that follow. Write your answers in corresponding
numbered boxes. (15 points)
Questions 1-13 which are based on Reading Passage 1 below.
Sending money home
the economics of migrant remittances
A
Every year millions of migrants travel vast distances using borrowed money for their airfares and
taking little or no cash with them. They seek a decent job to support themselves with money left over
that they can send home to their families in developing countries. These remittances exceeded $400
billion last year. It is true that the actual rate per person is only about $200 per month but it all adds
up to about triple the amount officially spent on development aid.
B
In some of the poorer, unstable or conflict-torn countries, these sums of money are a lifeline – the
only salvation for those left behind. The decision to send money home is often inspired by altruism –
an unselfish desire to help others. Then again, the cash might simply be an exchange for earlier
services rendered by the recipients or it could be intended for investment by the recipients. Often it
will be repayment of a loan used to finance the migrant’s travel and resettlement.
C
At the first sign of trouble, political or financial upheaval, these personal sources of support do not
suddenly dry up like official investment monies. Actually, they increase in order to ease the hardship
and suffering of the migrants’ families and, unlike development aid, which is channelled through
government or other official agencies, remittances go straight to those in need. Thus, they serve an
insurance role, responding in a countercyclical way to political and economic crises.
D
This flow of migrant money has a huge economic and social impact on the receiving countries. It
provides cash for food, housing and necessities. It funds education and healthcare and contributes
towards the upkeep of the elderly. Extra money is sent for special events such as weddings, funerals or
urgent medical procedures and other emergencies. Occasionally it becomes the capital for starting up
a small enterprise.
E
Unfortunately, recipients hardly ever receive the full value of the money sent back home because of
exorbitant transfer fees. Many money transfer companies and banks operate on a fixed fee, which is
unduly harsh for those sending small sums at a time. Others charge a percentage, which varies from
around 8% to 20% or more dependent on the recipient country. There are some countries where
there is a low fixed charge per transaction; however, these cheaper fees are not applied
internationally because of widespread concern over money laundering. Whether this is a genuine fear
or just an excuse is hard to say. If the recipients live in a small village somewhere, usually the only
option is to obtain their money through the local post office. Regrettably, many governments allow
post offices to have an exclusive affiliation with one particular money transfer operator so there is no
alternative but to pay the extortionate charge.
F
The sums of money being discussed here might seem negligible on an individual basis but they are
substantial in totality. If the transfer cost could be reduced to no more than one per cent, that would
release another $30 billion dollars annually – approximately the total aid budget of the USA, the
largest donor worldwide – directly into the hands of the world’s poorest. If this is not practicable,
governments could at least acknowledge that small remittances do not come from organised crime
networks, and ease regulations accordingly. They should put an end to restrictive alliances between
post offices and money transfer operators or at least open up the system to competition. Alternately, a
non-government humanitarian organisation, which would have the expertise to navigate the elaborate
red tape, could set up a non-profit remittance platform for migrants to send money home for little or
no cost.
G
Whilst contemplating the best system for transmission of migrant earnings to the home country, one
should consider the fact that migrants often manage to save reasonable amounts of money in their
adopted country. More often than not, that money is in the form of bank deposits earning a tiny
percentage of interest, none at all or even a negative rate of interest.
H
If a developing country or a large charitable society could sell bonds with a guaranteed return of three
or four per cent on the premise that the invested money would be used to build infrastructure in that
country, there would be a twofold benefit. Migrants would make a financial gain and see their savings
put to work in the development of their country of origin. The ideal point of sale for these bonds
would be the channel used for money transfers so that, when migrants show up to make their
monthly remittance, they could buy bonds as well. Advancing the idea one step further, why not make
this transmission hub the conduit for affluent migrants to donate to worthy causes in their homeland
so they may share their prosperity with their compatriots on a larger scale?

Questions 1-7
Reading Passage 1 has seven paragraphs, A–G.
Choose the correct heading for paragraphs B–H from the list of headings below.
Write the correct number, i–x, in boxes 1–7 on your answer sheet.
List of Headings
i Stability of remittances in difficult times
ii Effect of cutback in transaction fees
iii Targeted investments and contributions
iv Remittances for business investment
v How to lower transmission fees
vi Motivations behind remittances
vii Losses incurred during transmission
viii Remittances worth more than official aid
ix How recipients utilise remittances
x Frequency and size of remittances
xi Poor returns on migrant savings
Example:
Paragraph G xi
1 Paragraph A
2 Paragraph B
3 Paragraph C
4 Paragraph D
5 Paragraph E
6 Paragraph F
7 Paragraph H
Questions 8–13
Complete the summary below.
Choose NO MORE THAN TWO WORDS from the passage for each answer.
Write your answers in boxes 8–13 on your answer sheet.
Countries are unwilling to enforce lower transaction fees as they are worried about 8……………….., and
villagers lose out when post offices have a special relationship with one particular money transfer
agency.
Each remittance might be small but the total cost of remittance fees is huge. Governments
should 9……………….. on small amounts and end the current post office system or make it more
competitive. Another idea would be for a large non-profit association, capable of handling
complicated 10……………….. to take charge of migrant remittances.
Migrants who send money home are able to save money, too, but it receives little or no interest
from 11………………… If a country or organisation sold bonds that earned a reasonable rate of interest
for the investor, that money could fund the development of homeland 12………………… The bonds could
be sold at the remittance centre, which could also take donations from 13..……………….. to fund
charitable projects in their home country.
D. WRITING (50 points)
Part 1: Rewrite each sentence using the word in brackets so that the meaning stays the same. You
must use between TWO and SIX words, including the word given. (10 points)
1. Spending two weeks in hospital was very different from lying on the beach! (CRY)
Spending two weeks in hospital was …………………………………lying on the beach!
2. The parents of that girl are furious about her expulsion. (ARMS)
That’s the girl ……………………………………………………. about her expulsion.
3. I’m finding it difficult to cope with all the work I have to do.
(TOP)
All the work I have to do ………………………………………………. me.
4. I wouldn’t confront the manager if I were you.
(HORNS)
I wouldn’t ………………………………………. the manager if I were you.
5. He was arrested when they caught him driving a stolen vehicle.
(ACT)
Being ……………………………………………….. a stolen vehicle, he was arrested.

Part 2: Finish each of the following sentences in such a way that it means exactly the same as the
sentence printed before it, beginning as shown. (10 points)
1. I left without saying goodbye as I didn’t want to disturb the meeting.
Rather ………………………………………………………………………………..
2. You can eat as much as you like for £5 at the new lunch-bar.
There is no
3. We regret to inform you that your application has not been successful.
Much to
4. “I think the whole idea’s ridiculous,” he said.
He dismissed
5. The managing director refuses to resign.
The managing director has no
TEST 14
Part 1. Listen and complete the notes below. Write NO MORE THAN TWO WORDS OR A NUMBER
for each answer. Write your answers in the corresponding numbered boxes. (10 points)

Notes on University 2
Subject: International Business
Qualification: MIB

Entry requirements
Educational qualifications 1._______________________
English language 7 in IELTS or higher.

Course hours: 2 ._______________________ per week.


Extended stay: at the start of the 3 . _______________________

Course dates
Semester 1: 27th September to 22nd January.
Semester 2: 7th February to 4 ______________________ th May.

Course content
Study of 5 _______________________ in particular how they are managed and their changing
external context.

Your answers:

1. 2. 3. 4. 5.

Part 2. You will listen to the whole documentary, and for questions 1-5, choose the best answer (A,
B or C). Write your answers in the corresponding numbered boxes. (10 points)

1. In the past, science fiction fans imagined that jetpack


A. would become a part of everyday life.
B. would be used on the moon.
C. would only be flown by a few people.
2. What was wrong with the Rocket Belt developed by Wendell Moore?
A. It was too slow.
B. It couldn’t fly far enough.
C. It was too fast.
3. The thing which causes most difficulty for a pilot of a jetpack is
A. the terrible heat.
B. keeping stable.
C. trying not to land on water.
4. What is the main advantage of the EFV over a jetpack?
A. It can fly much faster.
B. It is much less heavy.
C. It can fly much further.
5. The company that makes the EFV also makes
A. a type of helicopter.
B. an unmanned flying vehicle.
C. a moon buggy.
Your answers:

1. 2. 3. 4. 5.

Part 3. Listen to an extract from a radio programme about the climber Annabelle Bond, and say
whether sentences 1-5 are true or false. Write your answers in the corresponding numbered boxes.
(10 points)
1. It took her about a year to climb the seven mountains.
2. Only four men have climbed the peaks faster than Annabelle.
3. Annabelle always wanted to be a marathon runner.
4. She raised £8,500 for a cancer charity.
5. She plans to do more climbing in the near future.
Your answers:

1. 2. 3. 4. 5.

Part 4: You will hear an interview between interviewer and Mick Davidson, a member of the ADG.
For questions 1-10, complete the sentences with NO MORE THAN THREE WORDS for each gap. Write
your answers in the corresponding numbered boxes. (20 points)

1. Animal rights protesters destroyed expensive _______________ at a research laboratory.


2. Davidson believes that using animals in experiments is a _______________.
3. Firms need a lot of _______________ to set up experiments.
4. Davidson hasn’t got any shoes that are made of _______________.
5. Newspapers publish _______________ that Mick Davidson has written.
6. Davidson damaged _______________ in a shop in London.
7. In one illegal action, Davidson removed _______________ from a laboratory, which halted the
research.
8. In the attack on a laboratory, Davidson and his ADG colleagues took _______________ away with
them.
9. Davidson doesn’t support the use of _______________ , except against property.
10. The ADG has apologised to people that they have _______________ without meaning to.
Your answers:

1. 5. 9.

2. 6. 10.

3. 7.

4. 8.

B. LEXICO - GRAMMAR (50 points)


Part 1. Choose one of the words marked A, B, C, or D which best completes each of the following
sentences. Write your answers in the corresponding numbered boxes. (20 points)
1. It seems our application has been refused ________.
A. point blank B. carte blanche C. bull’s eye D. long shot
2. The silence of the public library was _________ by the sound of transistor radio.
A. smashed B. fractured C. crushed D. broken
3. You could tell that she wasn’t happy about the news by the way she ________ her face in
disapproval.
A. crossed out B. blew up C. blacked out D. screwed up
4. Although we cannot be sure of the exact purposes of the early cave paintings, pictorial
images seem to be ________ linked to human culture as we understand it.
A. inexorably B. inextricably C. indubitably D. indomitably
5. He gave up hope of passing the examination, __________.
A. it failing twice already B. having already failed it twice
C. already to have failed twice D. it had been failed twice already
6. It is far too easy to lay the blame ________ on the shoulders of the management.
A. solidly B. flatly C. rightly D. squarely
7. __________ by venting about all the reasons you’re leaving is not a good idea when you
do decide to quit your job.
A. Burning bridges B. Wagging the dog
C. Giving yourself airs D. Passing the buck
8. Until this evening, Hannah is still ________ away at her unfinished report.
A. hammering B. grinding C. whiling D. axing
9. Although ________, she tried her best to realize her dream of becoming a sports woman.
A. being disabled B. her disability C. she is disabled D. disabled
10. Anticipating renewed rioting, the authorities erected ______ to block off certain streets.
A. barrages B. barricades C. ditches D. dykes
11. It’s time we change the __________ regarding the roles in the family.
A. idée fixe B. hoi polloi C. avant garde D. faux pas
12. The government intends to introduce a new bill on taxation, __________.
A. whose provisions will be the work of experts on both sides of the House will be study
B. of which the study of its provisions will be the work of experts on both sides of the House
C. the work of experts on both sides of the House will be the study of its provisions
D. the study of whose provisions will be the work of experts on both sides of the House
13. Apart from a parking fine ten years before, she had an __________ driving record.
A. unsullied B. unalloyed C. unblemished D. untarnished
14. Everyone was ___________ with him after he missed the penalty in the last 2 minutes
of the final match.
A. lamenting B. commiserating C. condoling D. identifying
15. We shall need also to ________ the foundations of mathematics, and even to question
the very nature of physical reality.
A. pertain to B. touch on C. delve into D. slur over
16. When she saw me nearly dropped the precious vase, she _________ in horror.
A. grasped B. grunted C. grumbled D. gasped
17. They have _________ a beautiful film by making these changes.
A. disfigured B. mutilated C. defaced D. amputated
18. You can try reformatting your computer, but once you open that _________, you’ll
probably be working on it for days.
A. apple of discord B. can of worms C. load of cobblers D. spot of bother
19. _________ nocturnal creature(s).
A. Owl is a B. An owl is a C. The owl is a D. The owls are
20. It’s natural for students to worry about whether they will ________ or not at a new
school.
A. measure up B. sprout up C. show up D. dig up
Your answers:

1. 2. 3. 4. 5.

6. 7. 8. 9. 10.

11. 12. 13. 14. 15.

16. 17. 18. 19. 20.

Part 2. Read the following text which contains 10 mistakes. Identify the mistakes and write the
corrections in the corresponding numbered boxes. (10 points)
Sir Walter Scott was the key figure in creating a colorful image of Scotland’s past, initially with
his bestselling narrative poem, with his even more celebrated novels, the first of which was Waverley.
It was published anonymously in 1814 and, in subsequent years, its successors were described as
being ‘by the author of Waverley’, which accounts the term ‘Waverley novels’. Although Scott gave no
public acknowledgement of his authorship until 1827, the writer’s identity was an open secret long
before then. He wrote ordinarily quickly, and the first collected edition of the Waverley novels was
published as early as 1819. The set of illustrations of Alexander Nasmyth was produced for the second
collected edition and these drawings used on the title pages.
Nasmyth has been called the father of landscape painting and, such as Walter Scott, he helped to
popularized his country’s romantic and picturesque scenery. The drawings were recently presented to
the National Library of Scotland, which now boasts a superb and rivalled collection of manuscripts and
papers relating to Scott and his circle.
What is less well known about Sir Walter Scott is that after his bankruptcy in 1826, his last years were
spent in frantic literary activity to pay off all the creditors whom he owned money.
Your answers:

Line Mistakes Corrections

1.

2.

3.

4.
5.

6.

7.

8.

9.

10.

Part 3. Give the correct form of the words in brackets. Write your answers in the corresponding
numbered boxes. (10 points)
THE SPIRAL AND THE HELIX
They are everywhere, graceful, curving shapes whose incredible (1) REGULAR _________ contrasts so
sharply with the random world around them. We call them spirals and helices but that hardly does
(2) JUST ____________ to their diversity or their significance.
Over the centuries, (3) MATHEMATICS __________ have identified many different types, but the
most intriguing are those that (4) REPEAT ____________ occur in the natural world.
The need to (5) RAVEL _____ the mysteries of the existence of spirals and helices has exercised some
of the best scientific brains in the world and opened the way to a number of
(6) BREAK ______________ in fields as widely varied as genetics and (7) METEOR ______________.
The most (8) SPECTACLE ________________ spirals on earth are also the most unwelcome -
hurricanes. Their (9) AWE _______________ power comes from the sun's heat, but they owe their
shape to the force caused by the rotation of the earth. After innumerable years of study, however,
Nature's spirals and helices have yet to (10)_____________ all their secrets. For example, why,
astronomers wonder, are so many galaxies spiral-shaped?

Your answers:

1. 5. 9.

2. 6. 10.

3. 7.

4. 8.

C. READING (60 points)


Part 1. Read the following passage and decide which answer (A, B, C, or D) best fits each gap. Write
your answers in corresponding numbered boxes. (10 pts)
Film directors usually make the least promising subjects for biography. They tend to stay
behind the camera and get on with making films, emerging only to make the particular
promotional statement. Only rarely is a film-maker interesting enough to 1_________
biographical interest, and some pay off the attention handsomely. What biographer could
2________ analyzing Hitchcock, Woody Allen or Polanski? These directors, in any case,
were themselves sufficiently absorbed in their own 3__________ to cross over the other
side of the camera and 4__________ themselves to the public.
Much of Jean Renoir’s public profile is 5___________ on his appearance in his final film.
But judging by the most recent biography, by Ronald Bergan, the man was simply not
interesting. He grew up in the benevolent 6________ of his painter father, against whom
he appears not to have 7__________ in any way, emerged to make his own 8_________
in the early life and went on making films for most of the rest of his life. Even when
9_________ with war, Renoir seems to have 10__________ through his career with
equanimity.
1. A. award B. prize C. value D. merit
2. A. oppose B. resist C. confront D. expel
3. A. view B. image C. trend D. expression
4. A. display B. examine C. allow D. distinguish
5. A. described B. based C. imagined D. gathered
6. A. memory B. vision C. shadow D. regard
7. A. rebelled B. related C. referred D. resisted
8. A. spot B. field C. mark D. point
9. A. faced B. tackled C. charged D. opposed
10. A. flown B. sailed C. wandered D. run
Your answers

1. 2. 3. 4. 5.
6. 7. 8. 9. 10.

Part 2. Read the text below and think of the word which best fits each space. Use only ONE word in
each space. Write your answers in the corresponding numbered boxes. (15 points)

Along with London’s West End theatres, New York’s Broadway theatres are thought to (2) _________
the pinnacle of theatrical production in the English-speaking world. For most American actors, landing
a role in one of these productions is far (2) _________ their wildest dreams, as working on Broadway
represents the highest (3) _________ in any theatrical actor’s career. Naturally, all actors must
keep their options (4) _________ when seeking theatrical work. For those just about to (5)
_________ the plunge, it might be wise to first pursue a role in what’s known as the Off-Broadway
theatres, or even better, Off-Off-Broadway theatres. These two types of theatre are defined by seating
capacity - the former being 100 to 499 seats, the latter (6) _________ 100. While the productions are
smaller, performances in these theatres can still (7) _________ respect from the theatrical
community. An actor can use the venues to get their craft down to a fine (8) _________ and
eventually turn in performances eliciting reviews (9) _________ of praise from critics who attend.
Conversely, there’s less need for an actor to worry too much about a bad performance. As it’s only an
Off-Off-Broadway production, it’s not the (10) _________ of the world.

Your answers

1. 2. 3. 4. 5.

6. 7. 8. 9. 10.

Part 3. Read the passage and choose the best option A, B, C, or D to answer the questions. Write
your answers in the corresponding numbered boxes. (10 points)

In the course of its history, human inventions have dramatically increased the
average amount of energy available for use per person. Primitive peoples in cold
regions burned wood and animal dung to heat their caves, cook food, and drive off
animals by fire. The first step toward the developing of more efficient fuels was
taken when people discovered that they could use vegetable oils and animal fats in
lieu of gathered or cut wood. Charcoal gave off more intensive heat than wood and
was more easily obtainable than organic fats. The Greeks first began to use coal for
metal smelting in the 4th century, but it did not come into extensive use until the
Industrial Revolution.
In the 1700s, at the beginning of the Industrial Revolution, most energy used in the
United States and other nations undergoing industrialization was obtained from
perpetual and renewable sources, such as wood, water streams, domesticated
animal labor, and wind. These were predominantly locally available supplies. By
mid-1800s, 91 percent of all commercial energy consumed in the United States and
European countries was obtained from wood. However, at the beginning of the
20th century, coal became a major energy source and replaced wood in
industrializing countries. Although in most regions and climate zones wood was
more readily accessible than coal, the latter represents a more concentrated
source of energy. In 1910, natural gas and oil firmly replaced coal as the main
source of fuel because they are lighter and, therefore, cheaper to transport. They
burned more cleanly than coal and polluted less. Unlike coal, oil could be refined to
manufacture liquid fuels for vehicles, a very important consideration in the early
1900s, when the automobile arrived on the scene.
By 1984, non-renewable fossil fuels, such as oil, coal, and natural gas, provided
over 82 percent of the commercial and industrial energy used in the world. Small
amounts of energy were derived from nuclear fission, and the remaining 16
percent came from burning direct perpetual and renewable fuels, such as biomass.
Between 1700 and 1986, a large number of countries shifted from the use of
energy from local sources to a centralized generation of hydropower and solar
energy converted to electricity. The energy derived from non-renewable fossil fuels
has been increasingly produced in one location and transported to another, as is
the case with most automobile fuels. In countries with private, rather than public
transportation, the age of non-renewable fuels has created a dependency on a
finite resource that will have to be replaced.
Alternative fuel sources are numerous, and shale oil and hydrocarbons are just two
examples. The extraction of shale oil from large deposits in Asian and European
regions has proven to be labor consuming and costly. The resulting product is
sulfur-and nitrogen-rich, and large scale extractions are presently prohibitive.
Similarly, the extraction of hydrocarbons from tar sands in Alberta and Utah is
complex. Semi-solid hydrocarbons cannot be easily separated from the sandstone
and limestone that carry them, and modern technology is not sufficiently versatile
for a large-scale removal of the material. However, both sources of fuel may
eventually be needed as petroleum prices continue to rise and limitations in fossil
fuel availability make alternative deposits more attractive.

1.What is the main topic of the passage?


A. Application of various fuels B. Natural resources and fossil fuels
C. A history of energy use D. A historical review of energy rates
2.The phrase “per persion” is close in meaning to
A. per capita B. per year C. per family D. per day
3.It can be inferred from the first paragraph that ______.
A. coal mining was essential for primitive peoples
B. the Greeks used coal in industrial productions
C. the development of efficient fuel was a gradual process
D. the discovery of efficient fuels was mostly accidental
4. The phrase “in lieu” is closest in meaning to
A. in spite B. in place C. in every way D. in charge
5 .The author of the passage implies that in the 1700s, sources of energy were ______.
A. used for commercial purposes
B. used in various combinations
C. not derived from mineral deposits
D. not always easy to locate
6 .According to the passage, what was the greatest advantage of oil as fuel?
A. It was a concentrated source of energy.
B. It was lighter and cheaper than coal.
C. It replaced wood and coal and reduced pollution.
D. It could be converted to automobile fuel.
7. According to the passage, the sources of fossil fuels will have to be replaced because _____ .
A. they need to be transported B. they are not efficient
C. their use is centralized D. their supply is limited.
8. It can be inferred from the passage that in the early 20th centurgy, energy was obtained primarily
from______.
A. fossil fuels B. nuclear fission
C. hydraulic and solar sources D. burning biomass
9. The author of the passage implies that alternative sources of fuel are currently ______.
A. being used for consumption B. being explored
C. available in few locations D. examined on a lar
10. The word “prohibitive” is closest in meaning to
A. prohibited B. provided
C. too expensive D. too expedient

Part 4. Read the text and do the following tasks. (15 points)
For questions 1-6, choose the correct heading for paragraphs A-F from the list of headings below.
Write your answers in the corresponding numbered boxes.

The Hollywood Film Industry


A This chapter examines the ‘Golden Age’ of the Hollywood film studio system and
explores how a particular kind of filmmaking developed during this period in US film
history. It also focuses on the two key elements which influenced the emergence of the
classic Hollywood studio system: the advent of sound and the business ideal of vertical
integration. In addition to its historical interest, inspecting the growth of the studio
system may offer clues regarding the kinds of struggles that accompany the growth of
any new medium. It might, in fact, be intriguing to examine which changes occurred
during the growth of Hollywood studio, and compare those changes to comtemporary
struggles in which production companies are trying to define and control emerging
industries, such as online film and interactive television.
B The shift of the industry away from ‘silent’ films began during the late 1920s. Warner
Bros.’ 1927 film The Jazz Singer was the first to feature synchronized speech, and with it
came a period of turmoil for the industry. Studios now had proof that ‘talkie’ films
would make them money, but the financial investment this kind of filmmaking would
require, from new camera equipment to new projection facilities, made the studios
hesitant to invest at first. In the end, the power of cinematic sound to both move
audiences and enhance the story persuaded studios that talkies were worth investing
in. Overall, the use of sound in film was well-received by audiences, but there were still
many technical factors to consider. Although full integration of sound into movies was
complete by 1930, it would take somewhat longer for them to regain their stylistic
elegance and dexterity. The camera now had to be encased in a big, clumsy,
unmoveable soundproof box. In addition, actors struggled, having to direct their speech
to awkwardly-hidden microphones in huge plants, telephones or even costumes.
C Vertical integration is the other key component in the rise of the Hollywood studio
system. The major studios realized they could increase their profits by handling each
stage of a film’s life: production (making the film), distribution (getting the film out to
people) and exhibition (owning the theaters in major cities where films were shown
first). Five studios, ‘The Big Five’, worked to achieve vertical integration through the late
1940s, owning vast real estate on which to construct elaborate sets. In addition, these
studios set the exact terms of films’ release dates and patterns. Warner Bros.,
Paramount, 20th Century Fox, MGM and RKO formed this exclusive club. ‘The Little
Three’ studios - Universal, Columbia and United Artists - also made pictures, but each
lacked one of the crucial elements of vertical integration. Together these eight
companies operated as a mature oligopoly, essentially running the entire market.
D During the Golden Age, the studios were remarkably consistent and stable
enterprises, due in large part to long-term management heads - the infamous ‘movie
moguls’ who ruled their kingdoms with iron fists. At MGM, Warner Bros, and Columbia,
the same men ran their studios for decades. The rise of the studio system also hinges
on the treatment of stars, who were constructed and exploited to suit a studio’s image
and schedule. Actors were bound up in seven-year contracts to a single studio, and the
studio boss generally held all the options. Stars could be loaned out to other production
companies at any time. Studio bosses could also force bad roles on actors, and
manipulate every single detail of stars’ images with their mammoth in-house publicity
departments. Some have compared the Hollywood studio system to a factory, and it is
useful
to remember that studios were out to make money first and art second.
E On the other hand, studios also had to cultivate flexibility, in addition to consistent
factory output. Studio heads realized that they couldn’t make virtually the same film
over and over again with the same cast of stars and still expect to keep turning a profit.
They also had to create product differentiation. Examining how each production
company tried to differentiate itself has led to loose characterizations of individual
studios’ styles. MGM tended to put out a lot of all-star productions while Paramount
excelled in comedy and Warner Bros, developed a reputation for gritty social realism.
20th Century Fox forged the musical and a great deal of prestige biographies, while
Universal specialized in classic horror movies.
F In 1948, struggling independent movie producers and exhibitors finally triumphed in
their battle against the big studios’ monopolistic behavior. In the United States versus
Paramount federal decree of that year, the studios were ordered to give up their
theaters in what is commonly referred to as ‘divestiture’ - opening the market to
smaller producers. This, coupled with the advent of television in the 1950s, seriously
compromised the studio system’s influence and profits. Hence, 1930 and 1948 are
generally considered bookends to Hollywood’s Golden Age.

For question 1- 4 : The reading passage has seven paragraphs A - G. Choose the correct heading for
paragraphs A - G from the list of headings below.

List of Headings
i. The power with each studio
ii. The movie industry adapts to innovation
iii. Contrast between cinema and other media of the time
iv. The value of studying Hollywood’s Golden Age
v. Distinguishing themselves from the rest of the market
vi. A double attack on film studios’ power
vii. Gaining control of the industry
viii The top movies of Hollywood’s Golden Age

Example: Paragraph A ______iv____


1. Paragraph B __________
2. Paragraph C __________
Example: Paragraph D ____i_____
3. Paragraph E __________
4. Paragraph F __________
For question 5- 7: Do the following statements agree with the information given in Reading
Passage?
TRUE if the statement agrees with the information
FALSE if the statement contradicts the information
NOT GIVEN if there is no information on this
5. After The Jazz Singer came out, other studios immediately began making movies with synchronized
sound.
6. There were some drawbacks to recording movie actors’ voices in the early 1930s.
7. There was intense competition between actors for contracts with the leading studios.
For questions 8 – 10, choose NO MORE THAN TWO WORDS from the passage for each answer.
THE HOLLYWOOD STUDIOS
Throughout its Golden Age, the Hollywood movie Industry was controlled by a handful of studios.
Using a system known as (8) ______________, the biggest studios not only made movies, but handled
their distribution and then finally showed them in their own theaters. These studios were often run by
autocratic bosses - men known as (9) ______________, who often remained at the head of
organisations for decades. However, the domination of the industry by the leading studios came to an
end in 1948, when they were forced to open the market to smaller producers - a process known
as (10) ______________.
Your answers

1. 2. 3. 4. 5.

6. 7. 8. 9. 10.

D. WRITING (40 points)


Part 1. Finish each of the following sentences in such a way that it means the same as the one
printed before it. Write your answers in the space provided. (10 points)

1. They believe that Oliver failed his exam because he was nervous.
--> Oliver’s failure _______________________________________________________
2. The inhabitants were far worse-off twenty years ago than they are now.
--> The inhabitants are nowhere ___________________________________________
3. If you don't know the art market, there's a risk you will spend a lot of money on rubbish.
--> If you don't know the art market, you are _____________________________________
4. Whatever the methods used to obtain the result, drugs were definitely not involved.
-> There was no question ___________________________________________
5. Those terrapins which survive their first year may live to be twenty.
-> Should _________________________________________

Part II. Rewrite the sentences below in such a way that their meanings stay the same. You must use
the words in capital without changing their forms. Write your answers in the space provided (5
points)
6. Every student will get good marks to express their gratitude towards teachers. (lengths)
_________________________________________
7 I am determined to become a teacher of maths. (heart)
_________________________________________
8 Some of the patients taken to the hospital have got an infectious disease. (diagnosed)
_________________________________________
9 This contract is as important and confidential as that one. (equally)
_________________________________________
10 He has called the meeting in order to raise money for the latest storm. (purpose)
_________________________________________
TEST 15
A. LISTENING: (50 pts)
Part 1.
You will hear a man telephoning to ask about a job in a hotel. For questions 1-5, write NO MORE
THAN THREE WORDS for each gap.
West Bay Hotel – details of job
Newspaper advert for 1. …………….
Vacancies for 2. ………..............
Two shifts
Can choose your 3. ………………………
Pay £5.50 per hour, including a 4. ……………….
A 5. ……………………. is provided in the hotel
Total weekly pay £231
Part 2.
You will hear two people discussing a survey connected to the “nanny state.” For each of the
following questions, choose the option which best fits to what you hear. You will listen to the
recording twice.
1 What does the man say about the woman’s opinion of the “nanny state?”
A He used to agree with it.
B It may not be widely shared.
C It isn’t logical.
2 The man says that the survey indicates that most people ________.
A have changed their minds about the “nanny state”
B feel that there is no such thing as the “nanny state”
C want the government to tell them what to do on certain issues
3 The woman believes that government action on various health issues ________.
A is the right thing for the government to do
B shows that the “nanny state” can be a good thing
C annoys a great many people
4 The woman thinks that the survey results ________.
A suggest that people have the wrong attitude
B show that people have become very confused
C do not indicate approval of the “nanny state”
5 The woman says that the report in the paper ________.
A may change people’s view on the “nanny state”
B won’t be believed by most readers
C has interpreted people’s opinions incorrectly
Part 3.
You will hear a conversation between a boy, William and a girl Sophie in a music shop. Listen and
decide if the following sentences are True (T) or False (F). Write your answers in the corresponding
numbered boxes.
1. Sophie’s mother works in the town where they live. _________
2. Sophie enjoys shopping in Birmingham. _________
3. William feels confidents about finding his way around Birmingham. _________
4. William can persuade Sophie to buy a different CD from him. _________
5. Sohie is disappointed to have her birthday present early. _________
Part 4
You will hear a radio presenter called Jim Dunne talking about local entertainment options. For
questions 1-10, write NO MORE THAN THREE WORDS for each gap.

Jim recommends Pagagnini to (1) ........................................


Jim saw a Pagagnini concert last year in (2) ........................................
As well as classical music, Pagagnini play rock, blues and (3) ........................................
Jim says the Pagagnini show has no (4) ........................................
The first performance of Rhythm of the Dance was in (5) ........................................ in Norway.
More than (6) ........................................ people have seen Rhythm of the Dance live on stage.
Jim suggests listeners look at the section entitled (7) ........................................ on the Rhythm of the
Dance website.
The name of the first show that the Cirque Éloize performed at the Regent Theatre
was (8) ........................................
Some performers in iD appear on (9) ........................................ and Rollerblades.
One review of iD says it is full of originality, energy and (10) ........................................
B. LEXICO-GRAMMAR (50 points)
Part 1. Choose the best option A, B, C, or D to complete the following sentences and write your
answers in the corresponding numbered boxes.
1. I am going to go round the _________ if they don’t turn that music down soon.
A. bell B. bend C. leg D. stock
2. I was so tired that I couldn’t even think _________
A. mind B. doubt C. focus D. straight
3. The cat slept peacefully _________ in the long grass.
A. huddled B. nestled C. snuggled D. cuddled
4. Mr. Wright _________ his vegetable garden very carefully.
A. tends B. grows C. maintains D. attends
5. She _________ a few clothes into the case and hurried to the airport.
A. shoved B. scattered C. piled D. heaved
6. No teenager really knows what _________ for them career-wise in the future.
A. knuckles own B. lies head C. whiles away D. crops up
7. The consultant called in by the firm brought a _________ of experience to bear on the problem.
A. wealth B. realm C. bank D. hoard
8. Several passengers received minor injuries when the train unexpectedly came to a _________
A. delay B. halt C. break D. stand
9. John refused to put his career in _________ by opposing his boss.
A. jeopardy B. hazard C. risk D. stake
10. The area is famous for its _________ gardens, where all kinds of different vegetables are grown for
cash.
A. market B. hothouse C. trade D. greenhouse
11. It is essential to be on the _________ for any signs of movement in the undergrowth since there
are poisonous snakes in the area.
A. guard B. care C. alarm D. alert
12. I could see the lantern _________ in the dark.
A. gleaming B. glowing C. glistening D. glimmering
13. It was a daring robbering, which took place in _________ daylight.
A. broad B. total C. wide D. absolute
14. For my _________ of mind, promise you’ll wear a life jacket in the boat.
A. satisfaction B. contentment C. peace D. calmness
15. By using all the latest technology, the yatchman managed to cross the Atlantic in _________ time.
A. quickest B. lightning C. top D. record
16. I’m afraid Tim doesn’t take much care over his homework. He usually does it _________
A. any old how B. any how C. how on earth D. how come
17. What a mad thing to do! You could all have been killed! It was _________ folly.
A. merely B. only C. sheer D. wild
18. The bark of a tree thickens _________
A. with age B. it gets older C. as older D. by age
19. Widely reproduced in magazines and books, _________
A. Ansel Adams depicted the Western wilderness in his photographs.
B. the Western wilderness was depicted in the photographs of Ansel Adams.
C. Ansel Adam’s photographs deppicted the Western wilderness.
D. it was through his photographs that Ansel Adams depicted the Western wilderness.
20. When I advised you to change jobs, I had your best _________ at heart.
A. feelings B. interests C. thoughts D. aspects
Your answers:

1. 2. 3. 4. 5.

6. 7. 8. 9. 10.

11. 12. 13. 14. 15.

16. 17. 18. 19. 20.


Part 2. Read the passage below which contains 10 mistakes. Identify the mistakes and write the
corrections in the corresponding numbered boxes.
Example: Line 1: wrongly à wrong
Human memory, formerly believing to be rather inefficient, is really more sophisticated
than that of a computer. Researchers approaching the problem from a variation of
viewpoints have all concluded that there is a great deal more storing in our minds than
has been generally supposed. Dr. Wilder Penfield, a Canadian neurosurgery, proved that
by stimulating their brains electrically, he can elicit the total recall of specific events in
his subjects’ lives. Even dreams and another minor events supposedly forgotten for
many years suddenly emerged in details. Although the physical basic for memory is not
yet understood, one theory is how the fantastic capacity for storage in the brain is the
result of an almost unlimited combination of interconnections between brain cell,
stimulated by patterns of activity. Repeated references with the same information
support recall. In other words, improved performance is the result of strengthening the
chemical bonds in the memory.
Your answers:

Line Mistakes Corrections

1.

2.

3.

4.

5.

6.

7.

8.

9.

10.

Part 3. Complete each sentence with one suitable particle or preposition. Write your answer in the
box provided.
1.Something’s cropped _________, I am afraid I won’t be able to make it this afternoon.
2. They are planning to wind ________ their operation in Greece and concentrate on Eastern Europe.
3. Tina is an authority _________ Byzaantine architecture.
4. His sense of fun has rubbed _________ his children.
5.– “ Will the concert start soon?”
_ “ It should get _________ way any minute now.”
6. Being rich doesn’t count _________ much on a desert island.
7. The company’s announced it’s laying _________ 1,000 workers.
8. Could you lend me some money to tide _________ me to the end of the month?
9. If the business does well, I’ll hopefully be able to take _________ a part-time assistant in the spring.
10. When I was younger I wanted to be an air pilot but I soon went _________ the idea when I
realised I hated flying.
Your answers:

1. 2. 3. 4. 5.

6. 7. 8. 9. 10.

Part 4. Write the correct form of the words given in the brackets. Write your answers in the spaces
provided below.
WITH MANY THANKS
Many people have given (0. ASSIST) ...assistance… to me during the writing of this book, but it is to
Miss Leigh Keith, senior editor of Ramsay and Brown that I am most deeply (1. DEBT) _________ for
her loyalty and (2. DEVOTE) _________ during the four years the project lasted. She gave her time and
advice (3. STINT) _________ in order for this work to be completed, giving both moral and (4.
PRACTICE) _________ support for the lengthy research into social conditions the project (5.
NECESSARY) _________ Her assurance and encouragement sustained me in my (6. BELIEVE)
_________ that this was valuable work and it was (7. DOUBT) _________ what enabled me to
continue in the face of often discouraging circumstances. I must also thank my father, who has been a
(8. WILL) _________ collaborator in all my efforts and who spent long hours in libraries and on trains
to distant parts of the country in search of material. I know that he will say that he enjoyed it, but
without his (9. FLAG) _________ enthusiasm this book would never have been written. Finally, I would
like to thank my friends and family, who have had to put up with what must have seemed to them an
(10. EXCEPT) _________ long drawn out piece of writing. Thank you, all of you, very much.
Your answers:

1. 2. 3. 4. 5.

6. 7. 8. 9. 10.

C. READING (50 points)


Part 1. Read the following passage and decide which answer (A, B, C, or D) best fits each gap. Write
your answers in corresponding numbered boxes.
WHAT IS GENEALOGY?
Genealogy is a branch of history. It concerns family history, 1______ than the national or
world history studied at school. It doesn’t 2_______ involve drawing a tree, however -
tracing your family history can also 3_______ in learning about your roots and your
identity. The internet enables millions of people worldwide to 4________ information
about their family history, without great 5_______.
People who research their family history often 6______ that it’s a fascinating hobby which
7_______ a lot about where they come from and whether they have famous ancestors.
According to a survey involving 900 people who had researched their family history, the
chances of 8________ a celebrity in your past are one in ten. The survey also concluded
that the 9________ back you follow your family line, the more likely you are to find a
relation who was much wealthier than you are. However, the vast majority of people who
10________ in the survey discovered they were better off than their ancestors.
1. A. instead B. rather C. except D. sooner
2. A. clearly B. merely C. rarely D. wholly
3. A. cause B. mean C. result D. lead
4. A. accomplish B. access C. approach D. admit
5. A. fee B. price C. charge D. expense
6. A. describe B. define C. remark D. regard
7. A. reveals B. opens C. begins D. arises
8. A. catching B. acquiring C. discovering D. revealing
9. A. older B. greater C. higher D. further
10. A. attended B. participated C. included D. associated
Your answer:

1. 2. 3. 4. 5.

6. 7. 8. 9. 10.

Part 2. Read the following text and fill in the blank with ONE suitable word. Write your answers in
corresponding numbered boxes.
TREES
All over the world, forests are safeguarding the health of the planet 1______. They do this
2______ protecting the soil, providing water, and regulating climate. Trees bind soil
mountain-sides. Hills, 3_______ the trees have been felled, lose 500 times as much soil a
year as those with trees.
Trees catch and store rainwater. Their leaves break the impact of the rains, robbing them
4_______ their destructive power. The roots of trees allow the water to go into the soil,
which gradually releases it to flow down rivers and refill ground-water reserves. Where
there are 5________ trees, the rains run in sheets of water off the land, carrying soil with
them. Land covered with trees and other plants absorbs 20 times more rainwater than bare
earth. As 6________ grow, trees absorb carbon dioxide, the main cause of the
“greenhouse 7________”, which threatens irreversibly to change the world’s climate.
Together, the world’s trees, plants and soils contain three 8________ as much carbon as
there is in the atmosphere.
The world’s forests contain 9_______ vast majority of its animal and plant species. The
tropical rainforests alone have well 10_________ half of them, even though they cover only
about 6% of the Earth’s land surface.
Your answers:

1. 2. 3. 4. 5.

6. 7. 8. 9. 10.
Part 3. Read the following passage and circle the best answer to each of the following questions.
Write your answers in corresponding numbered boxes.
(SOURCE: http://lopngoaingu.com/examinations/index.php?id=reading_comprehension/c-hiu-
toefl-bi-154)
Until recently, most American entrepreneurs were men. Discrimination against women in business,
the demands of caring for families, and lack of business training had kept the number of women
entrepreneurs small. Now, however, businesses owned by women account for more than $40 billion in
annual revenues, and this figure is likely to continue rising throughout the 1990s. As Carolyn Doppelt
Gray, an official of the Small Business Administration, has noted, "The 1970s was the decade of
women entering management, and the 1980s turned out to be the decade of the woman
entrepreneur". What are some of the factors behind this trend? For one thing, as more women earn
advanced degrees in business and enter the corporate world, they are finding obstacles. Women are
still excluded from most executive suites. Charlotte Taylor, a management consultant, had noted, "In
the 1970s women believed if they got an MBA and worked hard they could become chairman of the
board. Now they've found out that isn't going to happen, so they go out on their own".

In the past, most women entrepreneurs worked in "women's" fields: cosmetics and clothing, for
example. But this is changing. Consider ASK Computer Systems, a $22-million-a-year computer
software business. It was founded in 1973 by Sandra Kurtzig, who was then a housewife with degrees
in math and engineering. When Kurtzig founded the business, her first product was software that let
weekly newspapers keep tabs on their newspaper carriers-and her office was a bedroom at home,
with a shoebox under the bed to hold the company's cash. After she succeeded with the newspaper
software system, she hired several bright computer-science graduates to develop additional programs.
When these were marketed and sold, ASK began to grow. It now has 200 employees, and Sandra
Kurtzig owns $66.9 million of stock.
Of course, many women who start their own businesses fail, just as men often do. They still
face hurdles in the business world, especially problems in raising money; the banking and finance
world is still dominated by men, and old attitudes die hard. Most businesses owned by women are still
quite small. But the situation is changing; there are likely to be many more Sandra Kurtzigs in the years
ahead.
1. What is the main idea of this passage?
A. Women today are better educated than in the past, making them more attractive to the business
world.
B. The computer is especially lucrative for women today.
C. Women are better at small businesses than men are.
D. Women today are opening more businesses of their own.
2. The word "excluded" in line 13 is closest in meaning to
A. not permitted in
B. often invited to
C. decorators of
D. charged admission to
3. All of the following were mentioned in the passage as detriments to women in the business world
EXCEPT
A. women were required to stay at home with their families
B. women lacked ability to work in business
C. women faced discrimination in business
D. women were not trained in business
4. In line 17, "that" refers to
A. a woman becomes chairman of the board
B. women working hard
C. women achieving advanced degrees
D. women believing that business is a place for them
5. According to the passage, Charlotte Taylor believes that women in the 1970s
A. were unrealistic about their opportunities in business management
B. were still more interested in education than business opportunities
C. had fewer obstacles in business than they do today
D. were unable to work hard enough to succeed in business
6. The author mentions the "shoebox under the bed" in the third paragraph in order to
A. show the frugality of women in business
B. show the resourcefulness of Sandra Kurtzig
C. point out that initially the financial resources of Sandra Kurtzig's business were limited
D. suggest that the company needed to expand
7. The expression "keep tabs on" in line 24-25 is closest in meaning to
A. recognize the appearance of
B. keep records of
C. provide transportation for
D. pay the salaries of
8. The word "hurdles" in line 33 can be best replaced by
A. fences
B. obstacles
C. questions
D. small groups
9. It can be inferred from the passage that the author believes that businesses operated by women
are small because
A. women prefer a small intimate setting
B. women can't deal with money
C. women are not able to borrow money easily
D. many women fail at large businesses
10. The author's attitude about the future of women in business is
A. skeptical
B. optimistic
C. frustrated
D. negative
Your answers:

1. 2. 3. 4. 5.

6. 7. 8. 9. 10.

Part 4. Read the following text and do the tasks that follow.
Why companies should welcome disorder
A
Organisation is big business. Whether it is of our lives – all those inboxes and calendars – or how
companies are structured, a multi-billion-dollar industry helps to meet this need.
We have more strategies for time management, project management and self-organisation than at
any other time in human history. We are told that we ought to organize our company, our home life,
our week, our day and seven our sleep, all as a means to becoming more productive. Every week,
countless seminars and workshops take place around the world to tell a paying public that they ought
to structure their lives in order to achieve this.
This rhetoric has also crept into the thinking of business leaders and entrepreneurs, much to the
delight of self-proclaimed perfectionists with the need to get everything right. The number of business
schools and graduates has massively increased over the past 50 years, essentially teaching people how
to organise well.
B
Ironically, however, the number of business that fail has also steadily increased. Work-related stress
has increased. A large proportion of workers from all demographics claim to be dissatisfied with the
way their work is structured and the way they are managed.
This begs the question: what has gone wrong? Why is it that on paper the drive for organisation seems
a sure shot for increasing productivity, but in reality, falls well short of what is expected?
C
This has been a problem for a while now. Frederick Taylor was one of the forefathers of scientific
management. Writing in the first half of the 20th century, he designed a number of principles to
improve the efficiency of the work process, which have since become widespread in modern
companies. So the approach has been around for a while.
D
New research suggests that this obsession with efficiency is misguided. The problem is not necessarily
the management theories or strategies we use to organise our work; it’s the basic assumptions we
hold in approaching how we work. Here it’s the assumption that order is a necessary condition for
productivity. This assumption has also fostered the idea that disorder must be detrimental to
organizational productivity. The result is that businesses and people spend time and money organising
themselves for the sake of organising, rather than actually looking at the end goal and usefulness of
such an effort.
E
What’s more, recent studies show that order actually has diminishing returns. Order does increase
productivity to a certain extent, but eventually the usefulness of the process of organisation, and the
benefit it yields, reduce until the point where any further increase in order reduces productivity. Some
argue that in a business, if the cost of formally structuring something outweighs the benefit of doing
it, then that thing ought not to be formally structured. Instead, the resources involved can be better
used elsewhere.
F
In fact, research shows that, when innovating, the best approach is to create an environment devoid
of structure and hierarchy and enable everyone involved to engage as one organic group. These
environments can lead to new solutions that, under conventionally structured environments (filled
with bottlenecks in terms of information flow, power structures, rules, and routines) would never be
reached.
G
In recent times companies have slowly started to embrace this disorganisation. Many of them
embrace it in terms of perception (embracing the idea of disorder, as opposed to fearing it) and in
terms of process (putting mechanisms in place to reduce structure).
For example, Oticon, a large Danish manufacturer of hearing aids, used what it called a ‘spaghetti’
structure in order to reduce the organisation’s rigid hierarchies. This involved scrapping formal job
titles and giving staff huge amounts of ownership over their own time and projects. This approach
proved to be highly successful initially, with clear improvements in worker productivity in all facets of
the business.
In similar fashion, the former chairman of General Electric embraced disorganisation, putting forward
the idea of the ‘boundaryless’ organisation. Again, it involves breaking down the barriers between
different parts of a company and encouraging virtual collaboration and flexible working. Google and a
number of other tech companies have embraced (at least in part) these kinds of flexible structures,
facilitated by technology and strong company values which glue people together.

Questions 1-7
Reading Passage 4 has seven sections, A-G.
Choose the correct heading for each section from the list of headings below.
Write the correct number, i-ix, in boxes 1-7 on your answer sheet.
List of Headings
i Complaints about the impact of a certain approach
ii Fundamental beliefs that are in fact incorrect
iii Early recommendations concerning business activities
iv Organisations that put a new approach into practice
v Companies that have suffered from changing their approach
vi What people are increasingly expected to do
vii How to achieve outcomes that are currently impossible
viii Neither approach guarantees continuous improvement
ix Evidence that a certain approach can have more disadvantages that advantages
1. Section A ___________
2. Section B ___________
3. Section C ___________
4. Section D ___________
5. Section E ___________
6. Section F ___________
7. Section G ___________
Questions 8-10
Do the following statements agree with the information given in Reading Passage?
In boxes 8-10 on your answer sheet, write
TRUE if the statement agrees with the information
FALSE if the statement contradicts the information
NOT GIVEN if there is no information on this
8. Both businesses and people aim at order without really considering its value.
9. Innovation is most successful if the people involved have distinct roles.
10 Google was inspired to adopt flexibility by the success of General Electric.
Your answers:

1. 2. 3. 4. 5.

6. 7. 8. 9. 10.

D. WRITING (50 points)


Part 1: Finish each of the following sentences in such a way that it means exactly the same as the
sentences printed before it.
1. As far as I know, there’s no reason for James to be so unhappy.
→ To the ____________________________________________________________________
2. A rise in temperature in the next century seems likely.
→ In all _____________________________________________________________________
3. If we delay too long, we are unlikely to clinch the deal.
→ The longer_________________________________________________________________
4. The brochure gives hardly any useful information.
→ Precious little _____________________________________________________________
5. You could be arrested for not giving a breath sample to the police.
→ Refusal __________________________________________________________________
Part 2: Use the word(s) given in brackets and make any necessary additions to complete a new
sentence in such a way that it is as similar as possible in meaning to the original sentence. Do NOT
change the form of the given word(s).

1. Jenny isn't a bad accountant, but I don't think it is a very suitable occupation for her.
cut
I just don't think Jenny ___________________________ an accountant.
2. We had no problems at all during our holiday in Turkey.
plan
Everything ___________________________ during our holiday in Turkey.
3. It's possible Clare phoned while we were out.
may
Clare ___________________________ while we were out.
4. Our class has won the History Quiz for the third year running.
succession
For the ___________________________ , our class has won the History Quiz.
5. The thunderstorm brought their tennis match to an abrupt end.
cut
They had to ___________________________ because of the thunderstorm.

You might also like